[obm-l] Re: [obm-l] Re: [obm-l] Re: [obm-l] Teoria dos Números

2020-10-26 Por tôpico Otávio Araújo
De nada mano.

Em seg, 26 de out de 2020 09:40, joao pedro b menezes <
joaopedrobmene...@gmail.com> escreveu:

> Muito obrigado pela ajuda! Entendi o exercício agora.
>
> Em dom, 25 de out de 2020 às 19:59, Otávio Araújo <
> otavio17.ara...@gmail.com> escreveu:
>
>> Vc resolve essa questão mostrando q p=n^2+n+1.  Se n=1 acabou. Se n>1,Já
>> que p divide n^3-1 e é primo, temos que p divide n-1 ou n^2+n+1. Não
>> podemos ter p dividindo n-1 pois n divide p-1 -> n<= p-1 n-1> Portanto p divide n^2+n+1. Faca n^2+n+1 = kp, k inteiro positivo. Temos que
>> kp=n^2+n+1 é congruente a 1 módulo n. Do enunciado temos p congruente a 1
>> módulo n,  mas p é congruente a 1 módulo n e é diferente de 1(pois é primo)
>> -> p>= n+1 e  k será congruente a 1 módulo n também. Suponha que k>1,
>> k>1 implica k>= n+1 daí kp>=(n+1)^2 > n^2+n+1, contradição. Portanto
>> k=1 e p=n^2+n+1.
>>
>> Em dom, 25 de out de 2020 17:37, joao pedro b menezes <
>> joaopedrobmene...@gmail.com> escreveu:
>>
>>> Olá, boa tarde.
>>> Estou com dúvida nesse exercício:
>>> " Sejam n um inteiro positivo maior que 1 e p um primo positivo tal que
>>> n divide p − 1 e p divide n 3 − 1. Mostre que 4p − 3 ´e um quadrado
>>> perfeito."
>>> Já agradeço pela ajuda e pelo tempo!
>>>
>>>


[obm-l] Re: [obm-l] Re: [obm-l] Re: [obm-l] Re: [obm-l] Teoria dos números

2020-03-22 Por tôpico Israel Meireles Chrisostomo
O meu sonho tmbm é esse kk

Em dom., 22 de mar. de 2020 às 13:22, Israel Meireles Chrisostomo <
israelmchrisost...@gmail.com> escreveu:

> vc é engenheiro?
>
> Em dom., 22 de mar. de 2020 às 13:19, Israel Meireles Chrisostomo <
> israelmchrisost...@gmail.com> escreveu:
>
>> mas vc possui algum graduação ?
>>
>> Em dom., 22 de mar. de 2020 às 13:00, Pedro José 
>> escreveu:
>>
>>> Boa tarde!
>>> Perfeita a sua correção.
>>> Quanto ao questionamento, nem tenho formação em matemática, meu sonho é
>>> cursar no IMPA ao me aposentar. Sou pitaqueiro. Ouço um assunto que não
>>> conheço, tento aprendê-lo. Na verdade, gosto de matemática. Talvez seja ela
>>> o "Mundo das ideias", o mundo ideal, a qual Platão se referiu.
>>> Saudações,
>>> PJMS
>>>
>>> Em dom, 22 de mar de 2020 12:25, Israel Meireles Chrisostomo <
>>> israelmchrisost...@gmail.com> escreveu:
>>>
 Acho q tem uma ´pequena correção no seguinte passo "4k+1. Pegando os
 fatores (4n-1)^2 e (4n+1)^2, teremos que 2^6 |p(n) para qualquer n=4k+1."O
 correto seria "Para n=4k+1.Pegando os fatores (n-1)^2 e (n+1)^2"

 Em dom., 22 de mar. de 2020 às 10:14, Israel Meireles Chrisostomo <
 israelmchrisost...@gmail.com> escreveu:

> Primeiramente obrigado pela solução.Mas Pedro, tenho uma pergunta :   o
> sr. é professor de Matemática?
>
> Em dom., 22 de mar. de 2020 às 01:34, Pedro José 
> escreveu:
>
>> Bom dia!
>> Dei uma mancada.
>> O expoente de 3 é 3 e não 2.
>> Retornando às classes mod 3.
>> Ao último fator é côngruo à (n-1)*n
>> Para n=3k aparece outro fator e 3^3|p(n), n=3k.
>> n=3k+1, tenho (n-1)^2 e (n-1), 3^3|p(n), n=3k+1
>> n=3k+2, tenho(n-2)^2 é (n+1)^2, 3^3|p(n), n=3k+2,
>> Logo 3^3|p(n) para todo n inteiro.
>> D>=2^6*3^3*5. Mas D<=2^6*3^3*5, então D=8640
>> Desculpem-me pelo erro.
>> Saudações,
>> PJMS.
>>
>>
>>
>> Em sáb, 21 de mar de 2020 13:20, Pedro José 
>> escreveu:
>>
>>> Boa tarde!
>>> Nem carece método numérico.
>>> Para n=1 ou n=0 ou n=2 temos que qualquer inteiro divide o polinômio
>>> p(n)=(n-2)^2*(n-1)^2*n^2*(n+1)^2*(4n^2-4n-9)
>>>
>>> p(3)=8640
>>> p(4)=561600 então (p(3),p(4))=8640=2^6*3^3*5.
>>> Seja D o maior inteiro que divide p(n) para todo n inteiro, D<=8640
>>> Vamos pegar as classes de equivalência mod 4. Seja k um inteiro.
>>> Para 4k temos que n^2= 16k^2 e (n-2) é par logo (n-2)^2= 4s^2 com s
>>> inteiro. Logo 2^6 divide p(n) para qualquer n =4k.
>>> 4k+1. Pegando os fatores (4n-1)^2 e (4n+1)^2, teremos que 2^6 |p(n)
>>> para qualquer n=4k+1.
>>> 4k+2. Pegando (n-2)^2 e n^2, teremos que 2^6|p(n) para qualquer
>>> n=4k+2
>>> 4k+3, pegando os mesmos fatores de 4k+1, 2^6|p(n) para n=4k+3.
>>> Portanto 2^6|p(n) para qualquer inteiro
>>> Agora classes de equivalência mod 3
>>> 3k, pegando n^2, 3^2|p(n) para n=3k
>>> 3k+1, pegando (n-1)^2; 3^2| p(n), n=3k+1
>>> 3k+2, pegando (n-2)^2, 3^2| p(n), n=3k+2
>>> Daí 3^2|p(n) para qualquer n inteiro.
>>> Classes de equivalência mod 5.
>>> 5k, n^2 , 5 |p(n), n =5
>>> 5k +1, (n-1)^2, 5|p(n), n=5k+1
>>> 5k+2, (n-2)^2, 5|p(n), n=5k+2
>>> 5k+3, (4n^2-4n-9)=(100k^2-100k+15)
>>> 5|p(n), n=5k+3
>>> 5k+4, (n+1)^2, 5|p(n) , n=5k+4.
>>> Então 5|p(n) para todo inteiro
>>> D>=2^6*3^2×*5
>>> Mas D<=2^6*3^2*5, logo D=8640
>>>
>>> Saudações,
>>> PJMS
>>>
>>> Em sáb, 21 de mar de 2020 04:39, Pedro José 
>>> escreveu:
>>>
 Bom dia!
 Falta de novo, em seu questionamento, informar que n é inteiro ou
 natural e colocar a condição para qualquer valor de n. Chamando o 
 polinômio
 de p(n)
 Para n=0, 1 ou 2, qualquer inteiro divide.
 Faria mdc(p(3),p(4))= A1
 Se der "pequeno", com poucos fatores primos e expoentes pequenos.
 Paro em A1, se não.
 (p(5),A1)=A2 uso o mesmo critério de parar
 (p(6),A2)=A3 até parar em:
 Ai=(p(i+3),A(i-1)).
 Aí faço o polinômio módfi^xi, onde fi é um fator primo de Aí e xi
 seu expoente. verifico se para cada resíduon= 1, 2...fi^n-1 se P(n)=0 
 mod
 fi^si
 Se falhar diminuto xi em 1 e repito o teste para todos resíduos de
 fi^(xi-1)-1 até um dado xki em que todos os p(resíduos) foram 
 equivalente a
 zero módulo fi^xki ou quando fizer para o expoente 1  e não zerar para
 todos resíduos de fi, quando o fator será descartado.
 Depois repito para cada fator primo f e seu respectivo expoente.
 Ao final D = Produtório de cada fator fi elevado ao expoente xki
 que zerou p(n) mod fi^xki para todos os resíduos, descartando os fí em 
 que
 xji chegou a 1 e não atendeu ou considerando nesse caso xki=0.

 Mas resolveria por método numérico.
 Depois poste sua solução.

 

[obm-l] Re: [obm-l] Re: [obm-l] Re: [obm-l] Re: [obm-l] Teoria dos números

2020-03-22 Por tôpico Israel Meireles Chrisostomo
vc é engenheiro?

Em dom., 22 de mar. de 2020 às 13:19, Israel Meireles Chrisostomo <
israelmchrisost...@gmail.com> escreveu:

> mas vc possui algum graduação ?
>
> Em dom., 22 de mar. de 2020 às 13:00, Pedro José 
> escreveu:
>
>> Boa tarde!
>> Perfeita a sua correção.
>> Quanto ao questionamento, nem tenho formação em matemática, meu sonho é
>> cursar no IMPA ao me aposentar. Sou pitaqueiro. Ouço um assunto que não
>> conheço, tento aprendê-lo. Na verdade, gosto de matemática. Talvez seja ela
>> o "Mundo das ideias", o mundo ideal, a qual Platão se referiu.
>> Saudações,
>> PJMS
>>
>> Em dom, 22 de mar de 2020 12:25, Israel Meireles Chrisostomo <
>> israelmchrisost...@gmail.com> escreveu:
>>
>>> Acho q tem uma ´pequena correção no seguinte passo "4k+1. Pegando os
>>> fatores (4n-1)^2 e (4n+1)^2, teremos que 2^6 |p(n) para qualquer n=4k+1."O
>>> correto seria "Para n=4k+1.Pegando os fatores (n-1)^2 e (n+1)^2"
>>>
>>> Em dom., 22 de mar. de 2020 às 10:14, Israel Meireles Chrisostomo <
>>> israelmchrisost...@gmail.com> escreveu:
>>>
 Primeiramente obrigado pela solução.Mas Pedro, tenho uma pergunta :   o
 sr. é professor de Matemática?

 Em dom., 22 de mar. de 2020 às 01:34, Pedro José 
 escreveu:

> Bom dia!
> Dei uma mancada.
> O expoente de 3 é 3 e não 2.
> Retornando às classes mod 3.
> Ao último fator é côngruo à (n-1)*n
> Para n=3k aparece outro fator e 3^3|p(n), n=3k.
> n=3k+1, tenho (n-1)^2 e (n-1), 3^3|p(n), n=3k+1
> n=3k+2, tenho(n-2)^2 é (n+1)^2, 3^3|p(n), n=3k+2,
> Logo 3^3|p(n) para todo n inteiro.
> D>=2^6*3^3*5. Mas D<=2^6*3^3*5, então D=8640
> Desculpem-me pelo erro.
> Saudações,
> PJMS.
>
>
>
> Em sáb, 21 de mar de 2020 13:20, Pedro José 
> escreveu:
>
>> Boa tarde!
>> Nem carece método numérico.
>> Para n=1 ou n=0 ou n=2 temos que qualquer inteiro divide o polinômio
>> p(n)=(n-2)^2*(n-1)^2*n^2*(n+1)^2*(4n^2-4n-9)
>>
>> p(3)=8640
>> p(4)=561600 então (p(3),p(4))=8640=2^6*3^3*5.
>> Seja D o maior inteiro que divide p(n) para todo n inteiro, D<=8640
>> Vamos pegar as classes de equivalência mod 4. Seja k um inteiro.
>> Para 4k temos que n^2= 16k^2 e (n-2) é par logo (n-2)^2= 4s^2 com s
>> inteiro. Logo 2^6 divide p(n) para qualquer n =4k.
>> 4k+1. Pegando os fatores (4n-1)^2 e (4n+1)^2, teremos que 2^6 |p(n)
>> para qualquer n=4k+1.
>> 4k+2. Pegando (n-2)^2 e n^2, teremos que 2^6|p(n) para qualquer n=4k+2
>> 4k+3, pegando os mesmos fatores de 4k+1, 2^6|p(n) para n=4k+3.
>> Portanto 2^6|p(n) para qualquer inteiro
>> Agora classes de equivalência mod 3
>> 3k, pegando n^2, 3^2|p(n) para n=3k
>> 3k+1, pegando (n-1)^2; 3^2| p(n), n=3k+1
>> 3k+2, pegando (n-2)^2, 3^2| p(n), n=3k+2
>> Daí 3^2|p(n) para qualquer n inteiro.
>> Classes de equivalência mod 5.
>> 5k, n^2 , 5 |p(n), n =5
>> 5k +1, (n-1)^2, 5|p(n), n=5k+1
>> 5k+2, (n-2)^2, 5|p(n), n=5k+2
>> 5k+3, (4n^2-4n-9)=(100k^2-100k+15)
>> 5|p(n), n=5k+3
>> 5k+4, (n+1)^2, 5|p(n) , n=5k+4.
>> Então 5|p(n) para todo inteiro
>> D>=2^6*3^2×*5
>> Mas D<=2^6*3^2*5, logo D=8640
>>
>> Saudações,
>> PJMS
>>
>> Em sáb, 21 de mar de 2020 04:39, Pedro José 
>> escreveu:
>>
>>> Bom dia!
>>> Falta de novo, em seu questionamento, informar que n é inteiro ou
>>> natural e colocar a condição para qualquer valor de n. Chamando o 
>>> polinômio
>>> de p(n)
>>> Para n=0, 1 ou 2, qualquer inteiro divide.
>>> Faria mdc(p(3),p(4))= A1
>>> Se der "pequeno", com poucos fatores primos e expoentes pequenos.
>>> Paro em A1, se não.
>>> (p(5),A1)=A2 uso o mesmo critério de parar
>>> (p(6),A2)=A3 até parar em:
>>> Ai=(p(i+3),A(i-1)).
>>> Aí faço o polinômio módfi^xi, onde fi é um fator primo de Aí e xi
>>> seu expoente. verifico se para cada resíduon= 1, 2...fi^n-1 se P(n)=0 
>>> mod
>>> fi^si
>>> Se falhar diminuto xi em 1 e repito o teste para todos resíduos de
>>> fi^(xi-1)-1 até um dado xki em que todos os p(resíduos) foram 
>>> equivalente a
>>> zero módulo fi^xki ou quando fizer para o expoente 1  e não zerar para
>>> todos resíduos de fi, quando o fator será descartado.
>>> Depois repito para cada fator primo f e seu respectivo expoente.
>>> Ao final D = Produtório de cada fator fi elevado ao expoente xki que
>>> zerou p(n) mod fi^xki para todos os resíduos, descartando os fí em que 
>>> xji
>>> chegou a 1 e não atendeu ou considerando nesse caso xki=0.
>>>
>>> Mas resolveria por método numérico.
>>> Depois poste sua solução.
>>>
>>> Saudações,
>>> PJMS.
>>>
>>>
>>>
>>>
>>> Em sex, 20 de mar de 2020 12:42, Israel Meireles Chrisostomo <
>>> israelmchrisost...@gmail.com> escreveu:
>>>
 Qual o maior inteiro que divide (n - 2)^2 (n - 1)^2 n^2 (n + 1)^2
 (4 n^2 

[obm-l] Re: [obm-l] Re: [obm-l] Re: [obm-l] Re: [obm-l] Teoria dos números

2020-03-22 Por tôpico Israel Meireles Chrisostomo
mas vc possui algum graduação ?

Em dom., 22 de mar. de 2020 às 13:00, Pedro José 
escreveu:

> Boa tarde!
> Perfeita a sua correção.
> Quanto ao questionamento, nem tenho formação em matemática, meu sonho é
> cursar no IMPA ao me aposentar. Sou pitaqueiro. Ouço um assunto que não
> conheço, tento aprendê-lo. Na verdade, gosto de matemática. Talvez seja ela
> o "Mundo das ideias", o mundo ideal, a qual Platão se referiu.
> Saudações,
> PJMS
>
> Em dom, 22 de mar de 2020 12:25, Israel Meireles Chrisostomo <
> israelmchrisost...@gmail.com> escreveu:
>
>> Acho q tem uma ´pequena correção no seguinte passo "4k+1. Pegando os
>> fatores (4n-1)^2 e (4n+1)^2, teremos que 2^6 |p(n) para qualquer n=4k+1."O
>> correto seria "Para n=4k+1.Pegando os fatores (n-1)^2 e (n+1)^2"
>>
>> Em dom., 22 de mar. de 2020 às 10:14, Israel Meireles Chrisostomo <
>> israelmchrisost...@gmail.com> escreveu:
>>
>>> Primeiramente obrigado pela solução.Mas Pedro, tenho uma pergunta :   o
>>> sr. é professor de Matemática?
>>>
>>> Em dom., 22 de mar. de 2020 às 01:34, Pedro José 
>>> escreveu:
>>>
 Bom dia!
 Dei uma mancada.
 O expoente de 3 é 3 e não 2.
 Retornando às classes mod 3.
 Ao último fator é côngruo à (n-1)*n
 Para n=3k aparece outro fator e 3^3|p(n), n=3k.
 n=3k+1, tenho (n-1)^2 e (n-1), 3^3|p(n), n=3k+1
 n=3k+2, tenho(n-2)^2 é (n+1)^2, 3^3|p(n), n=3k+2,
 Logo 3^3|p(n) para todo n inteiro.
 D>=2^6*3^3*5. Mas D<=2^6*3^3*5, então D=8640
 Desculpem-me pelo erro.
 Saudações,
 PJMS.



 Em sáb, 21 de mar de 2020 13:20, Pedro José 
 escreveu:

> Boa tarde!
> Nem carece método numérico.
> Para n=1 ou n=0 ou n=2 temos que qualquer inteiro divide o polinômio
> p(n)=(n-2)^2*(n-1)^2*n^2*(n+1)^2*(4n^2-4n-9)
>
> p(3)=8640
> p(4)=561600 então (p(3),p(4))=8640=2^6*3^3*5.
> Seja D o maior inteiro que divide p(n) para todo n inteiro, D<=8640
> Vamos pegar as classes de equivalência mod 4. Seja k um inteiro.
> Para 4k temos que n^2= 16k^2 e (n-2) é par logo (n-2)^2= 4s^2 com s
> inteiro. Logo 2^6 divide p(n) para qualquer n =4k.
> 4k+1. Pegando os fatores (4n-1)^2 e (4n+1)^2, teremos que 2^6 |p(n)
> para qualquer n=4k+1.
> 4k+2. Pegando (n-2)^2 e n^2, teremos que 2^6|p(n) para qualquer n=4k+2
> 4k+3, pegando os mesmos fatores de 4k+1, 2^6|p(n) para n=4k+3.
> Portanto 2^6|p(n) para qualquer inteiro
> Agora classes de equivalência mod 3
> 3k, pegando n^2, 3^2|p(n) para n=3k
> 3k+1, pegando (n-1)^2; 3^2| p(n), n=3k+1
> 3k+2, pegando (n-2)^2, 3^2| p(n), n=3k+2
> Daí 3^2|p(n) para qualquer n inteiro.
> Classes de equivalência mod 5.
> 5k, n^2 , 5 |p(n), n =5
> 5k +1, (n-1)^2, 5|p(n), n=5k+1
> 5k+2, (n-2)^2, 5|p(n), n=5k+2
> 5k+3, (4n^2-4n-9)=(100k^2-100k+15)
> 5|p(n), n=5k+3
> 5k+4, (n+1)^2, 5|p(n) , n=5k+4.
> Então 5|p(n) para todo inteiro
> D>=2^6*3^2×*5
> Mas D<=2^6*3^2*5, logo D=8640
>
> Saudações,
> PJMS
>
> Em sáb, 21 de mar de 2020 04:39, Pedro José 
> escreveu:
>
>> Bom dia!
>> Falta de novo, em seu questionamento, informar que n é inteiro ou
>> natural e colocar a condição para qualquer valor de n. Chamando o 
>> polinômio
>> de p(n)
>> Para n=0, 1 ou 2, qualquer inteiro divide.
>> Faria mdc(p(3),p(4))= A1
>> Se der "pequeno", com poucos fatores primos e expoentes pequenos.
>> Paro em A1, se não.
>> (p(5),A1)=A2 uso o mesmo critério de parar
>> (p(6),A2)=A3 até parar em:
>> Ai=(p(i+3),A(i-1)).
>> Aí faço o polinômio módfi^xi, onde fi é um fator primo de Aí e xi seu
>> expoente. verifico se para cada resíduon= 1, 2...fi^n-1 se P(n)=0 mod 
>> fi^si
>> Se falhar diminuto xi em 1 e repito o teste para todos resíduos de
>> fi^(xi-1)-1 até um dado xki em que todos os p(resíduos) foram 
>> equivalente a
>> zero módulo fi^xki ou quando fizer para o expoente 1  e não zerar para
>> todos resíduos de fi, quando o fator será descartado.
>> Depois repito para cada fator primo f e seu respectivo expoente.
>> Ao final D = Produtório de cada fator fi elevado ao expoente xki que
>> zerou p(n) mod fi^xki para todos os resíduos, descartando os fí em que 
>> xji
>> chegou a 1 e não atendeu ou considerando nesse caso xki=0.
>>
>> Mas resolveria por método numérico.
>> Depois poste sua solução.
>>
>> Saudações,
>> PJMS.
>>
>>
>>
>>
>> Em sex, 20 de mar de 2020 12:42, Israel Meireles Chrisostomo <
>> israelmchrisost...@gmail.com> escreveu:
>>
>>> Qual o maior inteiro que divide (n - 2)^2 (n - 1)^2 n^2 (n + 1)^2 (4
>>> n^2 - 4 n - 9))?
>>> Eu sei resolver esse problema com meu algoritmo, porém gostaria de
>>> saber como os colegas o resolveriam.
>>> --
>>> Israel Meireles Chrisostomo
>>>
>>> --
>>> Esta mensagem foi verificada pelo sistema de antivírus e

[obm-l] Re: [obm-l] Re: [obm-l] Re: [obm-l] Teoria dos números

2020-03-22 Por tôpico Pedro José
Boa tarde!
Perfeita a sua correção.
Quanto ao questionamento, nem tenho formação em matemática, meu sonho é
cursar no IMPA ao me aposentar. Sou pitaqueiro. Ouço um assunto que não
conheço, tento aprendê-lo. Na verdade, gosto de matemática. Talvez seja ela
o "Mundo das ideias", o mundo ideal, a qual Platão se referiu.
Saudações,
PJMS

Em dom, 22 de mar de 2020 12:25, Israel Meireles Chrisostomo <
israelmchrisost...@gmail.com> escreveu:

> Acho q tem uma ´pequena correção no seguinte passo "4k+1. Pegando os
> fatores (4n-1)^2 e (4n+1)^2, teremos que 2^6 |p(n) para qualquer n=4k+1."O
> correto seria "Para n=4k+1.Pegando os fatores (n-1)^2 e (n+1)^2"
>
> Em dom., 22 de mar. de 2020 às 10:14, Israel Meireles Chrisostomo <
> israelmchrisost...@gmail.com> escreveu:
>
>> Primeiramente obrigado pela solução.Mas Pedro, tenho uma pergunta :   o
>> sr. é professor de Matemática?
>>
>> Em dom., 22 de mar. de 2020 às 01:34, Pedro José 
>> escreveu:
>>
>>> Bom dia!
>>> Dei uma mancada.
>>> O expoente de 3 é 3 e não 2.
>>> Retornando às classes mod 3.
>>> Ao último fator é côngruo à (n-1)*n
>>> Para n=3k aparece outro fator e 3^3|p(n), n=3k.
>>> n=3k+1, tenho (n-1)^2 e (n-1), 3^3|p(n), n=3k+1
>>> n=3k+2, tenho(n-2)^2 é (n+1)^2, 3^3|p(n), n=3k+2,
>>> Logo 3^3|p(n) para todo n inteiro.
>>> D>=2^6*3^3*5. Mas D<=2^6*3^3*5, então D=8640
>>> Desculpem-me pelo erro.
>>> Saudações,
>>> PJMS.
>>>
>>>
>>>
>>> Em sáb, 21 de mar de 2020 13:20, Pedro José 
>>> escreveu:
>>>
 Boa tarde!
 Nem carece método numérico.
 Para n=1 ou n=0 ou n=2 temos que qualquer inteiro divide o polinômio
 p(n)=(n-2)^2*(n-1)^2*n^2*(n+1)^2*(4n^2-4n-9)

 p(3)=8640
 p(4)=561600 então (p(3),p(4))=8640=2^6*3^3*5.
 Seja D o maior inteiro que divide p(n) para todo n inteiro, D<=8640
 Vamos pegar as classes de equivalência mod 4. Seja k um inteiro.
 Para 4k temos que n^2= 16k^2 e (n-2) é par logo (n-2)^2= 4s^2 com s
 inteiro. Logo 2^6 divide p(n) para qualquer n =4k.
 4k+1. Pegando os fatores (4n-1)^2 e (4n+1)^2, teremos que 2^6 |p(n)
 para qualquer n=4k+1.
 4k+2. Pegando (n-2)^2 e n^2, teremos que 2^6|p(n) para qualquer n=4k+2
 4k+3, pegando os mesmos fatores de 4k+1, 2^6|p(n) para n=4k+3.
 Portanto 2^6|p(n) para qualquer inteiro
 Agora classes de equivalência mod 3
 3k, pegando n^2, 3^2|p(n) para n=3k
 3k+1, pegando (n-1)^2; 3^2| p(n), n=3k+1
 3k+2, pegando (n-2)^2, 3^2| p(n), n=3k+2
 Daí 3^2|p(n) para qualquer n inteiro.
 Classes de equivalência mod 5.
 5k, n^2 , 5 |p(n), n =5
 5k +1, (n-1)^2, 5|p(n), n=5k+1
 5k+2, (n-2)^2, 5|p(n), n=5k+2
 5k+3, (4n^2-4n-9)=(100k^2-100k+15)
 5|p(n), n=5k+3
 5k+4, (n+1)^2, 5|p(n) , n=5k+4.
 Então 5|p(n) para todo inteiro
 D>=2^6*3^2×*5
 Mas D<=2^6*3^2*5, logo D=8640

 Saudações,
 PJMS

 Em sáb, 21 de mar de 2020 04:39, Pedro José 
 escreveu:

> Bom dia!
> Falta de novo, em seu questionamento, informar que n é inteiro ou
> natural e colocar a condição para qualquer valor de n. Chamando o 
> polinômio
> de p(n)
> Para n=0, 1 ou 2, qualquer inteiro divide.
> Faria mdc(p(3),p(4))= A1
> Se der "pequeno", com poucos fatores primos e expoentes pequenos. Paro
> em A1, se não.
> (p(5),A1)=A2 uso o mesmo critério de parar
> (p(6),A2)=A3 até parar em:
> Ai=(p(i+3),A(i-1)).
> Aí faço o polinômio módfi^xi, onde fi é um fator primo de Aí e xi seu
> expoente. verifico se para cada resíduon= 1, 2...fi^n-1 se P(n)=0 mod 
> fi^si
> Se falhar diminuto xi em 1 e repito o teste para todos resíduos de
> fi^(xi-1)-1 até um dado xki em que todos os p(resíduos) foram equivalente 
> a
> zero módulo fi^xki ou quando fizer para o expoente 1  e não zerar para
> todos resíduos de fi, quando o fator será descartado.
> Depois repito para cada fator primo f e seu respectivo expoente.
> Ao final D = Produtório de cada fator fi elevado ao expoente xki que
> zerou p(n) mod fi^xki para todos os resíduos, descartando os fí em que xji
> chegou a 1 e não atendeu ou considerando nesse caso xki=0.
>
> Mas resolveria por método numérico.
> Depois poste sua solução.
>
> Saudações,
> PJMS.
>
>
>
>
> Em sex, 20 de mar de 2020 12:42, Israel Meireles Chrisostomo <
> israelmchrisost...@gmail.com> escreveu:
>
>> Qual o maior inteiro que divide (n - 2)^2 (n - 1)^2 n^2 (n + 1)^2 (4
>> n^2 - 4 n - 9))?
>> Eu sei resolver esse problema com meu algoritmo, porém gostaria de
>> saber como os colegas o resolveriam.
>> --
>> Israel Meireles Chrisostomo
>>
>> --
>> Esta mensagem foi verificada pelo sistema de antivírus e
>> acredita-se estar livre de perigo.
>
>
>>> --
>>> Esta mensagem foi verificada pelo sistema de antivírus e
>>> acredita-se estar livre de perigo.
>>
>>
>>
>> --
>> Israel Meireles Chrisostomo
>>
>
>
> --
> Israel Meireles 

[obm-l] Re: [obm-l] Re: [obm-l] Re: [obm-l] Re: [obm-l] Re: [obm-l] Re: [obm-l] Teoria dos números

2020-03-17 Por tôpico Pedro José
Boa noite!
Aí, como dizia minha falecida vó, são outros quinhentos.
Como nas propostas anteriores n era natural. Vamos seguir nessa linha, se
não for reformule o problema.
Seja f(n)=   n (427 - 90n - 70n^2 + 45n^3 + 18n^4)
f(0)=0 qualquer natural divide, portanto, é indiferente.
f(1)= 330
f(2)= 1230
É fácil verificar que mdc(330,1230)=30 então D<=30, onde D é o máximo
inteiro que divide f(n) para todo n natural.
f(n) = 7n +5n^4 + 8 n^5 mod 10.
f(0)=0 mod10
f(1)= 20 = 0 mod10
f(2)= 350= 0 mod10
f(3)= 2370 = 0 mod10
f(4)= 9500 = 0 mod10
f(5)= 28160 = 0 mod10
f(6)=68730 = 0 mod10
f(7)=146510 = 0 mod10
f(8)=282680 = 0 mod10
f(9)=505260 = 0 mod10
logo 10 | f(n) para qualquer n natural.

f(n) = n -n^3 mod 3
f(0) = 0 mod 3
f(1) = 0 mod 3
f(2)= -6 = 0 mod 3
logo 3| f(n) para todo n natural
então D = 30.

Saudações,
PJMS



Em ter., 17 de mar. de 2020 às 11:57, Israel Meireles Chrisostomo <
israelmchrisost...@gmail.com> escreveu:

> Sim é isso q eu quis dizer
>
> Em ter, 17 de mar de 2020 11:12, Carlos Gustavo Tamm de Araujo Moreira <
> g...@impa.br> escreveu:
>
>> Acho que a pergunta deve ser qual é o maior inteiro positivo que divide
>> essa expressão para todo valor de n ao mesmo tempo.
>>
>> On Tue, Mar 17, 2020 at 6:58 AM Pedro José  wrote:
>>
>>> Bom dia!
>>> Se você considerar a expressão n(427-90n-70n^2+45n^3+18n^4)
>>> D=|n(427-90n-70n^2+45n^3+18n^4)|
>>> Por exemplo, n=1
>>> D=330.
>>> Agora se liberar n para variar D tende a oo.
>>>
>>> Se n for raiz da expressão, também tende a oi, pois qualquer inteiro
>>> divide 0.
>>>
>>>
>>> Em seg, 16 de mar de 2020 22:16, Israel Meireles Chrisostomo <
>>> israelmchrisost...@gmail.com> escreveu:
>>>
 não entendi

 Em seg., 16 de mar. de 2020 às 22:01, Pedro José 
 escreveu:

> Para um dado n é o módulo do valor da expressão.
>
> Em seg, 16 de mar de 2020 21:49, Pedro José 
> escreveu:
>
>> Boa noite!
>> O módulo dessa expressão tende a oo. Não existe máximo.
>> Saudações,
>> PJMS
>>
>> Em seg, 16 de mar de 2020 20:36, Israel Meireles Chrisostomo <
>> israelmchrisost...@gmail.com> escreveu:
>>
>>> Qual é o maior inteiro que divide  n (427 - 90n - 70n^2 + 45n^3 +
>>> 18n^4)?
>>>
>>> --
>>> Israel Meireles Chrisostomo
>>>
>>> --
>>> Esta mensagem foi verificada pelo sistema de antivírus e
>>> acredita-se estar livre de perigo.
>>
>>
> --
> Esta mensagem foi verificada pelo sistema de antivírus e
> acredita-se estar livre de perigo.



 --
 Israel Meireles Chrisostomo

 --
 Esta mensagem foi verificada pelo sistema de antivírus e
 acredita-se estar livre de perigo.
>>>
>>>
>>> --
>>> Esta mensagem foi verificada pelo sistema de antivírus e
>>> acredita-se estar livre de perigo.
>>
>>
>> --
>> Esta mensagem foi verificada pelo sistema de antivírus e
>> acredita-se estar livre de perigo.
>
>
> --
> Esta mensagem foi verificada pelo sistema de antivírus e
> acredita-se estar livre de perigo.

-- 
Esta mensagem foi verificada pelo sistema de antiv�rus e
 acredita-se estar livre de perigo.



[obm-l] Re: [obm-l] Re: [obm-l] Re: [obm-l] Re: [obm-l] Re: [obm-l] Teoria dos números

2020-03-17 Por tôpico Israel Meireles Chrisostomo
Sim é isso q eu quis dizer

Em ter, 17 de mar de 2020 11:12, Carlos Gustavo Tamm de Araujo Moreira <
g...@impa.br> escreveu:

> Acho que a pergunta deve ser qual é o maior inteiro positivo que divide
> essa expressão para todo valor de n ao mesmo tempo.
>
> On Tue, Mar 17, 2020 at 6:58 AM Pedro José  wrote:
>
>> Bom dia!
>> Se você considerar a expressão n(427-90n-70n^2+45n^3+18n^4)
>> D=|n(427-90n-70n^2+45n^3+18n^4)|
>> Por exemplo, n=1
>> D=330.
>> Agora se liberar n para variar D tende a oo.
>>
>> Se n for raiz da expressão, também tende a oi, pois qualquer inteiro
>> divide 0.
>>
>>
>> Em seg, 16 de mar de 2020 22:16, Israel Meireles Chrisostomo <
>> israelmchrisost...@gmail.com> escreveu:
>>
>>> não entendi
>>>
>>> Em seg., 16 de mar. de 2020 às 22:01, Pedro José 
>>> escreveu:
>>>
 Para um dado n é o módulo do valor da expressão.

 Em seg, 16 de mar de 2020 21:49, Pedro José 
 escreveu:

> Boa noite!
> O módulo dessa expressão tende a oo. Não existe máximo.
> Saudações,
> PJMS
>
> Em seg, 16 de mar de 2020 20:36, Israel Meireles Chrisostomo <
> israelmchrisost...@gmail.com> escreveu:
>
>> Qual é o maior inteiro que divide  n (427 - 90n - 70n^2 + 45n^3 +
>> 18n^4)?
>>
>> --
>> Israel Meireles Chrisostomo
>>
>> --
>> Esta mensagem foi verificada pelo sistema de antivírus e
>> acredita-se estar livre de perigo.
>
>
 --
 Esta mensagem foi verificada pelo sistema de antivírus e
 acredita-se estar livre de perigo.
>>>
>>>
>>>
>>> --
>>> Israel Meireles Chrisostomo
>>>
>>> --
>>> Esta mensagem foi verificada pelo sistema de antivírus e
>>> acredita-se estar livre de perigo.
>>
>>
>> --
>> Esta mensagem foi verificada pelo sistema de antivírus e
>> acredita-se estar livre de perigo.
>
>
> --
> Esta mensagem foi verificada pelo sistema de antivírus e
> acredita-se estar livre de perigo.

-- 
Esta mensagem foi verificada pelo sistema de antiv�rus e
 acredita-se estar livre de perigo.



[obm-l] Re: [obm-l] Re: [obm-l] Re: [obm-l] Re: [obm-l] Teoria dos números

2020-03-17 Por tôpico Carlos Gustavo Tamm de Araujo Moreira
Acho que a pergunta deve ser qual é o maior inteiro positivo que divide
essa expressão para todo valor de n ao mesmo tempo.

On Tue, Mar 17, 2020 at 6:58 AM Pedro José  wrote:

> Bom dia!
> Se você considerar a expressão n(427-90n-70n^2+45n^3+18n^4)
> D=|n(427-90n-70n^2+45n^3+18n^4)|
> Por exemplo, n=1
> D=330.
> Agora se liberar n para variar D tende a oo.
>
> Se n for raiz da expressão, também tende a oi, pois qualquer inteiro
> divide 0.
>
>
> Em seg, 16 de mar de 2020 22:16, Israel Meireles Chrisostomo <
> israelmchrisost...@gmail.com> escreveu:
>
>> não entendi
>>
>> Em seg., 16 de mar. de 2020 às 22:01, Pedro José 
>> escreveu:
>>
>>> Para um dado n é o módulo do valor da expressão.
>>>
>>> Em seg, 16 de mar de 2020 21:49, Pedro José 
>>> escreveu:
>>>
 Boa noite!
 O módulo dessa expressão tende a oo. Não existe máximo.
 Saudações,
 PJMS

 Em seg, 16 de mar de 2020 20:36, Israel Meireles Chrisostomo <
 israelmchrisost...@gmail.com> escreveu:

> Qual é o maior inteiro que divide  n (427 - 90n - 70n^2 + 45n^3 +
> 18n^4)?
>
> --
> Israel Meireles Chrisostomo
>
> --
> Esta mensagem foi verificada pelo sistema de antivírus e
> acredita-se estar livre de perigo.


>>> --
>>> Esta mensagem foi verificada pelo sistema de antivírus e
>>> acredita-se estar livre de perigo.
>>
>>
>>
>> --
>> Israel Meireles Chrisostomo
>>
>> --
>> Esta mensagem foi verificada pelo sistema de antivírus e
>> acredita-se estar livre de perigo.
>
>
> --
> Esta mensagem foi verificada pelo sistema de antivírus e
> acredita-se estar livre de perigo.

-- 
Esta mensagem foi verificada pelo sistema de antiv�rus e
 acredita-se estar livre de perigo.



[obm-l] Re: [obm-l] Re: [obm-l] Re: [obm-l] Teoria dos números

2020-03-17 Por tôpico Pedro José
Bom dia!
Se você considerar a expressão n(427-90n-70n^2+45n^3+18n^4)
D=|n(427-90n-70n^2+45n^3+18n^4)|
Por exemplo, n=1
D=330.
Agora se liberar n para variar D tende a oo.

Se n for raiz da expressão, também tende a oi, pois qualquer inteiro divide
0.


Em seg, 16 de mar de 2020 22:16, Israel Meireles Chrisostomo <
israelmchrisost...@gmail.com> escreveu:

> não entendi
>
> Em seg., 16 de mar. de 2020 às 22:01, Pedro José 
> escreveu:
>
>> Para um dado n é o módulo do valor da expressão.
>>
>> Em seg, 16 de mar de 2020 21:49, Pedro José 
>> escreveu:
>>
>>> Boa noite!
>>> O módulo dessa expressão tende a oo. Não existe máximo.
>>> Saudações,
>>> PJMS
>>>
>>> Em seg, 16 de mar de 2020 20:36, Israel Meireles Chrisostomo <
>>> israelmchrisost...@gmail.com> escreveu:
>>>
 Qual é o maior inteiro que divide  n (427 - 90n - 70n^2 + 45n^3 +
 18n^4)?

 --
 Israel Meireles Chrisostomo

 --
 Esta mensagem foi verificada pelo sistema de antivírus e
 acredita-se estar livre de perigo.
>>>
>>>
>> --
>> Esta mensagem foi verificada pelo sistema de antivírus e
>> acredita-se estar livre de perigo.
>
>
>
> --
> Israel Meireles Chrisostomo
>
> --
> Esta mensagem foi verificada pelo sistema de antivírus e
> acredita-se estar livre de perigo.

-- 
Esta mensagem foi verificada pelo sistema de antiv�rus e
 acredita-se estar livre de perigo.



[obm-l] Re: [obm-l] Re: [obm-l] Re: [obm-l] Re: [obm-l] Re: [obm-l] teoria dos números curiosidade

2019-07-04 Por tôpico Israel Meireles Chrisostomo
muito obrigado!!!

Em qui, 4 de jul de 2019 às 09:13, Claudio Buffara <
claudio.buff...@gmail.com> escreveu:

> Considere o seguinte algoritmo:
> Dada a/b (acho q precisa ser entre 0 e 1), tome o menor n1 tal que 1/n1 <=
> a/b.
> Daí, tome o menor n2 tal que 1/n2 <= a/b - 1/n1.
> Daí tome o menor n3 tal que 1/n3 <= a/b - 1/n1 - 1/n2
> Etc...
> Esse processo eventualmente para (quando uma desigualdade <= se torna uma
> igualdade), com:
> a/b = 1/n1 + 1/n2 + 1/n3 + ... + 1/nk, para algum k.
> Resta saber se produz uma fração egípcia (ou seja, se n1 < n2 < n3 < ...)
> e se a fração egípcia resultante é a menor possível.
> Vou pensar melhor é tentar simular alguns casos numa planilha.
>
>
> Enviado do meu iPhone
>
> Em 3 de jul de 2019, à(s) 22:11, Bernardo Freitas Paulo da Costa <
> bernardo...@gmail.com> escreveu:
>
> > On Wed, Jul 3, 2019 at 8:34 PM Claudio Buffara
> >  wrote:
> >> Infinitas.
> >> Basta usar recursivamente a relação  1/n = 1/(n+1) + 1/(n(n+1)), que
> cada vez você obtém uma representação mais longa.
> >> 1/2 = 1/3 + 1/6 = 1/3 + 1/7 + 1/42 = 1/3 + 1/7 + 1/43 + 1/1806 = ...
> >
> > Mais difícil, talvez, seria calcular qual o menor número de termos
> > necessários para representar p/q :)  Será que isso é NP completo?
> >
> > Abraços,
> > --
> > Bernardo Freitas Paulo da Costa
> >
> > --
> > Esta mensagem foi verificada pelo sistema de antivírus e
> > acredita-se estar livre de perigo.
> >
> >
> > =
> > Instruções para entrar na lista, sair da lista e usar a lista em
> > http://www.mat.puc-rio.br/~obmlistas/obm-l.html
> > =
>
> --
> Esta mensagem foi verificada pelo sistema de antivírus e
>  acredita-se estar livre de perigo.
>
>
> =
> Instru�ões para entrar na lista, sair da lista e usar a lista em
> http://www.mat.puc-rio.br/~obmlistas/obm-l.html
> =
>


-- 
Israel Meireles Chrisostomo

-- 
Esta mensagem foi verificada pelo sistema de antiv�rus e
 acredita-se estar livre de perigo.



Re: [obm-l] Re: [obm-l] Re: [obm-l] Re: [obm-l] Re: [obm-l] teoria dos números curiosidade

2019-07-04 Por tôpico Claudio Buffara
Considere o seguinte algoritmo:
Dada a/b (acho q precisa ser entre 0 e 1), tome o menor n1 tal que 1/n1 <= a/b.
Daí, tome o menor n2 tal que 1/n2 <= a/b - 1/n1.
Daí tome o menor n3 tal que 1/n3 <= a/b - 1/n1 - 1/n2
Etc...
Esse processo eventualmente para (quando uma desigualdade <= se torna uma 
igualdade), com:
a/b = 1/n1 + 1/n2 + 1/n3 + ... + 1/nk, para algum k.
Resta saber se produz uma fração egípcia (ou seja, se n1 < n2 < n3 < ...) e se 
a fração egípcia resultante é a menor possível.
Vou pensar melhor é tentar simular alguns casos numa planilha.


Enviado do meu iPhone

Em 3 de jul de 2019, à(s) 22:11, Bernardo Freitas Paulo da Costa 
 escreveu:

> On Wed, Jul 3, 2019 at 8:34 PM Claudio Buffara
>  wrote:
>> Infinitas.
>> Basta usar recursivamente a relação  1/n = 1/(n+1) + 1/(n(n+1)), que cada 
>> vez você obtém uma representação mais longa.
>> 1/2 = 1/3 + 1/6 = 1/3 + 1/7 + 1/42 = 1/3 + 1/7 + 1/43 + 1/1806 = ...
> 
> Mais difícil, talvez, seria calcular qual o menor número de termos
> necessários para representar p/q :)  Será que isso é NP completo?
> 
> Abraços,
> --
> Bernardo Freitas Paulo da Costa
> 
> -- 
> Esta mensagem foi verificada pelo sistema de antivírus e
> acredita-se estar livre de perigo.
> 
> 
> =
> Instruções para entrar na lista, sair da lista e usar a lista em
> http://www.mat.puc-rio.br/~obmlistas/obm-l.html
> =

-- 
Esta mensagem foi verificada pelo sistema de antiv�rus e
 acredita-se estar livre de perigo.


=
Instru��es para entrar na lista, sair da lista e usar a lista em
http://www.mat.puc-rio.br/~obmlistas/obm-l.html
=


[obm-l] Re: [obm-l] Re: [obm-l] Re: [obm-l] Re: [obm-l] teoria dos números curiosidade

2019-07-03 Por tôpico Bernardo Freitas Paulo da Costa
On Wed, Jul 3, 2019 at 8:34 PM Claudio Buffara
 wrote:
> Infinitas.
> Basta usar recursivamente a relação  1/n = 1/(n+1) + 1/(n(n+1)), que cada vez 
> você obtém uma representação mais longa.
> 1/2 = 1/3 + 1/6 = 1/3 + 1/7 + 1/42 = 1/3 + 1/7 + 1/43 + 1/1806 = ...

Mais difícil, talvez, seria calcular qual o menor número de termos
necessários para representar p/q :)  Será que isso é NP completo?

Abraços,
--
Bernardo Freitas Paulo da Costa

-- 
Esta mensagem foi verificada pelo sistema de antiv�rus e
 acredita-se estar livre de perigo.


=
Instru��es para entrar na lista, sair da lista e usar a lista em
http://www.mat.puc-rio.br/~obmlistas/obm-l.html
=


[obm-l] Re: [obm-l] Re: [obm-l] Re: [obm-l] teoria dos números curiosidade

2019-07-03 Por tôpico Claudio Buffara
Infinitas.
Basta usar recursivamente a relação  1/n = 1/(n+1) + 1/(n(n+1)), que cada
vez você obtém uma representação mais longa.
1/2 = 1/3 + 1/6 = 1/3 + 1/7 + 1/42 = 1/3 + 1/7 + 1/43 + 1/1806 = ...

On Wed, Jul 3, 2019 at 7:16 PM Israel Meireles Chrisostomo <
israelmchrisost...@gmail.com> wrote:

> Eu estive pensando para comigo mesmo, e então me perguntei qual é o número
> mínimo de representações distintas que se pode fazer com uma fração em suas
> representações unitárias.Alguém consegue chegar a alguma resposta?
>
>
> 
>  Livre
> de vírus. www.avast.com
> .
> <#m_-8728523408577579589_DAB4FAD8-2DD7-40BB-A1B8-4E2AA1F9FDF2>
>
> Em qua, 3 de jul de 2019 às 16:36, Israel Meireles Chrisostomo <
> israelmchrisost...@gmail.com> escreveu:
>
>> Muito obrigado pessoal!
>>
>>
>> 
>>  Livre
>> de vírus. www.avg.com
>> .
>>
>> <#m_-8728523408577579589_m_-7761868182500660031_DAB4FAD8-2DD7-40BB-A1B8-4E2AA1F9FDF2>
>>
>> Em qua, 3 de jul de 2019 às 16:06, Prof. Douglas Oliveira <
>> profdouglaso.del...@gmail.com> escreveu:
>>
>>> Lembro-me de uma resolucao feita por amigo aqui da lista, o Carlos
>>> Victor, na eureka número 2, no finalzinho, de uma olhada.
>>>
>>> Att
>>> Douglas Oliveira.
>>>
>>> Em qua, 3 de jul de 2019 15:08, Israel Meireles Chrisostomo <
>>> israelmchrisost...@gmail.com> escreveu:
>>>
 Esses dias eu estava estudando sobre frações unitárias, e assisti a um
 vídeo do pessoal impa sobre o assunto e fiquei sinceramente maravilhado com
 a engenhosidade dos egípcios.Mas uma questão não saiu da minha cabeça: um
 número inteiro pode ser separado em frações unitárias?Quais são as
 propriedades necessárias que uma fração deve ter para ser decomposta em
 frações egípcias
 --
 Israel Meireles Chrisostomo


 
  Livre
 de vírus. www.avg.com
 .

 <#m_-8728523408577579589_m_-7761868182500660031_m_-6615042783469650117_m_8002768564935167525_DAB4FAD8-2DD7-40BB-A1B8-4E2AA1F9FDF2>

 --
 Esta mensagem foi verificada pelo sistema de antivírus e
 acredita-se estar livre de perigo.
>>>
>>>
>>> --
>>> Esta mensagem foi verificada pelo sistema de antivírus e
>>> acredita-se estar livre de perigo.
>>
>>
>>
>> --
>> Israel Meireles Chrisostomo
>>
>
>
> --
> Israel Meireles Chrisostomo
>
> --
> Esta mensagem foi verificada pelo sistema de antivírus e
> acredita-se estar livre de perigo.

-- 
Esta mensagem foi verificada pelo sistema de antiv�rus e
 acredita-se estar livre de perigo.



[obm-l] Re: [obm-l] Re: [obm-l] Re: [obm-l] Re: [obm-l] Re: [obm-l] Re: [obm-l] Teoria dos números

2019-05-04 Por tôpico Pedro José
Bom dia!
Obrigado!
Encontrei uma demonstração, mas não tive bagavem para enrender. Vou ler as
publicações.

Saudações,
PJMS

Em sáb, 4 de mai de 2019 11:57, Anderson Torres <
torres.anderson...@gmail.com escreveu:

> Em seg, 29 de abr de 2019 às 16:38, Pedro José 
> escreveu:
> >
> > Boa tarde!
> > Pelo menos consegui descobrir que se um inteiro z >= não puder ser
> escrito da forma z=4^k (8m+7), com m,k >=0 e m,k inteiros então ele pode
> ser representado por uma soma de três parcelas, todas quadrados perfeitos.
> > Já a demonstração, não consegui compreender.
> >
>
> Essa é a parte chata. Mas tem paper pra caramba!
>
> https://en.wikipedia.org/wiki/Legendre%27s_three-square_theorem
> Legendre's three-square theorem - Wikipedia
> https://www.sciencedirect.com/science/article/pii/0022314X74900249
> A new proof of the three squares theorem - ScienceDirect
> https://brilliant.org/wiki/fermats-sum-of-two-squares-theorem/
> Sum of Squares Theorems | Brilliant Math & Science Wiki
>
> https://mathoverflow.net/questions/223939/proving-legendres-sum-of-3-squares-theorem-via-geometry-of-numbers
> nt.number theory - Proving Legendre's Sum of 3 Squares Theorem via
> Geometry of Numbers - MathOverflow
> https://core.ac.uk/download/pdf/82306476.pdf
> PII: 0022-314X(74)90024-9 - 82306476.pdf
>
> https://www.ams.org/journals/proc/1957-008-02/S0002-9939-1957-0085275-8/S0002-9939-1957-0085275-8.pdf
> S0002-9939-1957-0085275-8.pdf
> http://pollack.uga.edu/finding3squares-6.pdf
> finding3squares-6.pdf
> https://arxiv.org/pdf/0812.0540.pdf
> () - 0812.0540.pdf
>
>
>
> > Saudações,
> > PJMS
> >
> > Em seg, 29 de abr de 2019 às 14:14,  escreveu:
> >>
> >>
> >> Em 29 de abr de 2019 11:37, Pedro José  escreveu:
> >>
> >> Bom dia!
> >>
> >> Gostei desse problema. Fiz um montão de exemplos com números que não
> podem ser escritos como 4^n(8n+7) e todos puderam ser escritos como a soma
> de três quadrados.
> >> Vale para todos? Se sim, alguém poderia indicar uma demonstração?
> >>
> >> Saudações,
> >> PJMS
> >>
> >> Em dom, 7 de abr de 2019 às 16:16, Pedro José 
> escreveu:
> >>
> >> Boa tarde!
> >> Fiquei na dúvida se algoritmo valia para demonstração. Mas salvo engano
> para demonstração de quais números aceitam raízes primitivas usa-se
> algoritmo.
> >> Mas, agora com mais calma, poderia ter usado indução.
> >> 1) Foi provado que não vale para n=0.
> >> 2) Supondo que não vale para n, não valeria para n+1, por absurdo.
> Pois, se valesse, teria que valer para n.
> >> Creio que teria ficado mais elegante.
> >>
> >> Saudações,
> >> PJMS
> >>
> >>
> >> Em dom, 7 de abr de 2019 às 07:41, matematica10complicada <
> profdouglaso.del...@gmail.com> escreveu:
> >>
> >> Obrigado irmão. Está correto sim.
> >> Douglas O.
> >>
> >> Em qui, 4 de abr de 2019 às 19:44, Pedro José 
> escreveu:
> >>
> >> Boa noite!
> >> Estou mal, mesmo. Ao invés de nenhum li qualquer. Tinha simulado dois,
> três, quatro e deram fora, já iria questionar.
> >> Mas vamos lá:
> >> 0^2 = 0 mod8; 1^2 = 1 mod8; 2^2 = 4 mod8 3^2= 1 mod8; 4^2 = 0 mod 8;
> 5^2 = 1 mod 8 6^2 = 4 mod 8 e 7^2 = 1 mod8;
> >> Portanto o quadrado de um número, ou dá 0 ou da 1 ou 4 na equivalência
> mod8.
> >>
> >> Caso n=0 ==> x=8k+7= 7 mod8. Como mod conserva a soma, não há como
> somar 3 parcelas do conjunto, mesmo com repetição, {0,1,4} e obter 7. Então
> n>0
> >>
> >> Para n>0
> >> x = 4^n*(8K+7) ==> x pertence a 2 |N seja x = a^2 + b^2 + c^2 com a, b,
> c pertencentes a |N - {0}. teríamos que ter a,b,c pares ou um deles par e
> dois ímpares.
> >> mas 4 | x ==> x= 0 mod4. Mas se w pertence a 2|N + 1 ==> w^2 = 1 mod4.
> e se y pertence a 2 |N ==> y^2 = 0 mod 4. Como temos dois ímpares e um par
> e como a soma se conserva temos que x = 2 mod4, absurdo. Portanto só sobra
> a, b, c pares Se a,b,c pares podemos escrevê-los como a= 2s; b=2t e c=2u
> com s,t,u naturais.
> >> x = a^2+b^2+c^2= 4(s^2+t^2+u^2) ==> x1 = 4^(n-1) * (8m+7) = s^2+t^2+u^2
> e vale o mesmo raciocínio de que s,t,u são pares e poderão ser escritos
> como s=2f; t=2g; u= 2h, com f, g, h naturais e seguir nesse algoritmo até
> que tenhamos xj=4^0(8m+7)= p^2+q^2+r^2, absurdo. Pois, já vimos que n= 0
> não atende.
> >>
> >> Espero estar correto.
> >>
> >> Saudações.
> >>
> >>
> >>
> >>
> >>
> >> Em qua, 3 de abr de 2019 às 15:36, matematica10complicada <
> profdouglaso.del...@gmail.com> escreveu:
> >>
> >> Mostre que nenhum número da forma (4^n)(8k+7) , com n e k naturais pode
> ser escrito como soma de 3 tres quadrados
> >>
> >> Douglas Oliveira
> >>
> >> --
> >> Esta mensagem foi verificada pelo sistema de antivírus e
> >> acredita-se estar livre de perigo.
> >>
> >>
> >> --
> >> Esta mensagem foi verificada pelo sistema de antivírus e
> >> acredita-se estar livre de perigo.
> >>
> >>
> >> --
> >> Esta mensagem foi verificada pelo sistema de antivírus e
> >> acredita-se estar livre de perigo.
> >>
> >>
> >> --
> >> Esta mensagem foi verificada pelo sistema de antiv�rus e
> >> acredita-se estar livre de perigo.
> >>
> >>
> >>
> >> 

[obm-l] Re: [obm-l] Re: [obm-l] Re: [obm-l] Re: [obm-l] Re: [obm-l] Teoria dos números

2019-05-04 Por tôpico Anderson Torres
Em seg, 29 de abr de 2019 às 16:38, Pedro José  escreveu:
>
> Boa tarde!
> Pelo menos consegui descobrir que se um inteiro z >= não puder ser escrito da 
> forma z=4^k (8m+7), com m,k >=0 e m,k inteiros então ele pode ser 
> representado por uma soma de três parcelas, todas quadrados perfeitos.
> Já a demonstração, não consegui compreender.
>

Essa é a parte chata. Mas tem paper pra caramba!

https://en.wikipedia.org/wiki/Legendre%27s_three-square_theorem
Legendre's three-square theorem - Wikipedia
https://www.sciencedirect.com/science/article/pii/0022314X74900249
A new proof of the three squares theorem - ScienceDirect
https://brilliant.org/wiki/fermats-sum-of-two-squares-theorem/
Sum of Squares Theorems | Brilliant Math & Science Wiki
https://mathoverflow.net/questions/223939/proving-legendres-sum-of-3-squares-theorem-via-geometry-of-numbers
nt.number theory - Proving Legendre's Sum of 3 Squares Theorem via
Geometry of Numbers - MathOverflow
https://core.ac.uk/download/pdf/82306476.pdf
PII: 0022-314X(74)90024-9 - 82306476.pdf
https://www.ams.org/journals/proc/1957-008-02/S0002-9939-1957-0085275-8/S0002-9939-1957-0085275-8.pdf
S0002-9939-1957-0085275-8.pdf
http://pollack.uga.edu/finding3squares-6.pdf
finding3squares-6.pdf
https://arxiv.org/pdf/0812.0540.pdf
() - 0812.0540.pdf



> Saudações,
> PJMS
>
> Em seg, 29 de abr de 2019 às 14:14,  escreveu:
>>
>>
>> Em 29 de abr de 2019 11:37, Pedro José  escreveu:
>>
>> Bom dia!
>>
>> Gostei desse problema. Fiz um montão de exemplos com números que não podem 
>> ser escritos como 4^n(8n+7) e todos puderam ser escritos como a soma de três 
>> quadrados.
>> Vale para todos? Se sim, alguém poderia indicar uma demonstração?
>>
>> Saudações,
>> PJMS
>>
>> Em dom, 7 de abr de 2019 às 16:16, Pedro José  escreveu:
>>
>> Boa tarde!
>> Fiquei na dúvida se algoritmo valia para demonstração. Mas salvo engano para 
>> demonstração de quais números aceitam raízes primitivas usa-se algoritmo.
>> Mas, agora com mais calma, poderia ter usado indução.
>> 1) Foi provado que não vale para n=0.
>> 2) Supondo que não vale para n, não valeria para n+1, por absurdo. Pois, se 
>> valesse, teria que valer para n.
>> Creio que teria ficado mais elegante.
>>
>> Saudações,
>> PJMS
>>
>>
>> Em dom, 7 de abr de 2019 às 07:41, matematica10complicada 
>>  escreveu:
>>
>> Obrigado irmão. Está correto sim.
>> Douglas O.
>>
>> Em qui, 4 de abr de 2019 às 19:44, Pedro José  escreveu:
>>
>> Boa noite!
>> Estou mal, mesmo. Ao invés de nenhum li qualquer. Tinha simulado dois, três, 
>> quatro e deram fora, já iria questionar.
>> Mas vamos lá:
>> 0^2 = 0 mod8; 1^2 = 1 mod8; 2^2 = 4 mod8 3^2= 1 mod8; 4^2 = 0 mod 8; 5^2 = 1 
>> mod 8 6^2 = 4 mod 8 e 7^2 = 1 mod8;
>> Portanto o quadrado de um número, ou dá 0 ou da 1 ou 4 na equivalência  mod8.
>>
>> Caso n=0 ==> x=8k+7= 7 mod8. Como mod conserva a soma, não há como somar 3 
>> parcelas do conjunto, mesmo com repetição, {0,1,4} e obter 7. Então n>0
>>
>> Para n>0
>> x = 4^n*(8K+7) ==> x pertence a 2 |N seja x = a^2 + b^2 + c^2 com a, b, c 
>> pertencentes a |N - {0}. teríamos que ter a,b,c pares ou um deles par e dois 
>> ímpares.
>> mas 4 | x ==> x= 0 mod4. Mas se w pertence a 2|N + 1 ==> w^2 = 1 mod4. e se 
>> y pertence a 2 |N ==> y^2 = 0 mod 4. Como temos dois ímpares e um par e como 
>> a soma se conserva temos que x = 2 mod4, absurdo. Portanto só sobra a, b, c 
>> pares Se a,b,c pares podemos escrevê-los como a= 2s; b=2t e c=2u com s,t,u 
>> naturais.
>> x = a^2+b^2+c^2= 4(s^2+t^2+u^2) ==> x1 = 4^(n-1) * (8m+7) = s^2+t^2+u^2 e 
>> vale o mesmo raciocínio de que s,t,u são pares e poderão ser escritos como 
>> s=2f; t=2g; u= 2h, com f, g, h naturais e seguir nesse algoritmo até que 
>> tenhamos xj=4^0(8m+7)= p^2+q^2+r^2, absurdo. Pois, já vimos que n= 0 não 
>> atende.
>>
>> Espero estar correto.
>>
>> Saudações.
>>
>>
>>
>>
>>
>> Em qua, 3 de abr de 2019 às 15:36, matematica10complicada 
>>  escreveu:
>>
>> Mostre que nenhum número da forma (4^n)(8k+7) , com n e k naturais pode ser 
>> escrito como soma de 3 tres quadrados
>>
>> Douglas Oliveira
>>
>> --
>> Esta mensagem foi verificada pelo sistema de antivírus e
>> acredita-se estar livre de perigo.
>>
>>
>> --
>> Esta mensagem foi verificada pelo sistema de antivírus e
>> acredita-se estar livre de perigo.
>>
>>
>> --
>> Esta mensagem foi verificada pelo sistema de antivírus e
>> acredita-se estar livre de perigo.
>>
>>
>> --
>> Esta mensagem foi verificada pelo sistema de antiv�rus e
>> acredita-se estar livre de perigo.
>>
>>
>>
>> --
>> Esta mensagem foi verificada pelo sistema de antivírus e
>> acredita-se estar livre de perigo.
>
>
> --
> Esta mensagem foi verificada pelo sistema de antivírus e
> acredita-se estar livre de perigo.

-- 
Esta mensagem foi verificada pelo sistema de antiv�rus e
 acredita-se estar livre de perigo.


=
Instru��es para entrar na lista, sair da lista e usar a lista em

[obm-l] Re: [obm-l] Re: [obm-l] Re: [obm-l] Re: [obm-l] Teoria dos números

2019-04-29 Por tôpico Pedro José
Boa tarde!
Pelo menos consegui descobrir que se um inteiro z >= não puder ser escrito
da forma z=4^k (8m+7), com m,k >=0 e m,k inteiros então ele pode ser
representado por uma soma de três parcelas, todas quadrados perfeitos.
Já a demonstração, não consegui compreender.

Saudações,
PJMS

Em seg, 29 de abr de 2019 às 14:14,  escreveu:

>
> Em 29 de abr de 2019 11:37, Pedro José  escreveu:
>
> Bom dia!
>
> Gostei desse problema. Fiz um montão de exemplos com números que não podem
> ser escritos como 4^n(8n+7) e todos puderam ser escritos como a soma de
> três quadrados.
> Vale para todos? Se sim, alguém poderia indicar uma demonstração?
>
> Saudações,
> PJMS
>
> Em dom, 7 de abr de 2019 às 16:16, Pedro José 
> escreveu:
>
> Boa tarde!
> Fiquei na dúvida se algoritmo valia para demonstração. Mas salvo engano
> para demonstração de quais números aceitam raízes primitivas usa-se
> algoritmo.
> Mas, agora com mais calma, poderia ter usado indução.
> 1) Foi provado que não vale para n=0.
> 2) Supondo que não vale para n, não valeria para n+1, por absurdo. Pois,
> se valesse, teria que valer para n.
> Creio que teria ficado mais elegante.
>
> Saudações,
> PJMS
>
>
> Em dom, 7 de abr de 2019 às 07:41, matematica10complicada <
> profdouglaso.del...@gmail.com> escreveu:
>
> Obrigado irmão. Está correto sim.
> Douglas O.
>
> Em qui, 4 de abr de 2019 às 19:44, Pedro José 
> escreveu:
>
> Boa noite!
> Estou mal, mesmo. Ao invés de nenhum li qualquer. Tinha simulado dois,
> três, quatro e deram fora, já iria questionar.
> Mas vamos lá:
> 0^2 = 0 mod8; 1^2 = 1 mod8; 2^2 = 4 mod8 3^2= 1 mod8; 4^2 = 0 mod 8; 5^2 =
> 1 mod 8 6^2 = 4 mod 8 e 7^2 = 1 mod8;
> Portanto o quadrado de um número, ou dá 0 ou da 1 ou 4 na equivalência
> mod8.
>
> Caso n=0 ==> x=8k+7= 7 mod8. Como mod conserva a soma, não há como somar 3
> parcelas do conjunto, mesmo com repetição, {0,1,4} e obter 7. Então n>0
>
> Para n>0
> x = 4^n*(8K+7) ==> x pertence a 2 |N seja x = a^2 + b^2 + c^2 com a, b, c
> pertencentes a |N - {0}. teríamos que ter a,b,c pares ou um deles par e
> dois ímpares.
> mas 4 | x ==> x= 0 mod4. Mas se w pertence a 2|N + 1 ==> w^2 = 1 mod4. e
> se y pertence a 2 |N ==> y^2 = 0 mod 4. Como temos dois ímpares e um par e
> como a soma se conserva temos que x = 2 mod4, absurdo. Portanto só sobra a,
> b, c pares Se a,b,c pares podemos escrevê-los como a= 2s; b=2t e c=2u com
> s,t,u naturais.
> x = a^2+b^2+c^2= 4(s^2+t^2+u^2) ==> x1 = 4^(n-1) * (8m+7) = s^2+t^2+u^2 e
> vale o mesmo raciocínio de que s,t,u são pares e poderão ser escritos como
> s=2f; t=2g; u= 2h, com f, g, h naturais e seguir nesse algoritmo até que
> tenhamos xj=4^0(8m+7)= p^2+q^2+r^2, absurdo. Pois, já vimos que n= 0 não
> atende.
>
> Espero estar correto.
>
> Saudações.
>
>
>
>
>
> Em qua, 3 de abr de 2019 às 15:36, matematica10complicada <
> profdouglaso.del...@gmail.com> escreveu:
>
> Mostre que nenhum número da forma (4^n)(8k+7) , com n e k naturais pode
> ser escrito como soma de 3 tres quadrados
>
> Douglas Oliveira
>
> --
> Esta mensagem foi verificada pelo sistema de antivírus e
> acredita-se estar livre de perigo.
>
>
> --
> Esta mensagem foi verificada pelo sistema de antivírus e
> acredita-se estar livre de perigo.
>
>
> --
> Esta mensagem foi verificada pelo sistema de antivírus e
> acredita-se estar livre de perigo.
>
>
> --
> Esta mensagem foi verificada pelo sistema de antiv�rus e
> acredita-se estar livre de perigo.
>
>
>
> --
> Esta mensagem foi verificada pelo sistema de antivírus e
> acredita-se estar livre de perigo.

-- 
Esta mensagem foi verificada pelo sistema de antiv�rus e
 acredita-se estar livre de perigo.



Re: [obm-l] Re: [obm-l] Re: [obm-l] Re: [obm-l] Teoria dos números

2019-04-29 Por tôpico prmanso
Em 29 de abr de 2019 11:37, Pedro José  escreveu:Bom dia!Gostei desse problema. Fiz um montão de exemplos com números que não podem ser escritos como 4^n(8n+7) e todos puderam ser escritos como a soma de três quadrados.Vale para todos? Se sim, alguém poderia indicar uma demonstração?Saudações,PJMSEm dom, 7 de abr de 2019 às 16:16, Pedro José  escreveu:Boa tarde!Fiquei na dúvida se algoritmo valia para demonstração. Mas salvo engano para demonstração de quais números aceitam raízes primitivas usa-se algoritmo.Mas, agora com mais calma, poderia ter usado indução.1) Foi provado que não vale para n=0.2) Supondo que não vale para n, não valeria para n+1, por absurdo. Pois, se valesse, teria que valer para n.Creio que teria ficado mais elegante.Saudações,PJMSEm dom, 7 de abr de 2019 às 07:41, matematica10complicada  escreveu:Obrigado irmão. Está correto sim. Douglas O.Em qui, 4 de abr de 2019 às 19:44, Pedro José  escreveu:Boa noite!Estou mal, mesmo. Ao invés de nenhum li qualquer. Tinha simulado dois, três, quatro e deram fora, já iria questionar.Mas vamos lá:0^2 = 0 mod8; 1^2 = 1 mod8; 2^2 = 4 mod8 3^2= 1 mod8; 4^2 = 0 mod 8; 5^2 = 1 mod 8 6^2 = 4 mod 8 e 7^2 = 1 mod8;Portanto o quadrado de um número, ou dá 0 ou da 1 ou 4 na equivalência  mod8.Caso n=0 ==> x=8k+7= 7 mod8. Como mod conserva a soma, não há como somar 3 parcelas do conjunto, mesmo com repetição, {0,1,4} e obter 7. Então n>0Para n>0x = 4^n*(8K+7) ==> x pertence a 2 |N seja x = a^2 + b^2 + c^2 com a, b, c pertencentes a |N - {0}. teríamos que ter a,b,c pares ou um deles par e dois ímpares.mas 4 | x ==> x= 0 mod4. Mas se w pertence a 2|N + 1 ==> w^2 = 1 mod4. e se y pertence a 2 |N ==> y^2 = 0 mod 4. Como temos dois ímpares e um par e como a soma se conserva temos que x = 2 mod4, absurdo. Portanto só sobra a, b, c pares Se a,b,c pares podemos escrevê-los como a= 2s; b=2t e c=2u com s,t,u naturais.x = a^2+b^2+c^2= 4(s^2+t^2+u^2) ==> x1 = 4^(n-1) * (8m+7) = s^2+t^2+u^2 e vale o mesmo raciocínio de que s,t,u são pares e poderão ser escritos como s=2f; t=2g; u= 2h, com f, g, h naturais e seguir nesse algoritmo até que tenhamos xj=4^0(8m+7)= p^2+q^2+r^2, absurdo. Pois, já vimos que n= 0 não atende.Espero estar correto.Saudações. Em qua, 3 de abr de 2019 às 15:36, matematica10complicada  escreveu:Mostre que nenhum número da forma (4^n)(8k+7) , com n e k naturais pode ser escrito como soma de 3 tres quadradosDouglas Oliveira
--
Esta mensagem foi verificada pelo sistema de antivírus e 
 acredita-se estar livre de perigo.


--
Esta mensagem foi verificada pelo sistema de antivírus e 
 acredita-se estar livre de perigo.


--
Esta mensagem foi verificada pelo sistema de antivírus e 
 acredita-se estar livre de perigo.



--
Esta mensagem foi verificada pelo sistema de antiv�rus e 
 acredita-se estar livre de perigo.

--
Esta mensagem foi verificada pelo sistema de antivírus e 
 acredita-se estar livre de perigo.




[obm-l] Re: [obm-l] Re: [obm-l] Re: [obm-l] Teoria dos números

2019-04-29 Por tôpico Pedro José
Bom dia!

Gostei desse problema. Fiz um montão de exemplos com números que não podem
ser escritos como 4^n(8n+7) e todos puderam ser escritos como a soma de
três quadrados.
Vale para todos? Se sim, alguém poderia indicar uma demonstração?

Saudações,
PJMS

Em dom, 7 de abr de 2019 às 16:16, Pedro José 
escreveu:

> Boa tarde!
> Fiquei na dúvida se algoritmo valia para demonstração. Mas salvo engano
> para demonstração de quais números aceitam raízes primitivas usa-se
> algoritmo.
> Mas, agora com mais calma, poderia ter usado indução.
> 1) Foi provado que não vale para n=0.
> 2) Supondo que não vale para n, não valeria para n+1, por absurdo. Pois,
> se valesse, teria que valer para n.
> Creio que teria ficado mais elegante.
>
> Saudações,
> PJMS
>
>
> Em dom, 7 de abr de 2019 às 07:41, matematica10complicada <
> profdouglaso.del...@gmail.com> escreveu:
>
>> Obrigado irmão. Está correto sim.
>> Douglas O.
>>
>> Em qui, 4 de abr de 2019 às 19:44, Pedro José 
>> escreveu:
>>
>>> Boa noite!
>>> Estou mal, mesmo. Ao invés de nenhum li qualquer. Tinha simulado dois,
>>> três, quatro e deram fora, já iria questionar.
>>> Mas vamos lá:
>>> 0^2 = 0 mod8; 1^2 = 1 mod8; 2^2 = 4 mod8 3^2= 1 mod8; 4^2 = 0 mod 8; 5^2
>>> = 1 mod 8 6^2 = 4 mod 8 e 7^2 = 1 mod8;
>>> Portanto o quadrado de um número, ou dá 0 ou da 1 ou 4 na equivalência
>>> mod8.
>>>
>>> Caso n=0 ==> x=8k+7= 7 mod8. Como mod conserva a soma, não há como somar
>>> 3 parcelas do conjunto, mesmo com repetição, {0,1,4} e obter 7. Então n>0
>>>
>>> Para n>0
>>> x = 4^n*(8K+7) ==> x pertence a 2 |N seja x = a^2 + b^2 + c^2 com a, b,
>>> c pertencentes a |N - {0}. teríamos que ter a,b,c pares ou um deles par e
>>> dois ímpares.
>>> mas 4 | x ==> x= 0 mod4. Mas se w pertence a 2|N + 1 ==> w^2 = 1 mod4. e
>>> se y pertence a 2 |N ==> y^2 = 0 mod 4. Como temos dois ímpares e um par e
>>> como a soma se conserva temos que x = 2 mod4, absurdo. Portanto só sobra a,
>>> b, c pares Se a,b,c pares podemos escrevê-los como a= 2s; b=2t e c=2u com
>>> s,t,u naturais.
>>> x = a^2+b^2+c^2= 4(s^2+t^2+u^2) ==> x1 = 4^(n-1) * (8m+7) = s^2+t^2+u^2
>>> e vale o mesmo raciocínio de que s,t,u são pares e poderão ser escritos
>>> como s=2f; t=2g; u= 2h, com f, g, h naturais e seguir nesse algoritmo até
>>> que tenhamos xj=4^0(8m+7)= p^2+q^2+r^2, absurdo. Pois, já vimos que n= 0
>>> não atende.
>>>
>>> Espero estar correto.
>>>
>>> Saudações.
>>>
>>>
>>>
>>>
>>>
>>> Em qua, 3 de abr de 2019 às 15:36, matematica10complicada <
>>> profdouglaso.del...@gmail.com> escreveu:
>>>
 Mostre que nenhum número da forma (4^n)(8k+7) , com n e k naturais pode
 ser escrito como soma de 3 tres quadrados

 Douglas Oliveira

 --
 Esta mensagem foi verificada pelo sistema de antivírus e
 acredita-se estar livre de perigo.
>>>
>>>
>>> --
>>> Esta mensagem foi verificada pelo sistema de antivírus e
>>> acredita-se estar livre de perigo.
>>
>>
>> --
>> Esta mensagem foi verificada pelo sistema de antivírus e
>> acredita-se estar livre de perigo.
>
>

-- 
Esta mensagem foi verificada pelo sistema de antiv�rus e
 acredita-se estar livre de perigo.



[obm-l] Re: [obm-l] Re: [obm-l] Re: [obm-l] Teoria dos números

2019-04-07 Por tôpico Pedro José
Boa tarde!
Fiquei na dúvida se algoritmo valia para demonstração. Mas salvo engano
para demonstração de quais números aceitam raízes primitivas usa-se
algoritmo.
Mas, agora com mais calma, poderia ter usado indução.
1) Foi provado que não vale para n=0.
2) Supondo que não vale para n, não valeria para n+1, por absurdo. Pois, se
valesse, teria que valer para n.
Creio que teria ficado mais elegante.

Saudações,
PJMS


Em dom, 7 de abr de 2019 às 07:41, matematica10complicada <
profdouglaso.del...@gmail.com> escreveu:

> Obrigado irmão. Está correto sim.
> Douglas O.
>
> Em qui, 4 de abr de 2019 às 19:44, Pedro José 
> escreveu:
>
>> Boa noite!
>> Estou mal, mesmo. Ao invés de nenhum li qualquer. Tinha simulado dois,
>> três, quatro e deram fora, já iria questionar.
>> Mas vamos lá:
>> 0^2 = 0 mod8; 1^2 = 1 mod8; 2^2 = 4 mod8 3^2= 1 mod8; 4^2 = 0 mod 8; 5^2
>> = 1 mod 8 6^2 = 4 mod 8 e 7^2 = 1 mod8;
>> Portanto o quadrado de um número, ou dá 0 ou da 1 ou 4 na equivalência
>> mod8.
>>
>> Caso n=0 ==> x=8k+7= 7 mod8. Como mod conserva a soma, não há como somar
>> 3 parcelas do conjunto, mesmo com repetição, {0,1,4} e obter 7. Então n>0
>>
>> Para n>0
>> x = 4^n*(8K+7) ==> x pertence a 2 |N seja x = a^2 + b^2 + c^2 com a, b, c
>> pertencentes a |N - {0}. teríamos que ter a,b,c pares ou um deles par e
>> dois ímpares.
>> mas 4 | x ==> x= 0 mod4. Mas se w pertence a 2|N + 1 ==> w^2 = 1 mod4. e
>> se y pertence a 2 |N ==> y^2 = 0 mod 4. Como temos dois ímpares e um par e
>> como a soma se conserva temos que x = 2 mod4, absurdo. Portanto só sobra a,
>> b, c pares Se a,b,c pares podemos escrevê-los como a= 2s; b=2t e c=2u com
>> s,t,u naturais.
>> x = a^2+b^2+c^2= 4(s^2+t^2+u^2) ==> x1 = 4^(n-1) * (8m+7) = s^2+t^2+u^2 e
>> vale o mesmo raciocínio de que s,t,u são pares e poderão ser escritos como
>> s=2f; t=2g; u= 2h, com f, g, h naturais e seguir nesse algoritmo até que
>> tenhamos xj=4^0(8m+7)= p^2+q^2+r^2, absurdo. Pois, já vimos que n= 0 não
>> atende.
>>
>> Espero estar correto.
>>
>> Saudações.
>>
>>
>>
>>
>>
>> Em qua, 3 de abr de 2019 às 15:36, matematica10complicada <
>> profdouglaso.del...@gmail.com> escreveu:
>>
>>> Mostre que nenhum número da forma (4^n)(8k+7) , com n e k naturais pode
>>> ser escrito como soma de 3 tres quadrados
>>>
>>> Douglas Oliveira
>>>
>>> --
>>> Esta mensagem foi verificada pelo sistema de antivírus e
>>> acredita-se estar livre de perigo.
>>
>>
>> --
>> Esta mensagem foi verificada pelo sistema de antivírus e
>> acredita-se estar livre de perigo.
>
>
> --
> Esta mensagem foi verificada pelo sistema de antivírus e
> acredita-se estar livre de perigo.

-- 
Esta mensagem foi verificada pelo sistema de antiv�rus e
 acredita-se estar livre de perigo.



[obm-l] Re: [obm-l] Re: [obm-l] Re: [obm-l] Re: [obm-l] Teoria dos números

2018-11-28 Por tôpico Pedro José
Boa tarde!
Perdoem-me pela insistência.
Mas outra forma de pensar.
Se k>0, e se a>b e se pensarmos em duas soluções positivas logicamente
estamos assumindo que a seja máximo. Pois, se existe a1 solução e a1>=a
então a1.a=b^2-k>b^2, absurdo.
Portanto quando dizemos que a>b, estamos escolhendo conscientemente a
solução máxima. Então não é ocaso de pegar uma solução hipotética, supor
que é solução mínima  e provar que existe uma menor, gerando absurdo.
Saudações,
PJMS

Em Qua, 28 de nov de 2018 15:42, Pedro José  escreveu:

> Boa tarde!
> Preciso de ajuda.
> Após pensar mais um pouco é bem razoável que dada uma solução hipotética e
> se consiga provar que há uma menor, que seja um absurdo. Absurdo no
> sentido, que não há solução. Gostaria até que me sugerissem material
> didático sobre o tópico.
> Não obstante existe solução para a1 e
> x inteiro.
> Então há um paradoxo. Que por um lado se a é solução para a 0 Mas quando se achou a foi feita uma restrição, SPG, que a >=b e após
> estudar o caso a=b, ficamos com a restrição a>b, que é usada para provar
> que a1b. Só que:
> a1=(b^2-k)/ab.
> Então esse é o ponto a1 mesmo sendo maior que zero,  não é solução pois
> a1 solução. A prova para a  Por favor, alguém poderia opinar.
> Saudações,
> PJMS
>
> Em Seg, 26 de nov de 2018 01:59, Pedro José 
> escreveu:
>
>> Bom dia!
>> Refiro-me a solução recomendada por Israel.
>> A princípio eu encrenquei com a solução. Pois, sem nenhuma caracterização
>> definiu que a era mínimo. Então achei que a solução para a > capenga. Mas como não houve nenhuma crítica, julguei ser cisma minha. Mas
>> depois me veio o pensamento, usando a técnica usada na resolução sempre que
>> houver duas soluções(digo duas mesmo, distintas) haveria um absurdo. Pois
>> ele supôs que a era mínimo e provou que a1, solução, a1> como ele não usou nenhum argumento para supor que a era mínimo, apenas
>> arbitrou, poderia ter arbitrado que a era máximo e se a1>a, também seria
>> absurdo.
>> Aí, encrenquei mesmo com a soluçao e achei essa família de soluções para
>> a> K=x^2; b=x^3 e a =x^5-x, que para x>1, xinteiro, atende a> k=(a^2+b^2)/(ab+1); continua dando um quadrado perfeito, mas se não fosse?
>> A linha de argumento da solução,  desprezou essa possibilidade.
>> Preciso ajuda, estou correto ou errado?
>> Grato,
>> PJMS
>>
>> Em Seg, 27 de ago de 2018 11:01, Pedro José 
>> escreveu:
>>
>>> Bom dia!
>>>
>>> Linda solução pela simplicidade de ferramentas utilizadas.
>>> Todavia, creio eu que não foi de bom alvitre utilizar a imagem de um
>>> matemático famoso e divulgar que ele só ganhou um ponto na questão.
>>> A mensagem, não explícita, mas é uma mensagem:"Ele não resolveu mas eu
>>> sim."
>>> As condições de resolução são totalmente diversas. Inclusive, devido às
>>> questões anteriores ele já pode ter chegado a essa com o ponterinho do
>>> relógio pendurado.
>>> A solução do problema, mesmo tardia, sem a carga emocional que uma IMO
>>> deve impor aos seus participantes, ainda é carregada de méritos, e na minha
>>> visão, essa em particular, com uma beleza maior por ser sutil e singela.
>>> Mas usar a imagem da "fera", não obstante não ser dono da verdade, foi
>>> bola fora.
>>>
>>> Saudações,
>>> PJMS
>>>
>>>
>>> Em qui, 23 de ago de 2018 às 15:50, Israel Meireles Chrisostomo <
>>> israelmchrisost...@gmail.com> escreveu:
>>>
 Assista a esse vídeo:
 https://www.youtube.com/watch?v=Cy3Vyl-jxpk

 Em qui, 23 de ago de 2018 às 14:09, Daniel Quevedo 
 escreveu:

> Blz não sabia q era de lá, vou consultar. Obrigado!
>
> Em qui, 23 de ago de 2018 às 10:30, Claudio Buffara <
> claudio.buff...@gmail.com> escreveu:
>
>> Esse é clássico. Foi o problema 6 da IMO de 1988 e é usualmente
>> considerado o problema mais difícil proposto numa IMO, pelo menos até
>> aquela data.
>>
>> Um bom ponto de partida pode ser este:
>> https://en.wikipedia.org/wiki/Vieta_jumping
>> Ou então: https://mks.mff.cuni.cz/kalva/imo.html
>>
>> []s,
>> Claudio.
>>
>>
>>
>> 2018-08-23 9:57 GMT-03:00 Daniel Quevedo :
>>
>>> Sejam a e b inteiros estritamente positivos tais que (ab + 1) é um
>>> divisor de (a^2 + b^2). Sobre o número  (a^2 +b^2)/(ab +1) podemos 
>>> afirmar
>>> que é um quadrado perfeito:
>>> A) se, e só se, a e b também o forem.
>>> B) se, e só se, a e b tiverem acreana paridade
>>> C) se, e só se, a e b tiverem paridades distintas
>>> D) somente para um número finito de valores de a e b
>>> E) sempre
>>>
>>> R: e
>>>
>> --
>>> Fiscal: Daniel Quevedo
>>>
>>
>>> --
>>> Esta mensagem foi verificada pelo sistema de antivírus e
>>> acredita-se estar livre de perigo.
>>
>>
>>
>> --
>> Esta mensagem foi verificada pelo sistema de antivírus e
>> acredita-se estar livre de perigo.
>
> --
> Fiscal: Daniel Quevedo
>
> --
> Esta mensagem foi verificada pelo 

[obm-l] Re: [obm-l] Re: [obm-l] Re: [obm-l] Re: [obm-l] Teoria dos números

2018-11-28 Por tôpico Pedro José
Boa tarde!
Preciso de ajuda.
Após pensar mais um pouco é bem razoável que dada uma solução hipotética e
se consiga provar que há uma menor, que seja um absurdo. Absurdo no
sentido, que não há solução. Gostaria até que me sugerissem material
didático sobre o tópico.
Não obstante existe solução para a1 e
x inteiro.
Então há um paradoxo. Que por um lado se a é solução para a=b e após
estudar o caso a=b, ficamos com a restrição a>b, que é usada para provar
que a1b. Só que:
a1=(b^2-k)/ab.
Então esse é o ponto a1 mesmo sendo maior que zero,  não é solução pois
a1 escreveu:

> Bom dia!
> Refiro-me a solução recomendada por Israel.
> A princípio eu encrenquei com a solução. Pois, sem nenhuma caracterização
> definiu que a era mínimo. Então achei que a solução para a  capenga. Mas como não houve nenhuma crítica, julguei ser cisma minha. Mas
> depois me veio o pensamento, usando a técnica usada na resolução sempre que
> houver duas soluções(digo duas mesmo, distintas) haveria um absurdo. Pois
> ele supôs que a era mínimo e provou que a1, solução, a1 como ele não usou nenhum argumento para supor que a era mínimo, apenas
> arbitrou, poderia ter arbitrado que a era máximo e se a1>a, também seria
> absurdo.
> Aí, encrenquei mesmo com a soluçao e achei essa família de soluções para
> a K=x^2; b=x^3 e a =x^5-x, que para x>1, xinteiro, atende a k=(a^2+b^2)/(ab+1); continua dando um quadrado perfeito, mas se não fosse?
> A linha de argumento da solução,  desprezou essa possibilidade.
> Preciso ajuda, estou correto ou errado?
> Grato,
> PJMS
>
> Em Seg, 27 de ago de 2018 11:01, Pedro José 
> escreveu:
>
>> Bom dia!
>>
>> Linda solução pela simplicidade de ferramentas utilizadas.
>> Todavia, creio eu que não foi de bom alvitre utilizar a imagem de um
>> matemático famoso e divulgar que ele só ganhou um ponto na questão.
>> A mensagem, não explícita, mas é uma mensagem:"Ele não resolveu mas eu
>> sim."
>> As condições de resolução são totalmente diversas. Inclusive, devido às
>> questões anteriores ele já pode ter chegado a essa com o ponterinho do
>> relógio pendurado.
>> A solução do problema, mesmo tardia, sem a carga emocional que uma IMO
>> deve impor aos seus participantes, ainda é carregada de méritos, e na minha
>> visão, essa em particular, com uma beleza maior por ser sutil e singela.
>> Mas usar a imagem da "fera", não obstante não ser dono da verdade, foi
>> bola fora.
>>
>> Saudações,
>> PJMS
>>
>>
>> Em qui, 23 de ago de 2018 às 15:50, Israel Meireles Chrisostomo <
>> israelmchrisost...@gmail.com> escreveu:
>>
>>> Assista a esse vídeo:
>>> https://www.youtube.com/watch?v=Cy3Vyl-jxpk
>>>
>>> Em qui, 23 de ago de 2018 às 14:09, Daniel Quevedo 
>>> escreveu:
>>>
 Blz não sabia q era de lá, vou consultar. Obrigado!

 Em qui, 23 de ago de 2018 às 10:30, Claudio Buffara <
 claudio.buff...@gmail.com> escreveu:

> Esse é clássico. Foi o problema 6 da IMO de 1988 e é usualmente
> considerado o problema mais difícil proposto numa IMO, pelo menos até
> aquela data.
>
> Um bom ponto de partida pode ser este:
> https://en.wikipedia.org/wiki/Vieta_jumping
> Ou então: https://mks.mff.cuni.cz/kalva/imo.html
>
> []s,
> Claudio.
>
>
>
> 2018-08-23 9:57 GMT-03:00 Daniel Quevedo :
>
>> Sejam a e b inteiros estritamente positivos tais que (ab + 1) é um
>> divisor de (a^2 + b^2). Sobre o número  (a^2 +b^2)/(ab +1) podemos 
>> afirmar
>> que é um quadrado perfeito:
>> A) se, e só se, a e b também o forem.
>> B) se, e só se, a e b tiverem acreana paridade
>> C) se, e só se, a e b tiverem paridades distintas
>> D) somente para um número finito de valores de a e b
>> E) sempre
>>
>> R: e
>>
> --
>> Fiscal: Daniel Quevedo
>>
>
>> --
>> Esta mensagem foi verificada pelo sistema de antivírus e
>> acredita-se estar livre de perigo.
>
>
>
> --
> Esta mensagem foi verificada pelo sistema de antivírus e
> acredita-se estar livre de perigo.

 --
 Fiscal: Daniel Quevedo

 --
 Esta mensagem foi verificada pelo sistema de antivírus e
 acredita-se estar livre de perigo.
>>>
>>>
>>>
>>> --
>>> Israel Meireles Chrisostomo
>>>
>>> --
>>> Esta mensagem foi verificada pelo sistema de antivírus e
>>> acredita-se estar livre de perigo.
>>
>>

-- 
Esta mensagem foi verificada pelo sistema de antiv�rus e
 acredita-se estar livre de perigo.



[obm-l] Re: [obm-l] Re: [obm-l] Re: [obm-l] Re: [obm-l] Teoria dos números

2018-11-25 Por tôpico Pedro José
Bom dia!
Refiro-me a solução recomendada por Israel.
A princípio eu encrenquei com a solução. Pois, sem nenhuma caracterização
definiu que a era mínimo. Então achei que a solução para a a, também seria
absurdo.
Aí, encrenquei mesmo com a soluçao e achei essa família de soluções para
a1, xinteiro, atende a escreveu:

> Bom dia!
>
> Linda solução pela simplicidade de ferramentas utilizadas.
> Todavia, creio eu que não foi de bom alvitre utilizar a imagem de um
> matemático famoso e divulgar que ele só ganhou um ponto na questão.
> A mensagem, não explícita, mas é uma mensagem:"Ele não resolveu mas eu
> sim."
> As condições de resolução são totalmente diversas. Inclusive, devido às
> questões anteriores ele já pode ter chegado a essa com o ponterinho do
> relógio pendurado.
> A solução do problema, mesmo tardia, sem a carga emocional que uma IMO
> deve impor aos seus participantes, ainda é carregada de méritos, e na minha
> visão, essa em particular, com uma beleza maior por ser sutil e singela.
> Mas usar a imagem da "fera", não obstante não ser dono da verdade, foi
> bola fora.
>
> Saudações,
> PJMS
>
>
> Em qui, 23 de ago de 2018 às 15:50, Israel Meireles Chrisostomo <
> israelmchrisost...@gmail.com> escreveu:
>
>> Assista a esse vídeo:
>> https://www.youtube.com/watch?v=Cy3Vyl-jxpk
>>
>> Em qui, 23 de ago de 2018 às 14:09, Daniel Quevedo 
>> escreveu:
>>
>>> Blz não sabia q era de lá, vou consultar. Obrigado!
>>>
>>> Em qui, 23 de ago de 2018 às 10:30, Claudio Buffara <
>>> claudio.buff...@gmail.com> escreveu:
>>>
 Esse é clássico. Foi o problema 6 da IMO de 1988 e é usualmente
 considerado o problema mais difícil proposto numa IMO, pelo menos até
 aquela data.

 Um bom ponto de partida pode ser este:
 https://en.wikipedia.org/wiki/Vieta_jumping
 Ou então: https://mks.mff.cuni.cz/kalva/imo.html

 []s,
 Claudio.



 2018-08-23 9:57 GMT-03:00 Daniel Quevedo :

> Sejam a e b inteiros estritamente positivos tais que (ab + 1) é um
> divisor de (a^2 + b^2). Sobre o número  (a^2 +b^2)/(ab +1) podemos afirmar
> que é um quadrado perfeito:
> A) se, e só se, a e b também o forem.
> B) se, e só se, a e b tiverem acreana paridade
> C) se, e só se, a e b tiverem paridades distintas
> D) somente para um número finito de valores de a e b
> E) sempre
>
> R: e
>
 --
> Fiscal: Daniel Quevedo
>

> --
> Esta mensagem foi verificada pelo sistema de antivírus e
> acredita-se estar livre de perigo.



 --
 Esta mensagem foi verificada pelo sistema de antivírus e
 acredita-se estar livre de perigo.
>>>
>>> --
>>> Fiscal: Daniel Quevedo
>>>
>>> --
>>> Esta mensagem foi verificada pelo sistema de antivírus e
>>> acredita-se estar livre de perigo.
>>
>>
>>
>> --
>> Israel Meireles Chrisostomo
>>
>> --
>> Esta mensagem foi verificada pelo sistema de antivírus e
>> acredita-se estar livre de perigo.
>
>

-- 
Esta mensagem foi verificada pelo sistema de antiv�rus e
 acredita-se estar livre de perigo.



[obm-l] Re: [obm-l] Re: [obm-l] Re: [obm-l] Re: [obm-l] Teoria dos números

2018-08-27 Por tôpico Pedro José
Bom dia!

Linda solução pela simplicidade de ferramentas utilizadas.
Todavia, creio eu que não foi de bom alvitre utilizar a imagem de um
matemático famoso e divulgar que ele só ganhou um ponto na questão.
A mensagem, não explícita, mas é uma mensagem:"Ele não resolveu mas eu sim."
As condições de resolução são totalmente diversas. Inclusive, devido às
questões anteriores ele já pode ter chegado a essa com o ponterinho do
relógio pendurado.
A solução do problema, mesmo tardia, sem a carga emocional que uma IMO deve
impor aos seus participantes, ainda é carregada de méritos, e na minha
visão, essa em particular, com uma beleza maior por ser sutil e singela.
Mas usar a imagem da "fera", não obstante não ser dono da verdade, foi bola
fora.

Saudações,
PJMS


Em qui, 23 de ago de 2018 às 15:50, Israel Meireles Chrisostomo <
israelmchrisost...@gmail.com> escreveu:

> Assista a esse vídeo:
> https://www.youtube.com/watch?v=Cy3Vyl-jxpk
>
> Em qui, 23 de ago de 2018 às 14:09, Daniel Quevedo 
> escreveu:
>
>> Blz não sabia q era de lá, vou consultar. Obrigado!
>>
>> Em qui, 23 de ago de 2018 às 10:30, Claudio Buffara <
>> claudio.buff...@gmail.com> escreveu:
>>
>>> Esse é clássico. Foi o problema 6 da IMO de 1988 e é usualmente
>>> considerado o problema mais difícil proposto numa IMO, pelo menos até
>>> aquela data.
>>>
>>> Um bom ponto de partida pode ser este:
>>> https://en.wikipedia.org/wiki/Vieta_jumping
>>> Ou então: https://mks.mff.cuni.cz/kalva/imo.html
>>>
>>> []s,
>>> Claudio.
>>>
>>>
>>>
>>> 2018-08-23 9:57 GMT-03:00 Daniel Quevedo :
>>>
 Sejam a e b inteiros estritamente positivos tais que (ab + 1) é um
 divisor de (a^2 + b^2). Sobre o número  (a^2 +b^2)/(ab +1) podemos afirmar
 que é um quadrado perfeito:
 A) se, e só se, a e b também o forem.
 B) se, e só se, a e b tiverem acreana paridade
 C) se, e só se, a e b tiverem paridades distintas
 D) somente para um número finito de valores de a e b
 E) sempre

 R: e

>>> --
 Fiscal: Daniel Quevedo

>>>
 --
 Esta mensagem foi verificada pelo sistema de antivírus e
 acredita-se estar livre de perigo.
>>>
>>>
>>>
>>> --
>>> Esta mensagem foi verificada pelo sistema de antivírus e
>>> acredita-se estar livre de perigo.
>>
>> --
>> Fiscal: Daniel Quevedo
>>
>> --
>> Esta mensagem foi verificada pelo sistema de antivírus e
>> acredita-se estar livre de perigo.
>
>
>
> --
> Israel Meireles Chrisostomo
>
> --
> Esta mensagem foi verificada pelo sistema de antivírus e
> acredita-se estar livre de perigo.

-- 
Esta mensagem foi verificada pelo sistema de antiv�rus e
 acredita-se estar livre de perigo.



[obm-l] Re: [obm-l] Re: [obm-l] Re: [obm-l] Teoria dos números

2018-08-23 Por tôpico Israel Meireles Chrisostomo
Assista a esse vídeo:
https://www.youtube.com/watch?v=Cy3Vyl-jxpk

Em qui, 23 de ago de 2018 às 14:09, Daniel Quevedo 
escreveu:

> Blz não sabia q era de lá, vou consultar. Obrigado!
>
> Em qui, 23 de ago de 2018 às 10:30, Claudio Buffara <
> claudio.buff...@gmail.com> escreveu:
>
>> Esse é clássico. Foi o problema 6 da IMO de 1988 e é usualmente
>> considerado o problema mais difícil proposto numa IMO, pelo menos até
>> aquela data.
>>
>> Um bom ponto de partida pode ser este:
>> https://en.wikipedia.org/wiki/Vieta_jumping
>> Ou então: https://mks.mff.cuni.cz/kalva/imo.html
>>
>> []s,
>> Claudio.
>>
>>
>>
>> 2018-08-23 9:57 GMT-03:00 Daniel Quevedo :
>>
>>> Sejam a e b inteiros estritamente positivos tais que (ab + 1) é um
>>> divisor de (a^2 + b^2). Sobre o número  (a^2 +b^2)/(ab +1) podemos afirmar
>>> que é um quadrado perfeito:
>>> A) se, e só se, a e b também o forem.
>>> B) se, e só se, a e b tiverem acreana paridade
>>> C) se, e só se, a e b tiverem paridades distintas
>>> D) somente para um número finito de valores de a e b
>>> E) sempre
>>>
>>> R: e
>>>
>> --
>>> Fiscal: Daniel Quevedo
>>>
>>
>>> --
>>> Esta mensagem foi verificada pelo sistema de antivírus e
>>> acredita-se estar livre de perigo.
>>
>>
>>
>> --
>> Esta mensagem foi verificada pelo sistema de antivírus e
>> acredita-se estar livre de perigo.
>
> --
> Fiscal: Daniel Quevedo
>
> --
> Esta mensagem foi verificada pelo sistema de antivírus e
> acredita-se estar livre de perigo.



-- 
Israel Meireles Chrisostomo

-- 
Esta mensagem foi verificada pelo sistema de antiv�rus e
 acredita-se estar livre de perigo.



[obm-l] Re: [obm-l] Re: [obm-l] Re: [obm-l] Re: [obm-l] Re: [obm-l] Teoria dos números

2018-06-02 Por tôpico Claudio Buffara
Eu acho que o enunciado pede a soma dos elementos simplesmente porque é uma
questão de múltipla escolha.
Já vi isso antes.

E perguntei a proveniência porque me parece muito difícil para ser uma
questão de vestibular. Talvez do ITA ou da OBM (1a fase)...

***

Sobre as soluções, acho interessante que certos "preconceitos" nos levam a
caminhos totalmente distintos.

Por exemplo, ao ver aquela expressão horrorosa, com X, X5 e X5^5, a
primeira coisa que me ocorreu foi resolver numericamente, com uma planilha.
Daí me pareceu natural procurar uma cota inferior e uma cota superior pra
X, já que a planilha é finita...
Foi aí que me ocorreu a ideia de mudar a notação pra N e R, que facilitou
muito a obtenção das cotas.
Tendo obtido a solução numericamente e redigido a mensagem, dei o caso por
encerrado.
Simplesmente não me ocorreu a ideia de manipular algebricamente o N e o R
pra chegar numa expressão mais reveladora, que foi o caminho que o Bernardo
seguiu, e conseguiu resolver o problema sem planilha.

Enfim, acho interessantíssimo este tema de como surgem as idéias na
matemática, mesmo num problema elementar (mas não trivial) como esse.

***

Com relação à observação do Pedro José, me parece que existe uma diferença
entre o "conjunto solução" e "as raízes" de uma equação.

Como, em se tratando de conjuntos, {1} = {1,1,1}, eu diria que, em relação
à equação x^3 - 3x^2 + 3x - 1 = 0, por exemplo,
o conjunto solução é {1} (igual a {1,1,1}) e a soma das raízes é 3.
Ou seja, esta equação tem 3 raízes (iguais) e um conjunto solução unitário.

Sobre retas coincidentes serem paralelas, acho que esta convenção permite
que paralelismo seja uma relação de equivalência.
E ser expulso por causa disso...ou o professor era um idiota ou, num ímpeto
de adolescente, você usou um linguajar forte demais pra defender sua
posição...

Tem uma passagem no Curso de Análise, do Elon, da qual eu nunca me esqueci:
"No caso de conjuntos, escrever A = B significa que A e B são o mesmo
conjunto, ou seja, que A e B possuem os mesmos elementos." Em outras
palavras, A e B são nomes diferentes para a mesma entidade.


[]s,
Claudio.


2018-06-02 18:08 GMT-03:00 Pedro José :

> Boa noite!
> O que também achei legal nesse problema foi o fato do questionamento ser
> quanto a soma dos elementos do conjunto  solução. Embora bem sutil,
> filosoficamente falando é forte. Pois, ela descarta a interpretação de n
> raízes iguais ao invés de uma raiz de multiplicidade n.
> Todas demonstrações de que algo era único, suponha-se que havia dois e se
> provava, por absudo, que eram iguais. Então se são iguais é único.
> Fui expulso de sala de aula, uma feita, no científico, pois, argumentei
> com o professor que não estava correto o conceito de posição relativa de
> duas retas, chamando-as de duas retas paralelas coincidentes. É uma reta só
> e ela é paralela a ela mesma. Nesse exercício a raiz um tem multiplicidade
> 2, e 0 tem multiplicidade 6, mas não interfere na soma. As demais raízes
> têm multiplicidade 1.
> Para os que seguem que a soma das raízes de um polinômios é
> -An-1/An ao invés de o somatório do produto das raízes pela sua
> multiplicidade, daria 1301, o que, considero incorreto.
> Desculpem-me pela lamúria, mas até hoje não me conformo com n entidades
> iguais com n >1. Talvez a expulsão tenha gerado um trauma.
> Saudações,
> PJMS
>
>
> Em Sáb, 2 de jun de 2018 17:29, Claudio Buffara 
> escreveu:
>
>> De onde é este problema?
>> 1a fase de alguma olimpíada?
>>
>> Abs
>>
>> Enviado do meu iPhone
>>
>> Em 2 de jun de 2018, à(s) 16:15, Daniel Quevedo 
>> escreveu:
>>
>> Muito obrigado a todos. De fato com a mudança de variável fica td mais
>> fácil. Não tinha visto isso.Â
>> Obrigado
>>
>> Em sáb, 2 de jun de 2018 às 16:02, Pedro José 
>> escreveu:
>>
>>> Boa tarde.
>>> A limitação para X5 obviamente não inclui 5, foi lambança.Â
>>> Saudações,Â
>>> PJMSÂ
>>>
>>> Em Sáb, 2 de jun de 2018 15:22, Claudio Buffara <
>>> claudio.buff...@gmail.com> escreveu:
>>>
 Para |X| suficientemente grande, X^6 domina a soma dos outros termos.

 Mudando a notação, eu pus N = X e R = X5.

 Então: R^5*N^5 + R*N = N^6 + R^6.

 Caso 1: N >= 0:
 R <= 4, de modo que o lado esquerdo <= N*(1024*N^4 + 4).
 Já o lado direito >= N^6.
 N*(1024*N^4 + 4) < N^6Â
 ==> 1024*N^4 + 4 < N^5Â Â
 ==> 1024 + 4/N^4 < N
 ==> N >= 1025.
 Então, para a equação ser satisfeita, é necessário que N <= 1024.

 Caso 2: N < 0.
 Então o lado esquerdo <= 0 (com igualdade sss R = 0) e o lado direito
 é positivo.
 Logo, a equação não tem soluções com N < 0.

 Com uma planilha, eu achei apenas 5 soluções:
 0, 1, 32, 243, 1024.

 A soma destes três números é 1300.

 []s,
 Claudio.


 2018-06-02 14:10 GMT-03:00 Daniel Quevedo :

> Seja Xn o resto da divisão de X por n. ParavX inteiro a soma de todos
> os elementos do conjunto solução da 

[obm-l] Re: [obm-l] Re: [obm-l] Re: [obm-l] Re: [obm-l] Teoria dos números

2018-06-02 Por tôpico Pedro José
Boa noite!
O que também achei legal nesse problema foi o fato do questionamento ser
quanto a soma dos elementos do conjunto  solução. Embora bem sutil,
filosoficamente falando é forte. Pois, ela descarta a interpretação de n
raízes iguais ao invés de uma raiz de multiplicidade n.
Todas demonstrações de que algo era único, suponha-se que havia dois e se
provava, por absudo, que eram iguais. Então se são iguais é único.
Fui expulso de sala de aula, uma feita, no científico, pois, argumentei com
o professor que não estava correto o conceito de posição relativa de duas
retas, chamando-as de duas retas paralelas coincidentes. É uma reta só e
ela é paralela a ela mesma. Nesse exercício a raiz um tem multiplicidade 2,
e 0 tem multiplicidade 6, mas não interfere na soma. As demais raízes têm
multiplicidade 1.
Para os que seguem que a soma das raízes de um polinômios é
-An-1/An ao invés de o somatório do produto das raízes pela sua
multiplicidade, daria 1301, o que, considero incorreto.
Desculpem-me pela lamúria, mas até hoje não me conformo com n entidades
iguais com n >1. Talvez a expulsão tenha gerado um trauma.
Saudações,
PJMS

Em Sáb, 2 de jun de 2018 17:29, Claudio Buffara 
escreveu:

> De onde é este problema?
> 1a fase de alguma olimpíada?
>
> Abs
>
> Enviado do meu iPhone
>
> Em 2 de jun de 2018, à(s) 16:15, Daniel Quevedo 
> escreveu:
>
> Muito obrigado a todos. De fato com a mudança de variável fica td mais
> fácil. Não tinha visto isso.Â
> Obrigado
>
> Em sáb, 2 de jun de 2018 às 16:02, Pedro José 
> escreveu:
>
>> Boa tarde.
>> A limitação para X5 obviamente não inclui 5, foi lambança.Â
>> Saudações,Â
>> PJMSÂ
>>
>> Em Sáb, 2 de jun de 2018 15:22, Claudio Buffara <
>> claudio.buff...@gmail.com> escreveu:
>>
>>> Para |X| suficientemente grande, X^6 domina a soma dos outros termos.
>>>
>>> Mudando a notação, eu pus N = X e R = X5.
>>>
>>> Então: R^5*N^5 + R*N = N^6 + R^6.
>>>
>>> Caso 1: N >= 0:
>>> R <= 4, de modo que o lado esquerdo <= N*(1024*N^4 + 4).
>>> Já o lado direito >= N^6.
>>> N*(1024*N^4 + 4) < N^6Â
>>> ==> 1024*N^4 + 4 < N^5Â Â
>>> ==> 1024 + 4/N^4 < N
>>> ==> N >= 1025.
>>> Então, para a equação ser satisfeita, é necessário que N <= 1024.
>>>
>>> Caso 2: N < 0.
>>> Então o lado esquerdo <= 0 (com igualdade sss R = 0) e o lado direito
>>> é positivo.
>>> Logo, a equação não tem soluções com N < 0.
>>>
>>> Com uma planilha, eu achei apenas 5 soluções:
>>> 0, 1, 32, 243, 1024.
>>>
>>> A soma destes três números é 1300.
>>>
>>> []s,
>>> Claudio.
>>>
>>>
>>> 2018-06-02 14:10 GMT-03:00 Daniel Quevedo :
>>>
 Seja Xn o resto da divisão de X por n. ParavX inteiro a soma de todos
 os elementos do conjunto solução da equação: [(X5)^5].X^5 - X^6 -
 (X5)^6 +X.(X5) = 0
 É igual a:
 A) 1100
 B) 1300
 C) 1500
 D) 1700
 E) 1900

 R: b
 --
 Fiscal: Daniel Quevedo

 --
 Esta mensagem foi verificada pelo sistema de antivírus e
 acredita-se estar livre de perigo.
>>>
>>>
>>>
>>> --
>>> Esta mensagem foi verificada pelo sistema de antivírus e
>>> acredita-se estar livre de perigo.
>>
>>
>> --
>> Esta mensagem foi verificada pelo sistema de antivírus e
>> acredita-se estar livre de perigo.
>
> --
> Fiscal: Daniel Quevedo
>
> --
> Esta mensagem foi verificada pelo sistema de antivírus e
> acredita-se estar livre de perigo.
>
>
> --
> Esta mensagem foi verificada pelo sistema de antivírus e
> acredita-se estar livre de perigo.
>

-- 
Esta mensagem foi verificada pelo sistema de antiv�rus e
 acredita-se estar livre de perigo.



[obm-l] Re: [obm-l] Re: [obm-l] Re: [obm-l] Re: [obm-l] Teoria dos números

2018-06-02 Por tôpico Daniel Quevedo
De um livro q tenho. Não duvidaria q fosse d alguma olimpíada pq há muitas
questões q são tiradas daí. O nome é Problemas Selecionados de Matemática,
do Gandhi

Em sáb, 2 de jun de 2018 às 17:29, Claudio Buffara <
claudio.buff...@gmail.com> escreveu:

> De onde é este problema?
> 1a fase de alguma olimpíada?
>
> Abs
>
> Enviado do meu iPhone
>
> Em 2 de jun de 2018, à(s) 16:15, Daniel Quevedo 
> escreveu:
>
> Muito obrigado a todos. De fato com a mudança de variável fica td mais
> fácil. Não tinha visto isso.Â
> Obrigado
>
> Em sáb, 2 de jun de 2018 às 16:02, Pedro José 
> escreveu:
>
>> Boa tarde.
>> A limitação para X5 obviamente não inclui 5, foi lambança.Â
>> Saudações,Â
>> PJMSÂ
>>
>> Em Sáb, 2 de jun de 2018 15:22, Claudio Buffara <
>> claudio.buff...@gmail.com> escreveu:
>>
>>> Para |X| suficientemente grande, X^6 domina a soma dos outros termos.
>>>
>>> Mudando a notação, eu pus N = X e R = X5.
>>>
>>> Então: R^5*N^5 + R*N = N^6 + R^6.
>>>
>>> Caso 1: N >= 0:
>>> R <= 4, de modo que o lado esquerdo <= N*(1024*N^4 + 4).
>>> Já o lado direito >= N^6.
>>> N*(1024*N^4 + 4) < N^6Â
>>> ==> 1024*N^4 + 4 < N^5Â Â
>>> ==> 1024 + 4/N^4 < N
>>> ==> N >= 1025.
>>> Então, para a equação ser satisfeita, é necessário que N <= 1024.
>>>
>>> Caso 2: N < 0.
>>> Então o lado esquerdo <= 0 (com igualdade sss R = 0) e o lado direito
>>> é positivo.
>>> Logo, a equação não tem soluções com N < 0.
>>>
>>> Com uma planilha, eu achei apenas 5 soluções:
>>> 0, 1, 32, 243, 1024.
>>>
>>> A soma destes três números é 1300.
>>>
>>> []s,
>>> Claudio.
>>>
>>>
>>> 2018-06-02 14:10 GMT-03:00 Daniel Quevedo :
>>>
 Seja Xn o resto da divisão de X por n. ParavX inteiro a soma de todos
 os elementos do conjunto solução da equação: [(X5)^5].X^5 - X^6 -
 (X5)^6 +X.(X5) = 0
 É igual a:
 A) 1100
 B) 1300
 C) 1500
 D) 1700
 E) 1900

 R: b
 --
 Fiscal: Daniel Quevedo

 --
 Esta mensagem foi verificada pelo sistema de antivírus e
 acredita-se estar livre de perigo.
>>>
>>>
>>>
>>> --
>>> Esta mensagem foi verificada pelo sistema de antivírus e
>>> acredita-se estar livre de perigo.
>>
>>
>> --
>> Esta mensagem foi verificada pelo sistema de antivírus e
>> acredita-se estar livre de perigo.
>
> --
> Fiscal: Daniel Quevedo
>
> --
> Esta mensagem foi verificada pelo sistema de antivírus e
> acredita-se estar livre de perigo.
>
>
> --
> Esta mensagem foi verificada pelo sistema de antivírus e
> acredita-se estar livre de perigo.
>
-- 
Fiscal: Daniel Quevedo

-- 
Esta mensagem foi verificada pelo sistema de antiv�rus e
 acredita-se estar livre de perigo.



Re: [obm-l] Re: [obm-l] Re: [obm-l] Re: [obm-l] Teoria dos números

2018-06-02 Por tôpico Claudio Buffara
De onde é este problema?
1a fase de alguma olimpíada?

Abs

Enviado do meu iPhone

Em 2 de jun de 2018, à(s) 16:15, Daniel Quevedo  escreveu:

> Muito obrigado a todos. De fato com a mudança de variável fica td mais 
> fácil. Não tinha visto isso. 
> Obrigado
> 
> Em sáb, 2 de jun de 2018 às 16:02, Pedro José  
> escreveu:
>> Boa tarde.
>> A limitação para X5 obviamente não inclui 5, foi lambança. 
>> Saudações, 
>> PJMSÂ 
>> 
>> Em Sáb, 2 de jun de 2018 15:22, Claudio Buffara  
>> escreveu:
>>> Para |X| suficientemente grande, X^6 domina a soma dos outros termos.
>>> 
>>> Mudando a notação, eu pus N = X e R = X5.
>>> 
>>> Então: R^5*N^5 + R*N = N^6 + R^6.
>>> 
>>> Caso 1: N >= 0:
>>> R <= 4, de modo que o lado esquerdo <= N*(1024*N^4 + 4).
>>> Já o lado direito >= N^6.
>>> N*(1024*N^4 + 4) < N^6Â 
>>> ==> 1024*N^4 + 4 < N^5Â Â 
>>> ==> 1024 + 4/N^4 < N
>>> ==> N >= 1025.
>>> Então, para a equação ser satisfeita, é necessário que N <= 1024.
>>> 
>>> Caso 2: N < 0.
>>> Então o lado esquerdo <= 0 (com igualdade sss R = 0) e o lado direito é 
>>> positivo.
>>> Logo, a equação não tem soluções com N < 0.
>>> 
>>> Com uma planilha, eu achei apenas 5 soluções:
>>> 0, 1, 32, 243, 1024.
>>> 
>>> A soma destes três números é 1300.
>>> 
>>> []s,
>>> Claudio.
>>> 
>>> 
>>> 2018-06-02 14:10 GMT-03:00 Daniel Quevedo :
 Seja Xn o resto da divisão de X por n. ParavX inteiro a soma de todos os 
 elementos do conjunto solução da equação: [(X5)^5].X^5 - X^6 - (X5)^6 
 +X.(X5) = 0
 É igual a:
 A) 1100
 B) 1300
 C) 1500
 D) 1700
 E) 1900
 
 R: b
 -- 
 Fiscal: Daniel Quevedo
 
 -- 
 Esta mensagem foi verificada pelo sistema de antivírus e 
 acredita-se estar livre de perigo.
>>> 
>>> 
>>> -- 
>>> Esta mensagem foi verificada pelo sistema de antivírus e 
>>> acredita-se estar livre de perigo.
>> 
>> -- 
>> Esta mensagem foi verificada pelo sistema de antivírus e 
>> acredita-se estar livre de perigo.
> -- 
> Fiscal: Daniel Quevedo
> 
> -- 
> Esta mensagem foi verificada pelo sistema de antivírus e 
> acredita-se estar livre de perigo.

-- 
Esta mensagem foi verificada pelo sistema de antiv�rus e
 acredita-se estar livre de perigo.



[obm-l] Re: [obm-l] Re: [obm-l] Re: [obm-l] Teoria dos números

2018-06-02 Por tôpico Daniel Quevedo
Muito obrigado a todos. De fato com a mudança de variável fica td mais
fácil. Não tinha visto isso.
Obrigado

Em sáb, 2 de jun de 2018 às 16:02, Pedro José 
escreveu:

> Boa tarde.
> A limitação para X5 obviamente não inclui 5, foi lambança.
> Saudações,
> PJMS
>
> Em Sáb, 2 de jun de 2018 15:22, Claudio Buffara 
> escreveu:
>
>> Para |X| suficientemente grande, X^6 domina a soma dos outros termos.
>>
>> Mudando a notação, eu pus N = X e R = X5.
>>
>> Então: R^5*N^5 + R*N = N^6 + R^6.
>>
>> Caso 1: N >= 0:
>> R <= 4, de modo que o lado esquerdo <= N*(1024*N^4 + 4).
>> Já o lado direito >= N^6.
>> N*(1024*N^4 + 4) < N^6
>> ==> 1024*N^4 + 4 < N^5
>> ==> 1024 + 4/N^4 < N
>> ==> N >= 1025.
>> Então, para a equação ser satisfeita, é necessário que N <= 1024.
>>
>> Caso 2: N < 0.
>> Então o lado esquerdo <= 0 (com igualdade sss R = 0) e o lado direito é
>> positivo.
>> Logo, a equação não tem soluções com N < 0.
>>
>> Com uma planilha, eu achei apenas 5 soluções:
>> 0, 1, 32, 243, 1024.
>>
>> A soma destes três números é 1300.
>>
>> []s,
>> Claudio.
>>
>>
>> 2018-06-02 14:10 GMT-03:00 Daniel Quevedo :
>>
>>> Seja Xn o resto da divisão de X por n. ParavX inteiro a soma de todos os
>>> elementos do conjunto solução da equação: [(X5)^5].X^5 - X^6 - (X5)^6
>>> +X.(X5) = 0
>>> É igual a:
>>> A) 1100
>>> B) 1300
>>> C) 1500
>>> D) 1700
>>> E) 1900
>>>
>>> R: b
>>> --
>>> Fiscal: Daniel Quevedo
>>>
>>> --
>>> Esta mensagem foi verificada pelo sistema de antivírus e
>>> acredita-se estar livre de perigo.
>>
>>
>>
>> --
>> Esta mensagem foi verificada pelo sistema de antivírus e
>> acredita-se estar livre de perigo.
>
>
> --
> Esta mensagem foi verificada pelo sistema de antivírus e
> acredita-se estar livre de perigo.

-- 
Fiscal: Daniel Quevedo

-- 
Esta mensagem foi verificada pelo sistema de antiv�rus e
 acredita-se estar livre de perigo.



[obm-l] Re: [obm-l] Re: [obm-l] Re: [obm-l] Re: [obm-l] Re: [obm-l] Teoria dos números

2018-04-10 Por tôpico Pedro José
Bom dia!

Dei uma "roubadinha" e achei outra solução, pois veio de trás para a
frente. Veio da observação que nas respostas u=st.

(s-1)(t-1)(u-1) | ust-1 1=2 e só atende quando k(s,t,u) é
inteiro.
Fixando-se duas váriaveis  k é monótona decrescente para a outra; assim
kmax(s) = k(s,s+1,s+2)= (s(s+1)(s+2)-1)/(s-1)s(s+1)>=2, então
s(s+1)(s+2)/s(s-1)(s+1)>2; s < 4.

(s-1)(t-1)(u-1) | ust-1 então: (u-1) | ust-1, então: (u-1) | ust -st; então
(u-1) | st-1

temos que m (u-1) = st -1, (u-1) = (st-1)/m, com m inteiro. Então m < s,
para que u > t.

s=2, só serve m =1==> u=st. donde: (s-1)(t-1)(u-1) | (st)^2-1 ;
(s-1)(t-1)(u-1) | (u-1)(st+1); (s-1)(t-1) | (st+1)

(t-1)| | 2t+1; (t-1) | 3. t-1 <= 3 ==> t=4 (paridade -se uma das incógnitas
for par todas serão e se uma for ímpar todas serão - e t>s)  e u =st=8, que
verificando atende a proposição. (2,4,8)

s=3 há duas opções m=1 ou m=2

com m=1. (s-1)(t-1) | (st+1);  2(t-1) | 3t+1 ; 2(t-1) | t-1 <=4 ==> t=5
(paridade e t>s) c= st = 15 e atende a proposição (3,5,15)

com m=2 u= (st-1)/2 +1 ==> (s-1)(t-1) | ((st-1)/2 +1)st -1; 2(t-1) <=
(9t^2+3t)/2 -1; impossível.

Logo só há as soluções anteriores (2,4,8) e (3,5,15).

Saudações,
PJMS.

Em 26 de março de 2018 10:49, Pedro José  escreveu:

> Bom dia!
> Agora estou contente. Posso alardear que pelo menos matei um problema da
> IMO.
>
> (s-1)(t-1)(u-1) | ust-1 1 k(s,t,u) = (stu-1)/(s-1)(t-1)(u-1)
>
> Como k(s,t,u) > 1 temos que k(s,t,u) >=2 e só atende quando k(s,t,u) é
> inteiro.
> Fixando-se duas váriaveis  k é monótona decrescente para a outra; assim
> kmax(s) = k(s,s+1,s+2)= (s(s+1)(s+2)-1)/(s-1)s(s+1)>=2, então
> s(s+1)(s+2)/s(s-1)(s+1)>2; s < 4.
>
> fazendo um estudo de paridade: se uma das variáveis for par as outras duas
> também serão e k será ímpar. Se uma das variáveis for ímpar, todas serão
> ímpares e k poderá ser tanto ímpar quanto par.
>
> u   s   v  k
> P  P   P I
> III  -
>
> s=2. k>=3 Para kmax (2,t) = k(2,t,t+1) = (2t(t+1)-1)/t(t-1)>=3 então:
> 2t(t+1)/t(t-1) >3 : t < 5, pela paridade t=4 e kmax(2,4) = 47/15, só serve
> k = 3.
>
> s=2, t=4 e k=3 temos v=8. (2,4,8)
>
> s=3 k>=2 Para kmax (3,t) = k(3,t,t+1) = (3t(t+1)-1)/2t(t-1)>=2 então:
> 3t(t+1)/2t(t-1) >2 : t < 7, pela paridade t=5 e kmax(3,5) = 13/6, só serve
> k = 2.
>
> s=3, t= 5 e k=2 temos v= 15. (3,5,15)
>
> Só atendem: (2,4,8) e (3,5,15)
>
> Achei curioso que em ambas soluções, u=st.
>
> Saudações,
> PJMS
>
>
>
> Em 26 de março de 2018 09:44, Matheus Secco 
> escreveu:
>
>> De fato, trata-se do problema 1 da IMO 1992.
>>
>> Abs,
>>
>> Matheus Secco
>>
>> Em Seg, 26 de mar de 2018 09:24, Claudio Buffara <
>> claudio.buff...@gmail.com> escreveu:
>>
>>> Muito fácil pra ser de IMO...
>>>
>>> 2018-03-26 6:58 GMT-03:00 Anderson Torres 
>>> :
>>>
 Este não é o problema de alguma IMO não? Eu lembro de ter resolvido,
 quase igual à solução oficial: substituir s,t,u por a+1,b+1,c+1 e
 calcular os possiveis valores de
 1/a+1/b+1/c + 1/ab+1/ac+1/bc usando desigualdades - para daí limitar
 os valores de a,b,c.

 Em 23 de março de 2018 17:01, Claudio Buffara
  escreveu:
 > Enfim, nesse meio tempo acho que resolvi o problema...
 >
 > Devemos achar inteiros s, t, u, com 1 < s < t < u e tais que:
 > (stu -1)/((s-1)(t-1)(u-1)) = k  (k inteiro positivo)
 >
 > Após diversas aplicações do truque (método?) de somar e subtrair a
 mesma
 > coisa, chegamos a:
 > stu - 1 =  (s-1)(t-1)(u-1) + (s-1)(t-1) + (s-1)(u-1) + (t-1)(u-1) +
 (s-1) +
 > (t-1) + (u-1)
 >
 > Dividindo isso por (s-1)(t-1)(u-1), obtemos:
 > 1 + 1/(u-1) + 1/(t-1) + 1/(s-1) + 1/((t-1)(u-1)) + 1/((s-1)(u-1)) +
 > 1/((s-1)(t-1)) = k ==>
 >
 > 1/(u-1) + 1/(t-1) + 1/(s-1) + 1/((t-1)(u-1)) + 1/((s-1)(u-1)) +
 > 1/((s-1)(t-1)) = k-1
 >
 > Agora a ideia é achar cotas para s e para k.
 >
 > 1 < s < t < u ==> s >= 2, t >= 3 e u >= 4 ==> o lado esquerdo é menor
 ou
 > igual que:
 > 1/3 + 1/2 + 1 + 1/6 + 1/3 + 1/2 = 2+5/6
 >
 > Ou seja, como o lado esquerdo é inteiro (e positivo), só poderá ser
 igual a
 > 1 ou a 2 ==> k = 2 ou k = 3.
 >
 > Se s >= 4, então t >= 5 e u >= 6, e o lado esquerdo será, no máximo,
 igual
 > a:
 > 1/5 + 1/4 + 1/3 + 1/20 + 1/15 + 1/12 < 1.
 >
 > Logo, devemos ter s = 2 ou s = 3.
 >
 > s = 2 ==>
 > 1/(u-1) + 1/(t-1) + 1 + 1/((t-1)(u-1)) + 1/(u-1) + 1/(t-1) = k-1 ==>
 > 2/(t-1) + 2/(u-1) + 1/((t-1)(u-1)) = k-2 ==>
 > Como k-2 deve ser inteiro positivo, k só pode ser 3 e, portanto:
 > 2/(t-1) + 2/(u-1) + 1/((t-1)(u-1)) = 1 ==>
 > (2 + 1/(t-1))/(u-1) = 1 - 2/(t-1) ==>
 > u = 1 + (2t - 1)/(t - 3) = 3 + 5/(t-3) ==>
 > t = 4 e u = 8   ou   t = 8 e u = 4 (não serve pois t deve ser menor
 do que
 > u)
 >
 > s = 3 ==>
 > 1/(u-1) + 

[obm-l] Re: [obm-l] Re: [obm-l] Re: [obm-l] Re: [obm-l] Re: [obm-l] Teoria dos números

2018-03-26 Por tôpico Pedro José
Bom dia!
Agora estou contente. Posso alardear que pelo menos matei um problema da
IMO.

(s-1)(t-1)(u-1) | ust-1 1=2 e só atende quando k(s,t,u) é
inteiro.
Fixando-se duas váriaveis  k é monótona decrescente para a outra; assim
kmax(s) = k(s,s+1,s+2)= (s(s+1)(s+2)-1)/(s-1)s(s+1)>=2, então
s(s+1)(s+2)/s(s-1)(s+1)>2; s < 4.

fazendo um estudo de paridade: se uma das variáveis for par as outras duas
também serão e k será ímpar. Se uma das variáveis for ímpar, todas serão
ímpares e k poderá ser tanto ímpar quanto par.

u   s   v  k
P  P   P I
III  -

s=2. k>=3 Para kmax (2,t) = k(2,t,t+1) = (2t(t+1)-1)/t(t-1)>=3 então:
2t(t+1)/t(t-1) >3 : t < 5, pela paridade t=4 e kmax(2,4) = 47/15, só serve
k = 3.

s=2, t=4 e k=3 temos v=8. (2,4,8)

s=3 k>=2 Para kmax (3,t) = k(3,t,t+1) = (3t(t+1)-1)/2t(t-1)>=2 então:
3t(t+1)/2t(t-1) >2 : t < 7, pela paridade t=5 e kmax(3,5) = 13/6, só serve
k = 2.

s=3, t= 5 e k=2 temos v= 15. (3,5,15)

Só atendem: (2,4,8) e (3,5,15)

Achei curioso que em ambas soluções, u=st.

Saudações,
PJMS



Em 26 de março de 2018 09:44, Matheus Secco 
escreveu:

> De fato, trata-se do problema 1 da IMO 1992.
>
> Abs,
>
> Matheus Secco
>
> Em Seg, 26 de mar de 2018 09:24, Claudio Buffara <
> claudio.buff...@gmail.com> escreveu:
>
>> Muito fácil pra ser de IMO...
>>
>> 2018-03-26 6:58 GMT-03:00 Anderson Torres :
>>
>>> Este não é o problema de alguma IMO não? Eu lembro de ter resolvido,
>>> quase igual à solução oficial: substituir s,t,u por a+1,b+1,c+1 e
>>> calcular os possiveis valores de
>>> 1/a+1/b+1/c + 1/ab+1/ac+1/bc usando desigualdades - para daí limitar
>>> os valores de a,b,c.
>>>
>>> Em 23 de março de 2018 17:01, Claudio Buffara
>>>  escreveu:
>>> > Enfim, nesse meio tempo acho que resolvi o problema...
>>> >
>>> > Devemos achar inteiros s, t, u, com 1 < s < t < u e tais que:
>>> > (stu -1)/((s-1)(t-1)(u-1)) = k  (k inteiro positivo)
>>> >
>>> > Após diversas aplicações do truque (método?) de somar e subtrair a
>>> mesma
>>> > coisa, chegamos a:
>>> > stu - 1 =  (s-1)(t-1)(u-1) + (s-1)(t-1) + (s-1)(u-1) + (t-1)(u-1) +
>>> (s-1) +
>>> > (t-1) + (u-1)
>>> >
>>> > Dividindo isso por (s-1)(t-1)(u-1), obtemos:
>>> > 1 + 1/(u-1) + 1/(t-1) + 1/(s-1) + 1/((t-1)(u-1)) + 1/((s-1)(u-1)) +
>>> > 1/((s-1)(t-1)) = k ==>
>>> >
>>> > 1/(u-1) + 1/(t-1) + 1/(s-1) + 1/((t-1)(u-1)) + 1/((s-1)(u-1)) +
>>> > 1/((s-1)(t-1)) = k-1
>>> >
>>> > Agora a ideia é achar cotas para s e para k.
>>> >
>>> > 1 < s < t < u ==> s >= 2, t >= 3 e u >= 4 ==> o lado esquerdo é menor
>>> ou
>>> > igual que:
>>> > 1/3 + 1/2 + 1 + 1/6 + 1/3 + 1/2 = 2+5/6
>>> >
>>> > Ou seja, como o lado esquerdo é inteiro (e positivo), só poderá ser
>>> igual a
>>> > 1 ou a 2 ==> k = 2 ou k = 3.
>>> >
>>> > Se s >= 4, então t >= 5 e u >= 6, e o lado esquerdo será, no máximo,
>>> igual
>>> > a:
>>> > 1/5 + 1/4 + 1/3 + 1/20 + 1/15 + 1/12 < 1.
>>> >
>>> > Logo, devemos ter s = 2 ou s = 3.
>>> >
>>> > s = 2 ==>
>>> > 1/(u-1) + 1/(t-1) + 1 + 1/((t-1)(u-1)) + 1/(u-1) + 1/(t-1) = k-1 ==>
>>> > 2/(t-1) + 2/(u-1) + 1/((t-1)(u-1)) = k-2 ==>
>>> > Como k-2 deve ser inteiro positivo, k só pode ser 3 e, portanto:
>>> > 2/(t-1) + 2/(u-1) + 1/((t-1)(u-1)) = 1 ==>
>>> > (2 + 1/(t-1))/(u-1) = 1 - 2/(t-1) ==>
>>> > u = 1 + (2t - 1)/(t - 3) = 3 + 5/(t-3) ==>
>>> > t = 4 e u = 8   ou   t = 8 e u = 4 (não serve pois t deve ser menor do
>>> que
>>> > u)
>>> >
>>> > s = 3 ==>
>>> > 1/(u-1) + 1/(t-1) + 1/2 + 1/((t-1)(u-1)) + 1/(2(u-1)) + 1/(2(t-1)) =
>>> k-1 ==>
>>> > (3/2)/(u-1) + (3/2)/(t-1) + 1/((t-1)(u-1)) = k - 3/2 ==>
>>> > 3/(u-1) + 3/(t-1) + 2/((t-1)(u-1)) = 2k - 3 ==>
>>> > (3 + 2/(t-1))/(u-1) = 2k - 3t/(t-1) ==>
>>> > (3t - 1)/(u-1) = 2k(t-1) - 3t ==>
>>> > u = 1 + (3t - 1)/((2k-3)t - 2k)
>>> >
>>> > k = 2 ==> u = 1 + (3t-1)/(t-4) = 4 + 11/(t-4) ==> t = 5 e u = 15
>>> >
>>> > k = 3 ==> u = 1 + (3t-1)/(3t-6) = 2 + 5/(3t-6) ==> XXX
>>> >
>>> > As únicas soluções são:
>>> > (2,4,8) e (3,5,15)
>>> >
>>> > []s,
>>> > Claudio.
>>> >
>>> > 2018-03-23 15:38 GMT-03:00 Pedro José :
>>> >>
>>> >> Boa tarde!
>>> >>
>>> >> Aproveitando que deu o que falar o problema postado pelo Douglas, tem
>>> um
>>> >> que achei mais interessante.
>>> >>
>>> >> (s-1)(t-1).(u-1) | stu -1, com s, t, u inteiros  e 1 > >>
>>> >> Saudações,
>>> >> Pedro
>>> >>
>>> >> --
>>> >> Esta mensagem foi verificada pelo sistema de antivírus e
>>> >> acredita-se estar livre de perigo.
>>> >
>>> >
>>> >
>>> > --
>>> > Esta mensagem foi verificada pelo sistema de antivírus e
>>> > acredita-se estar livre de perigo.
>>>
>>> --
>>> Esta mensagem foi verificada pelo sistema de antivírus e
>>>  acredita-se estar livre de perigo.
>>>
>>>
>>> 
>>> =
>>> Instru�ões para entrar na lista, sair da lista e usar a lista em
>>> http://www.mat.puc-rio.br/~obmlistas/obm-l.html
>>> 

[obm-l] Re: [obm-l] Re: [obm-l] Re: [obm-l] Re: [obm-l] Teoria dos números

2018-03-26 Por tôpico Matheus Secco
De fato, trata-se do problema 1 da IMO 1992.

Abs,

Matheus Secco

Em Seg, 26 de mar de 2018 09:24, Claudio Buffara 
escreveu:

> Muito fácil pra ser de IMO...
>
> 2018-03-26 6:58 GMT-03:00 Anderson Torres :
>
>> Este não é o problema de alguma IMO não? Eu lembro de ter resolvido,
>> quase igual à solução oficial: substituir s,t,u por a+1,b+1,c+1 e
>> calcular os possiveis valores de
>> 1/a+1/b+1/c + 1/ab+1/ac+1/bc usando desigualdades - para daí limitar
>> os valores de a,b,c.
>>
>> Em 23 de março de 2018 17:01, Claudio Buffara
>>  escreveu:
>> > Enfim, nesse meio tempo acho que resolvi o problema...
>> >
>> > Devemos achar inteiros s, t, u, com 1 < s < t < u e tais que:
>> > (stu -1)/((s-1)(t-1)(u-1)) = k  (k inteiro positivo)
>> >
>> > Após diversas aplicações do truque (método?) de somar e subtrair a mesma
>> > coisa, chegamos a:
>> > stu - 1 =  (s-1)(t-1)(u-1) + (s-1)(t-1) + (s-1)(u-1) + (t-1)(u-1) +
>> (s-1) +
>> > (t-1) + (u-1)
>> >
>> > Dividindo isso por (s-1)(t-1)(u-1), obtemos:
>> > 1 + 1/(u-1) + 1/(t-1) + 1/(s-1) + 1/((t-1)(u-1)) + 1/((s-1)(u-1)) +
>> > 1/((s-1)(t-1)) = k ==>
>> >
>> > 1/(u-1) + 1/(t-1) + 1/(s-1) + 1/((t-1)(u-1)) + 1/((s-1)(u-1)) +
>> > 1/((s-1)(t-1)) = k-1
>> >
>> > Agora a ideia é achar cotas para s e para k.
>> >
>> > 1 < s < t < u ==> s >= 2, t >= 3 e u >= 4 ==> o lado esquerdo é menor ou
>> > igual que:
>> > 1/3 + 1/2 + 1 + 1/6 + 1/3 + 1/2 = 2+5/6
>> >
>> > Ou seja, como o lado esquerdo é inteiro (e positivo), só poderá ser
>> igual a
>> > 1 ou a 2 ==> k = 2 ou k = 3.
>> >
>> > Se s >= 4, então t >= 5 e u >= 6, e o lado esquerdo será, no máximo,
>> igual
>> > a:
>> > 1/5 + 1/4 + 1/3 + 1/20 + 1/15 + 1/12 < 1.
>> >
>> > Logo, devemos ter s = 2 ou s = 3.
>> >
>> > s = 2 ==>
>> > 1/(u-1) + 1/(t-1) + 1 + 1/((t-1)(u-1)) + 1/(u-1) + 1/(t-1) = k-1 ==>
>> > 2/(t-1) + 2/(u-1) + 1/((t-1)(u-1)) = k-2 ==>
>> > Como k-2 deve ser inteiro positivo, k só pode ser 3 e, portanto:
>> > 2/(t-1) + 2/(u-1) + 1/((t-1)(u-1)) = 1 ==>
>> > (2 + 1/(t-1))/(u-1) = 1 - 2/(t-1) ==>
>> > u = 1 + (2t - 1)/(t - 3) = 3 + 5/(t-3) ==>
>> > t = 4 e u = 8   ou   t = 8 e u = 4 (não serve pois t deve ser menor do
>> que
>> > u)
>> >
>> > s = 3 ==>
>> > 1/(u-1) + 1/(t-1) + 1/2 + 1/((t-1)(u-1)) + 1/(2(u-1)) + 1/(2(t-1)) =
>> k-1 ==>
>> > (3/2)/(u-1) + (3/2)/(t-1) + 1/((t-1)(u-1)) = k - 3/2 ==>
>> > 3/(u-1) + 3/(t-1) + 2/((t-1)(u-1)) = 2k - 3 ==>
>> > (3 + 2/(t-1))/(u-1) = 2k - 3t/(t-1) ==>
>> > (3t - 1)/(u-1) = 2k(t-1) - 3t ==>
>> > u = 1 + (3t - 1)/((2k-3)t - 2k)
>> >
>> > k = 2 ==> u = 1 + (3t-1)/(t-4) = 4 + 11/(t-4) ==> t = 5 e u = 15
>> >
>> > k = 3 ==> u = 1 + (3t-1)/(3t-6) = 2 + 5/(3t-6) ==> XXX
>> >
>> > As únicas soluções são:
>> > (2,4,8) e (3,5,15)
>> >
>> > []s,
>> > Claudio.
>> >
>> > 2018-03-23 15:38 GMT-03:00 Pedro José :
>> >>
>> >> Boa tarde!
>> >>
>> >> Aproveitando que deu o que falar o problema postado pelo Douglas, tem
>> um
>> >> que achei mais interessante.
>> >>
>> >> (s-1)(t-1).(u-1) | stu -1, com s, t, u inteiros  e 1  >>
>> >> Saudações,
>> >> Pedro
>> >>
>> >> --
>> >> Esta mensagem foi verificada pelo sistema de antivírus e
>> >> acredita-se estar livre de perigo.
>> >
>> >
>> >
>> > --
>> > Esta mensagem foi verificada pelo sistema de antivírus e
>> > acredita-se estar livre de perigo.
>>
>> --
>> Esta mensagem foi verificada pelo sistema de antivírus e
>>  acredita-se estar livre de perigo.
>>
>>
>> =
>> Instru�ões para entrar na lista, sair da lista e usar a lista em
>> http://www.mat.puc-rio.br/~obmlistas/obm-l.html
>> =
>>
>
>
> --
> Esta mensagem foi verificada pelo sistema de antivírus e
> acredita-se estar livre de perigo.

-- 
Esta mensagem foi verificada pelo sistema de antiv�rus e
 acredita-se estar livre de perigo.



[obm-l] Re: [obm-l] Re: [obm-l] Re: [obm-l] Teoria dos números

2018-03-26 Por tôpico Claudio Buffara
Muito fácil pra ser de IMO...

2018-03-26 6:58 GMT-03:00 Anderson Torres :

> Este não é o problema de alguma IMO não? Eu lembro de ter resolvido,
> quase igual à solução oficial: substituir s,t,u por a+1,b+1,c+1 e
> calcular os possiveis valores de
> 1/a+1/b+1/c + 1/ab+1/ac+1/bc usando desigualdades - para daí limitar
> os valores de a,b,c.
>
> Em 23 de março de 2018 17:01, Claudio Buffara
>  escreveu:
> > Enfim, nesse meio tempo acho que resolvi o problema...
> >
> > Devemos achar inteiros s, t, u, com 1 < s < t < u e tais que:
> > (stu -1)/((s-1)(t-1)(u-1)) = k  (k inteiro positivo)
> >
> > Após diversas aplicações do truque (método?) de somar e subtrair a mesma
> > coisa, chegamos a:
> > stu - 1 =  (s-1)(t-1)(u-1) + (s-1)(t-1) + (s-1)(u-1) + (t-1)(u-1) +
> (s-1) +
> > (t-1) + (u-1)
> >
> > Dividindo isso por (s-1)(t-1)(u-1), obtemos:
> > 1 + 1/(u-1) + 1/(t-1) + 1/(s-1) + 1/((t-1)(u-1)) + 1/((s-1)(u-1)) +
> > 1/((s-1)(t-1)) = k ==>
> >
> > 1/(u-1) + 1/(t-1) + 1/(s-1) + 1/((t-1)(u-1)) + 1/((s-1)(u-1)) +
> > 1/((s-1)(t-1)) = k-1
> >
> > Agora a ideia é achar cotas para s e para k.
> >
> > 1 < s < t < u ==> s >= 2, t >= 3 e u >= 4 ==> o lado esquerdo é menor ou
> > igual que:
> > 1/3 + 1/2 + 1 + 1/6 + 1/3 + 1/2 = 2+5/6
> >
> > Ou seja, como o lado esquerdo é inteiro (e positivo), só poderá ser
> igual a
> > 1 ou a 2 ==> k = 2 ou k = 3.
> >
> > Se s >= 4, então t >= 5 e u >= 6, e o lado esquerdo será, no máximo,
> igual
> > a:
> > 1/5 + 1/4 + 1/3 + 1/20 + 1/15 + 1/12 < 1.
> >
> > Logo, devemos ter s = 2 ou s = 3.
> >
> > s = 2 ==>
> > 1/(u-1) + 1/(t-1) + 1 + 1/((t-1)(u-1)) + 1/(u-1) + 1/(t-1) = k-1 ==>
> > 2/(t-1) + 2/(u-1) + 1/((t-1)(u-1)) = k-2 ==>
> > Como k-2 deve ser inteiro positivo, k só pode ser 3 e, portanto:
> > 2/(t-1) + 2/(u-1) + 1/((t-1)(u-1)) = 1 ==>
> > (2 + 1/(t-1))/(u-1) = 1 - 2/(t-1) ==>
> > u = 1 + (2t - 1)/(t - 3) = 3 + 5/(t-3) ==>
> > t = 4 e u = 8   ou   t = 8 e u = 4 (não serve pois t deve ser menor do
> que
> > u)
> >
> > s = 3 ==>
> > 1/(u-1) + 1/(t-1) + 1/2 + 1/((t-1)(u-1)) + 1/(2(u-1)) + 1/(2(t-1)) = k-1
> ==>
> > (3/2)/(u-1) + (3/2)/(t-1) + 1/((t-1)(u-1)) = k - 3/2 ==>
> > 3/(u-1) + 3/(t-1) + 2/((t-1)(u-1)) = 2k - 3 ==>
> > (3 + 2/(t-1))/(u-1) = 2k - 3t/(t-1) ==>
> > (3t - 1)/(u-1) = 2k(t-1) - 3t ==>
> > u = 1 + (3t - 1)/((2k-3)t - 2k)
> >
> > k = 2 ==> u = 1 + (3t-1)/(t-4) = 4 + 11/(t-4) ==> t = 5 e u = 15
> >
> > k = 3 ==> u = 1 + (3t-1)/(3t-6) = 2 + 5/(3t-6) ==> XXX
> >
> > As únicas soluções são:
> > (2,4,8) e (3,5,15)
> >
> > []s,
> > Claudio.
> >
> > 2018-03-23 15:38 GMT-03:00 Pedro José :
> >>
> >> Boa tarde!
> >>
> >> Aproveitando que deu o que falar o problema postado pelo Douglas, tem um
> >> que achei mais interessante.
> >>
> >> (s-1)(t-1).(u-1) | stu -1, com s, t, u inteiros  e 1 >
> >> Saudações,
> >> Pedro
> >>
> >> --
> >> Esta mensagem foi verificada pelo sistema de antivírus e
> >> acredita-se estar livre de perigo.
> >
> >
> >
> > --
> > Esta mensagem foi verificada pelo sistema de antivírus e
> > acredita-se estar livre de perigo.
>
> --
> Esta mensagem foi verificada pelo sistema de antivírus e
>  acredita-se estar livre de perigo.
>
>
> =
> Instru�ões para entrar na lista, sair da lista e usar a lista em
> http://www.mat.puc-rio.br/~obmlistas/obm-l.html
> =
>

-- 
Esta mensagem foi verificada pelo sistema de antiv�rus e
 acredita-se estar livre de perigo.



[obm-l] Re: [obm-l] Re: [obm-l] Re: [obm-l] Re: [obm-l] Re: [obm-l] Teoria dos números

2018-03-26 Por tôpico Anderson Torres
Em 23 de março de 2018 10:35, Claudio Buffara
 escreveu:
> Na verdade os meus questionamentos surgiram por causa do meu interesse em
> ensino de matemática.
>
> Por exemplo, produtos notáveis e fatorações são notoriamente mal ensinados,
> pelo menos nos livros didáticos de 8o e 9o ano que eu examinei.

Eu acho que uma motivação mais geométrica pode ser bastante útil para
muitos produtos notáveis.

Por exemplo, a diferença de quadrados é bem facilmente explicada de
forma geométrica:
um quadrado com um quadradinho a menos no canto pode ser quebrado em
dois trapézios que formam
um retângulo.
O quadrado da soma é mais fácil ainda.

Por outro lado, eu não penso que minha solução foi a mais mágica de
todas, apenas era desconhecida.
Sempre que noto uma expressão simétrica, eu penso em como escrevê-la
em função ou dos
polinômios simétricos elementares ou da soma de potências (x^k+y^k+z^k).

> Nenhum menciona que:
> a) as generalizações de (x+y)^2 = x^2 + 2xy + y^2  e  x^2 - y^2 = (x-y)(x+y)
> para expoentes maiores levam ao teorema do binômio (erroneamente chamado de
> binômio de Newton - nota histórica: Newton generalizou o teorema para
> expoentes racionais) e à fórmula da soma dos termos de uma PG;
> b) (x+y)^2 = x^2 + 2xy + y^2 é a base para a ideia de se completar
> quadrados, a qual, por sua vez, não só resulta na fórmula para as raízes de
> uma equação quadrática, mas também na elucidação das propriedades da função
> quadrática;
> c) o uso inteligente da expansão de (x+y)^3 leva à formula das raízes de uma
> equação cúbica.

Essa eu não conhecia. Ainda penso que as formas mais naturais de lidar
com a cúbica são o Método Gugu-Euler
(tentar uma solução da forma x=raizcúbica(y1)+raizcúbica(y2)) ou usar
a fatoração de x^3+y^3+z^3-3xyz.

Mas o cubo da soma, per se? Isso me parece mágico demais.

(Acho que até imagino o que seja: identificar o termo constante com a
soma de cubos e o termo "linear" com o triplo do produto...)

>
> ***
>
> Há tempos, o Hermann, participante desta lista, postou uma dúvida sobre
> produtos notáveis e pediu dicas de livros com exercícios sobre produtos
> notáveis e fatoração.
> Eu tenho duas sugestões, ambas em inglês:
> - Algebra, de I.M.Gelfand e A.Shen - Birkhäuser (este faz as generalizações
> que eu mencionei acima)
> - A Problem Book in Algebra, de V.A. Krechmar - Mir Publishers (pros
> entusiastas)
> Ambos estão disponíveis na Amazon.
>
> ***
>
> Anos atrás eu gostava de soluções "mágicas", obtidas por meio de alguma
> sacada brilhante que eu jamais conseguiria ter.
> Após me deparar com várias destas soluções, me ocorreu que elas talvez
> tivessem um efeito perverso na motivação dos estudantes de matemática, pois
> passavam a impressão de que é preciso ser um gênio para dominar a matéria.
> Daí o meu interesse em saber como vocês obtiveram certas fatorações.
> Entendo que trabalho braçal, experiência, alguma lógica e um pouco de
> otimismo são, para a maioria de nós, as únicas formas de progredir na
> resolução de um problema como o que deu origem a este thread.
>
> Dito isso (e posso estar enganado) nem o Pedro José e nem mais ninguém
> explicou de onde veio a conjectura (correta) de que:
> z = -(x+y)/2 é solução de (x + y)(y + z)(z + x)/2 + (x + y + z)3 =  – xyz
>
> []s,
> Claudio.
>
>
>
> 2018-03-23 6:20 GMT-03:00 Anderson Torres :
>>
>> Em 21 de março de 2018 09:47, Claudio Buffara
>>  escreveu:
>> > Como você passou de:
>> > 4abc + (a+b+c)^3 + (-a+b+c)(a-b+c)(a+b-c) = 1
>> >
>> > Para:
>> > 4(a+b+c)(ab+ac+bc) - 4abc = 1
>>
>> It's kind of magic. Eu simplesmente abri tudo com vontade e notei
>> certas repetições
>> que sempre aparecem em certas fatorações; ou melhor dizendo, estava
>> pensando em
>> escrever tudo em termos dos famigerados polinômios simétricos e cheguei
>> nisso.
>>
>> Sempre que vejo algo como (a^2b+ab^2), já escrevo ab(a+b) e tento
>> procurar um abc
>> para isso resultar em ab(a+b+c).
>>
>> Mas não avancei daí. Penso que dá para fatorar ainda mais...
>>
>> >
>> > ???
>> >
>> > []s,
>> > Claudio.
>> >
>> >
>> > 2018-03-20 23:14 GMT-03:00 Anderson Torres
>> > :
>> >>
>> >> Em 13 de março de 2018 20:19, Douglas Oliveira de Lima
>> >>  escreveu:
>> >> > Essa achei legal e estou postando.
>> >> >
>> >> > Resolva nos inteiros a seguinte equação:  (x + y)(y + z)(z + x)/2 +
>> >> > (x +
>> >> > y +
>> >> > z)3 = 1 – xyz .
>> >> >
>> >>
>> >> Substituição mágica: x=-a+b+c, y=a-b+c, z=a+b-c. Com isso, x+y=2c,
>> >> x+y+z=a+b+c e
>> >>
>> >> 4abc + (a+b+c)^3 + (-a+b+c)(a-b+c)(a+b-c) = 1
>> >>
>> >> Usando polinômios simétricos,
>> >>
>> >> 4(a+b+c)(ab+ac+bc) - 4abc = 1
>> >>
>> >> Agora estou confuso...
>> >>
>> >> > Abraço do
>> >> > Douglas Oliveira
>> >> >
>> >> > --
>> >> > Esta mensagem foi verificada pelo sistema de antivírus e
>> >> > acredita-se estar livre de perigo.
>> >>
>> >> --
>> >> Esta mensagem foi 

[obm-l] Re: [obm-l] Re: [obm-l] Re: [obm-l] Teoria dos números

2018-03-23 Por tôpico Claudio Buffara
Pois do jeito que você propôs, alguém poderia pensar que se trata de provar
que (s-1)(t-1)(u-1) divide stu - 1 para todos os inteiros s, t, u com 1 < s
< t < u, o que certamente não é verdade.


2018-03-23 16:55 GMT-03:00 Claudio Buffara :

> Sim. Eu só quis ter certeza de que o problema era:
> achar todos os inteiros s, t, u com 1 < s < t < u tais que (s-1)(t-1)(u-1)
> divide stu - 1
>
> 2018-03-23 16:45 GMT-03:00 Pedro José :
>
>> Boa tarde!
>> Seria bom desconsiderar o problema aqui, pois já tem um caminhão de
>> notas. Criei uma mensagem nova, não sei porque foi parar aqui, não sei se
>> pelo assunto ter o mesmo nome.
>> Alguém postá-lo independente dessa leva.
>> Cláudio, o que você propôs, não tem solução. Não creio que ajude.
>> Não compreendi a sua primeira pergunta, um esclarecimento o síbolo "|"
>> não é tal que é divide, ou seja (s-1)(t-1) divide st - 1,
>>
>> Saudações,
>> PJMS
>>
>> Em 23 de março de 2018 15:49, Claudio Buffara 
>> escreveu:
>>
>>> Será que ajuda começar com um mais simples?
>>>
>>> Achar s, t tais que (s-1)(t-1) | st - 1, com 1 < s < t.
>>>
>>> 2018-03-23 15:38 GMT-03:00 Pedro José :
>>>
 Boa tarde!

 Aproveitando que deu o que falar o problema postado pelo Douglas, tem
 um que achei mais interessante.

 (s-1)(t-1).(u-1) | stu -1, com s, t, u inteiros  e 1 >>
 Saudações,
 Pedro

 --
 Esta mensagem foi verificada pelo sistema de antivírus e
 acredita-se estar livre de perigo.
>>>
>>>
>>>
>>> --
>>> Esta mensagem foi verificada pelo sistema de antivírus e
>>> acredita-se estar livre de perigo.
>>>
>>
>>
>> --
>> Esta mensagem foi verificada pelo sistema de antivírus e
>> acredita-se estar livre de perigo.
>>
>
>

-- 
Esta mensagem foi verificada pelo sistema de antiv�rus e
 acredita-se estar livre de perigo.



[obm-l] Re: [obm-l] Re: [obm-l] Re: [obm-l] Teoria dos números

2018-03-23 Por tôpico Claudio Buffara
Sim. Eu só quis ter certeza de que o problema era:
achar todos os inteiros s, t, u com 1 < s < t < u tais que (s-1)(t-1)(u-1)
divide stu - 1

2018-03-23 16:45 GMT-03:00 Pedro José :

> Boa tarde!
> Seria bom desconsiderar o problema aqui, pois já tem um caminhão de notas.
> Criei uma mensagem nova, não sei porque foi parar aqui, não sei se pelo
> assunto ter o mesmo nome.
> Alguém postá-lo independente dessa leva.
> Cláudio, o que você propôs, não tem solução. Não creio que ajude.
> Não compreendi a sua primeira pergunta, um esclarecimento o síbolo "|" não
> é tal que é divide, ou seja (s-1)(t-1) divide st - 1,
>
> Saudações,
> PJMS
>
> Em 23 de março de 2018 15:49, Claudio Buffara 
> escreveu:
>
>> Será que ajuda começar com um mais simples?
>>
>> Achar s, t tais que (s-1)(t-1) | st - 1, com 1 < s < t.
>>
>> 2018-03-23 15:38 GMT-03:00 Pedro José :
>>
>>> Boa tarde!
>>>
>>> Aproveitando que deu o que falar o problema postado pelo Douglas, tem um
>>> que achei mais interessante.
>>>
>>> (s-1)(t-1).(u-1) | stu -1, com s, t, u inteiros  e 1 >
>>> Saudações,
>>> Pedro
>>>
>>> --
>>> Esta mensagem foi verificada pelo sistema de antivírus e
>>> acredita-se estar livre de perigo.
>>
>>
>>
>> --
>> Esta mensagem foi verificada pelo sistema de antivírus e
>> acredita-se estar livre de perigo.
>>
>
>
> --
> Esta mensagem foi verificada pelo sistema de antivírus e
> acredita-se estar livre de perigo.
>

-- 
Esta mensagem foi verificada pelo sistema de antiv�rus e
 acredita-se estar livre de perigo.



[obm-l] Re: [obm-l] Re: [obm-l] Re: [obm-l] Re: [obm-l] Re: [obm-l] Teoria dos números

2018-03-23 Por tôpico Claudio Buffara
Você não havia explicado que* "fui fazer um experimento tirando o "1" da
equação. Usei um par (x,y) com a mesma paridade e achei um z inteiro.
Novamente usei outro par e deu outro z inteiro. Olhando para os
experimentos. Vi que nos  dois casos z = -(x+y)/2. Ai tornou-se uma
conjectura."*

Ou seja, experimentos numéricos levaram à conjectura. Beleza!

De toda forma, fazer experimentos tirando o "1" da equação foi uma bela
sacada. Você primeiro examinou a versão "homogênea". Boa ideia.

[]s,
Claudio.




2018-03-23 14:30 GMT-03:00 Pedro José :

> Boa tarde!
>
> Cláudio,
> desculpe-me discordar, mas eu disse  de onde veio. Só não veio de nenhuma
> técnica.
> Estava vendo que a parcela do problema: (x+y) (x+z) (y+z)/2 sempre seria
> inteira pois dois desses valores teriam paridade iguais.
> Aí fui fazer um experimento tirando o "1" da equação. Usei um par (x,y)
> com a mesma paridade e achei um z inteiro. Novamente usei outro par e deu
> outro z inteiro. Olhando para os experimentos. Vi que nos  dois casos z =
> -(x+y)/2. Ai tornou-se uma conjectura. Pura sorte.
> Mas, no braço, desenvolvi para para [x,y, -(x+y)/2] e atendeu para essa
> família, aqui deixou de ser conjectura. Foi provado, na grosseria, por
> substituição mas foi.
>
> Saudações,
> PJMS
>
> Em 23 de março de 2018 11:07, Pedro José  escreveu:
>
>> Bom dia!
>> Anderson,
>> o Gugu já avançou, em uma nota acima. E é passível.
>> Revendo a solução do Ralph, fica claro que essa transformação seria de
>> valia.
>> Pois essa transformação leva a :
>> a = (y+z)/2
>> b=  (x+z)/2
>> c= (x+y)/2
>>
>> Então na ordem que o Ralph apresentou: 1/2*(2x+y+z)(x+2y+z)(x+y+2z)=1
>>
>> (b+c) dá a metade do primeiro fator (excetuando-se o fator 1/2) , (a+c) a
>> metade do segundo e (a+b) a metade do terceiro.
>>
>> Saudações,
>>
>>
>>
>> Em 23 de março de 2018 06:20, Anderson Torres <
>> torres.anderson...@gmail.com> escreveu:
>>
>>> Em 21 de março de 2018 09:47, Claudio Buffara
>>>  escreveu:
>>> > Como você passou de:
>>> > 4abc + (a+b+c)^3 + (-a+b+c)(a-b+c)(a+b-c) = 1
>>> >
>>> > Para:
>>> > 4(a+b+c)(ab+ac+bc) - 4abc = 1
>>>
>>> It's kind of magic. Eu simplesmente abri tudo com vontade e notei
>>> certas repetições
>>> que sempre aparecem em certas fatorações; ou melhor dizendo, estava
>>> pensando em
>>> escrever tudo em termos dos famigerados polinômios simétricos e cheguei
>>> nisso.
>>>
>>> Sempre que vejo algo como (a^2b+ab^2), já escrevo ab(a+b) e tento
>>> procurar um abc
>>> para isso resultar em ab(a+b+c).
>>>
>>> Mas não avancei daí. Penso que dá para fatorar ainda mais...
>>>
>>> >
>>> > ???
>>> >
>>> > []s,
>>> > Claudio.
>>> >
>>> >
>>> > 2018-03-20 23:14 GMT-03:00 Anderson Torres <
>>> torres.anderson...@gmail.com>:
>>> >>
>>> >> Em 13 de março de 2018 20:19, Douglas Oliveira de Lima
>>> >>  escreveu:
>>> >> > Essa achei legal e estou postando.
>>> >> >
>>> >> > Resolva nos inteiros a seguinte equação:  (x + y)(y + z)(z + x)/2 +
>>> (x +
>>> >> > y +
>>> >> > z)3 = 1 – xyz .
>>> >> >
>>> >>
>>> >> Substituição mágica: x=-a+b+c, y=a-b+c, z=a+b-c. Com isso, x+y=2c,
>>> >> x+y+z=a+b+c e
>>> >>
>>> >> 4abc + (a+b+c)^3 + (-a+b+c)(a-b+c)(a+b-c) = 1
>>> >>
>>> >> Usando polinômios simétricos,
>>> >>
>>> >> 4(a+b+c)(ab+ac+bc) - 4abc = 1
>>> >>
>>> >> Agora estou confuso...
>>> >>
>>> >> > Abraço do
>>> >> > Douglas Oliveira
>>> >> >
>>> >> > --
>>> >> > Esta mensagem foi verificada pelo sistema de antivírus e
>>> >> > acredita-se estar livre de perigo.
>>> >>
>>> >> --
>>> >> Esta mensagem foi verificada pelo sistema de antivírus e
>>> >>  acredita-se estar livre de perigo.
>>> >>
>>> >>
>>> >> 
>>> =
>>> >> Instru�ões para entrar na lista, sair da lista e usar a lista em
>>> >> http://www.mat.puc-rio.br/~obmlistas/obm-l.html
>>> >> 
>>> =
>>> >
>>> >
>>> >
>>> > --
>>> > Esta mensagem foi verificada pelo sistema de antivírus e
>>> > acredita-se estar livre de perigo.
>>>
>>> --
>>> Esta mensagem foi verificada pelo sistema de antivírus e
>>>  acredita-se estar livre de perigo.
>>>
>>>
>>> 
>>> =
>>> Instru�ões para entrar na lista, sair da lista e usar a lista em
>>> http://www.mat.puc-rio.br/~obmlistas/obm-l.html
>>> 
>>> =
>>>
>>
>>
>
> --
> Esta mensagem foi verificada pelo sistema de antivírus e
> acredita-se estar livre de perigo.
>

-- 
Esta mensagem foi verificada pelo sistema de antiv�rus e
 acredita-se estar livre de perigo.



[obm-l] Re: [obm-l] Re: [obm-l] Re: [obm-l] Re: [obm-l] Teoria dos números

2018-03-23 Por tôpico Pedro José
Boa tarde!

Cláudio,
desculpe-me discordar, mas eu disse  de onde veio. Só não veio de nenhuma
técnica.
Estava vendo que a parcela do problema: (x+y) (x+z) (y+z)/2 sempre seria
inteira pois dois desses valores teriam paridade iguais.
Aí fui fazer um experimento tirando o "1" da equação. Usei um par (x,y) com
a mesma paridade e achei um z inteiro. Novamente usei outro par e deu outro
z inteiro. Olhando para os experimentos. Vi que nos  dois casos z =
-(x+y)/2. Ai tornou-se uma conjectura. Pura sorte.
Mas, no braço, desenvolvi para para [x,y, -(x+y)/2] e atendeu para essa
família, aqui deixou de ser conjectura. Foi provado, na grosseria, por
substituição mas foi.

Saudações,
PJMS

Em 23 de março de 2018 11:07, Pedro José  escreveu:

> Bom dia!
> Anderson,
> o Gugu já avançou, em uma nota acima. E é passível.
> Revendo a solução do Ralph, fica claro que essa transformação seria de
> valia.
> Pois essa transformação leva a :
> a = (y+z)/2
> b=  (x+z)/2
> c= (x+y)/2
>
> Então na ordem que o Ralph apresentou: 1/2*(2x+y+z)(x+2y+z)(x+y+2z)=1
>
> (b+c) dá a metade do primeiro fator (excetuando-se o fator 1/2) , (a+c) a
> metade do segundo e (a+b) a metade do terceiro.
>
> Saudações,
>
>
>
> Em 23 de março de 2018 06:20, Anderson Torres <
> torres.anderson...@gmail.com> escreveu:
>
>> Em 21 de março de 2018 09:47, Claudio Buffara
>>  escreveu:
>> > Como você passou de:
>> > 4abc + (a+b+c)^3 + (-a+b+c)(a-b+c)(a+b-c) = 1
>> >
>> > Para:
>> > 4(a+b+c)(ab+ac+bc) - 4abc = 1
>>
>> It's kind of magic. Eu simplesmente abri tudo com vontade e notei
>> certas repetições
>> que sempre aparecem em certas fatorações; ou melhor dizendo, estava
>> pensando em
>> escrever tudo em termos dos famigerados polinômios simétricos e cheguei
>> nisso.
>>
>> Sempre que vejo algo como (a^2b+ab^2), já escrevo ab(a+b) e tento
>> procurar um abc
>> para isso resultar em ab(a+b+c).
>>
>> Mas não avancei daí. Penso que dá para fatorar ainda mais...
>>
>> >
>> > ???
>> >
>> > []s,
>> > Claudio.
>> >
>> >
>> > 2018-03-20 23:14 GMT-03:00 Anderson Torres <
>> torres.anderson...@gmail.com>:
>> >>
>> >> Em 13 de março de 2018 20:19, Douglas Oliveira de Lima
>> >>  escreveu:
>> >> > Essa achei legal e estou postando.
>> >> >
>> >> > Resolva nos inteiros a seguinte equação:  (x + y)(y + z)(z + x)/2 +
>> (x +
>> >> > y +
>> >> > z)3 = 1 – xyz .
>> >> >
>> >>
>> >> Substituição mágica: x=-a+b+c, y=a-b+c, z=a+b-c. Com isso, x+y=2c,
>> >> x+y+z=a+b+c e
>> >>
>> >> 4abc + (a+b+c)^3 + (-a+b+c)(a-b+c)(a+b-c) = 1
>> >>
>> >> Usando polinômios simétricos,
>> >>
>> >> 4(a+b+c)(ab+ac+bc) - 4abc = 1
>> >>
>> >> Agora estou confuso...
>> >>
>> >> > Abraço do
>> >> > Douglas Oliveira
>> >> >
>> >> > --
>> >> > Esta mensagem foi verificada pelo sistema de antivírus e
>> >> > acredita-se estar livre de perigo.
>> >>
>> >> --
>> >> Esta mensagem foi verificada pelo sistema de antivírus e
>> >>  acredita-se estar livre de perigo.
>> >>
>> >>
>> >> 
>> =
>> >> Instru�ões para entrar na lista, sair da lista e usar a lista em
>> >> http://www.mat.puc-rio.br/~obmlistas/obm-l.html
>> >> 
>> =
>> >
>> >
>> >
>> > --
>> > Esta mensagem foi verificada pelo sistema de antivírus e
>> > acredita-se estar livre de perigo.
>>
>> --
>> Esta mensagem foi verificada pelo sistema de antivírus e
>>  acredita-se estar livre de perigo.
>>
>>
>> =
>> Instru�ões para entrar na lista, sair da lista e usar a lista em
>> http://www.mat.puc-rio.br/~obmlistas/obm-l.html
>> =
>>
>
>

-- 
Esta mensagem foi verificada pelo sistema de antiv�rus e
 acredita-se estar livre de perigo.



[obm-l] Re: [obm-l] Re: [obm-l] Re: [obm-l] Re: [obm-l] Teoria dos números

2018-03-23 Por tôpico Pedro José
Bom dia!
Anderson,
o Gugu já avançou, em uma nota acima. E é passível.
Revendo a solução do Ralph, fica claro que essa transformação seria de
valia.
Pois essa transformação leva a :
a = (y+z)/2
b=  (x+z)/2
c= (x+y)/2

Então na ordem que o Ralph apresentou: 1/2*(2x+y+z)(x+2y+z)(x+y+2z)=1

(b+c) dá a metade do primeiro fator (excetuando-se o fator 1/2) , (a+c) a
metade do segundo e (a+b) a metade do terceiro.

Saudações,



Em 23 de março de 2018 06:20, Anderson Torres 
escreveu:

> Em 21 de março de 2018 09:47, Claudio Buffara
>  escreveu:
> > Como você passou de:
> > 4abc + (a+b+c)^3 + (-a+b+c)(a-b+c)(a+b-c) = 1
> >
> > Para:
> > 4(a+b+c)(ab+ac+bc) - 4abc = 1
>
> It's kind of magic. Eu simplesmente abri tudo com vontade e notei
> certas repetições
> que sempre aparecem em certas fatorações; ou melhor dizendo, estava
> pensando em
> escrever tudo em termos dos famigerados polinômios simétricos e cheguei
> nisso.
>
> Sempre que vejo algo como (a^2b+ab^2), já escrevo ab(a+b) e tento
> procurar um abc
> para isso resultar em ab(a+b+c).
>
> Mas não avancei daí. Penso que dá para fatorar ainda mais...
>
> >
> > ???
> >
> > []s,
> > Claudio.
> >
> >
> > 2018-03-20 23:14 GMT-03:00 Anderson Torres  >:
> >>
> >> Em 13 de março de 2018 20:19, Douglas Oliveira de Lima
> >>  escreveu:
> >> > Essa achei legal e estou postando.
> >> >
> >> > Resolva nos inteiros a seguinte equação:  (x + y)(y + z)(z + x)/2 +
> (x +
> >> > y +
> >> > z)3 = 1 – xyz .
> >> >
> >>
> >> Substituição mágica: x=-a+b+c, y=a-b+c, z=a+b-c. Com isso, x+y=2c,
> >> x+y+z=a+b+c e
> >>
> >> 4abc + (a+b+c)^3 + (-a+b+c)(a-b+c)(a+b-c) = 1
> >>
> >> Usando polinômios simétricos,
> >>
> >> 4(a+b+c)(ab+ac+bc) - 4abc = 1
> >>
> >> Agora estou confuso...
> >>
> >> > Abraço do
> >> > Douglas Oliveira
> >> >
> >> > --
> >> > Esta mensagem foi verificada pelo sistema de antivírus e
> >> > acredita-se estar livre de perigo.
> >>
> >> --
> >> Esta mensagem foi verificada pelo sistema de antivírus e
> >>  acredita-se estar livre de perigo.
> >>
> >>
> >> 
> =
> >> Instru�ões para entrar na lista, sair da lista e usar a lista em
> >> http://www.mat.puc-rio.br/~obmlistas/obm-l.html
> >> 
> =
> >
> >
> >
> > --
> > Esta mensagem foi verificada pelo sistema de antivírus e
> > acredita-se estar livre de perigo.
>
> --
> Esta mensagem foi verificada pelo sistema de antivírus e
>  acredita-se estar livre de perigo.
>
>
> =
> Instru�ões para entrar na lista, sair da lista e usar a lista em
> http://www.mat.puc-rio.br/~obmlistas/obm-l.html
> =
>

-- 
Esta mensagem foi verificada pelo sistema de antiv�rus e
 acredita-se estar livre de perigo.



[obm-l] Re: [obm-l] Re: [obm-l] Re: [obm-l] Re: [obm-l] Teoria dos números

2018-03-23 Por tôpico Claudio Buffara
Na verdade os meus questionamentos surgiram por causa do meu interesse em
ensino de matemática.

Por exemplo, produtos notáveis e fatorações são notoriamente mal ensinados,
pelo menos nos livros didáticos de 8o e 9o ano que eu examinei.
Nenhum menciona que:
a) as generalizações de (x+y)^2 = x^2 + 2xy + y^2  e  x^2 - y^2 =
(x-y)(x+y) para expoentes maiores levam ao teorema do binômio (erroneamente
chamado de binômio de Newton - nota histórica: Newton generalizou o teorema
para expoentes racionais) e à fórmula da soma dos termos de uma PG;
b) (x+y)^2 = x^2 + 2xy + y^2 é a base para a ideia de se completar
quadrados, a qual, por sua vez, não só resulta na fórmula para as raízes de
uma equação quadrática, mas também na elucidação das propriedades da função
quadrática;
c) o uso inteligente da expansão de (x+y)^3 leva à formula das raízes de
uma equação cúbica.

***

Há tempos, o Hermann, participante desta lista, postou uma dúvida sobre
produtos notáveis e pediu dicas de livros com exercícios sobre produtos
notáveis e fatoração.
Eu tenho duas sugestões, ambas em inglês:
- Algebra, de I.M.Gelfand e A.Shen - Birkhäuser (este faz as generalizações
que eu mencionei acima)
- A Problem Book in Algebra, de V.A. Krechmar - Mir Publishers (pros
entusiastas)
Ambos estão disponíveis na Amazon.

***

Anos atrás eu gostava de soluções "mágicas", obtidas por meio de alguma
sacada brilhante que eu jamais conseguiria ter.
Após me deparar com várias destas soluções, me ocorreu que elas talvez
tivessem um efeito perverso na motivação dos estudantes de matemática, pois
passavam a impressão de que é preciso ser um gênio para dominar a matéria.
Daí o meu interesse em saber como vocês obtiveram certas fatorações.
Entendo que trabalho braçal, experiência, alguma lógica e um pouco de
otimismo são, para a maioria de nós, as únicas formas de progredir na
resolução de um problema como o que deu origem a este thread.

Dito isso (e posso estar enganado) nem o Pedro José e nem mais ninguém
explicou de onde veio a conjectura (correta) de que:
z = -(x+y)/2 é solução de *(x + y)(y + z)(z + x)/2 + (x + y + z)3 =  – xyz*

[]s,
Claudio.



2018-03-23 6:20 GMT-03:00 Anderson Torres :

> Em 21 de março de 2018 09:47, Claudio Buffara
>  escreveu:
> > Como você passou de:
> > 4abc + (a+b+c)^3 + (-a+b+c)(a-b+c)(a+b-c) = 1
> >
> > Para:
> > 4(a+b+c)(ab+ac+bc) - 4abc = 1
>
> It's kind of magic. Eu simplesmente abri tudo com vontade e notei
> certas repetições
> que sempre aparecem em certas fatorações; ou melhor dizendo, estava
> pensando em
> escrever tudo em termos dos famigerados polinômios simétricos e cheguei
> nisso.
>
> Sempre que vejo algo como (a^2b+ab^2), já escrevo ab(a+b) e tento
> procurar um abc
> para isso resultar em ab(a+b+c).
>
> Mas não avancei daí. Penso que dá para fatorar ainda mais...
>
> >
> > ???
> >
> > []s,
> > Claudio.
> >
> >
> > 2018-03-20 23:14 GMT-03:00 Anderson Torres  >:
> >>
> >> Em 13 de março de 2018 20:19, Douglas Oliveira de Lima
> >>  escreveu:
> >> > Essa achei legal e estou postando.
> >> >
> >> > Resolva nos inteiros a seguinte equação:  (x + y)(y + z)(z + x)/2 +
> (x +
> >> > y +
> >> > z)3 = 1 – xyz .
> >> >
> >>
> >> Substituição mágica: x=-a+b+c, y=a-b+c, z=a+b-c. Com isso, x+y=2c,
> >> x+y+z=a+b+c e
> >>
> >> 4abc + (a+b+c)^3 + (-a+b+c)(a-b+c)(a+b-c) = 1
> >>
> >> Usando polinômios simétricos,
> >>
> >> 4(a+b+c)(ab+ac+bc) - 4abc = 1
> >>
> >> Agora estou confuso...
> >>
> >> > Abraço do
> >> > Douglas Oliveira
> >> >
> >> > --
> >> > Esta mensagem foi verificada pelo sistema de antivírus e
> >> > acredita-se estar livre de perigo.
> >>
> >> --
> >> Esta mensagem foi verificada pelo sistema de antivírus e
> >>  acredita-se estar livre de perigo.
> >>
> >>
> >> 
> =
> >> Instru�ões para entrar na lista, sair da lista e usar a lista em
> >> http://www.mat.puc-rio.br/~obmlistas/obm-l.html
> >> 
> =
> >
> >
> >
> > --
> > Esta mensagem foi verificada pelo sistema de antivírus e
> > acredita-se estar livre de perigo.
>
> --
> Esta mensagem foi verificada pelo sistema de antivírus e
>  acredita-se estar livre de perigo.
>
>
> =
> Instru�ões para entrar na lista, sair da lista e usar a lista em
> http://www.mat.puc-rio.br/~obmlistas/obm-l.html
> =
>

-- 
Esta mensagem foi verificada pelo sistema de antiv�rus e
 acredita-se estar livre de perigo.



[obm-l] Re: [obm-l] Re: [obm-l] Re: [obm-l] Teoria dos números

2018-03-23 Por tôpico Anderson Torres
Em 21 de março de 2018 09:47, Claudio Buffara
 escreveu:
> Como você passou de:
> 4abc + (a+b+c)^3 + (-a+b+c)(a-b+c)(a+b-c) = 1
>
> Para:
> 4(a+b+c)(ab+ac+bc) - 4abc = 1

It's kind of magic. Eu simplesmente abri tudo com vontade e notei
certas repetições
que sempre aparecem em certas fatorações; ou melhor dizendo, estava pensando em
escrever tudo em termos dos famigerados polinômios simétricos e cheguei nisso.

Sempre que vejo algo como (a^2b+ab^2), já escrevo ab(a+b) e tento
procurar um abc
para isso resultar em ab(a+b+c).

Mas não avancei daí. Penso que dá para fatorar ainda mais...

>
> ???
>
> []s,
> Claudio.
>
>
> 2018-03-20 23:14 GMT-03:00 Anderson Torres :
>>
>> Em 13 de março de 2018 20:19, Douglas Oliveira de Lima
>>  escreveu:
>> > Essa achei legal e estou postando.
>> >
>> > Resolva nos inteiros a seguinte equação:  (x + y)(y + z)(z + x)/2 + (x +
>> > y +
>> > z)3 = 1 – xyz .
>> >
>>
>> Substituição mágica: x=-a+b+c, y=a-b+c, z=a+b-c. Com isso, x+y=2c,
>> x+y+z=a+b+c e
>>
>> 4abc + (a+b+c)^3 + (-a+b+c)(a-b+c)(a+b-c) = 1
>>
>> Usando polinômios simétricos,
>>
>> 4(a+b+c)(ab+ac+bc) - 4abc = 1
>>
>> Agora estou confuso...
>>
>> > Abraço do
>> > Douglas Oliveira
>> >
>> > --
>> > Esta mensagem foi verificada pelo sistema de antivírus e
>> > acredita-se estar livre de perigo.
>>
>> --
>> Esta mensagem foi verificada pelo sistema de antivírus e
>>  acredita-se estar livre de perigo.
>>
>>
>> =
>> Instru�ões para entrar na lista, sair da lista e usar a lista em
>> http://www.mat.puc-rio.br/~obmlistas/obm-l.html
>> =
>
>
>
> --
> Esta mensagem foi verificada pelo sistema de antivírus e
> acredita-se estar livre de perigo.

-- 
Esta mensagem foi verificada pelo sistema de antiv�rus e
 acredita-se estar livre de perigo.


=
Instru��es para entrar na lista, sair da lista e usar a lista em
http://www.mat.puc-rio.br/~obmlistas/obm-l.html
=


[obm-l] Re: [obm-l] Re: [obm-l] Re: [obm-l] Re: [obm-l] Teoria dos números

2018-03-22 Por tôpico Pedro José
Boa noite!
Nem havia reparado que a transformação do Gugu, foi feita em composição com
a anterior que fora postada. Acabou sendo a mesma que postei.
Escolhi porque fazia sumir os termos com expoente 3.

Saudações,
PJMS

Em 22 de mar de 2018 22:59, "Pedro José"  escreveu:

> Boa noite!
> Vi duas proposições de substituições de variáveis, nas notas anteriores e
> ratifico os questionamentos do Cláudio.
> Aventurei uma substituição:
> a=x+y ; b=x+z; c = y + z.
> Aí, na munheca cancelam-se os termos com expoentes cúbicos. E separando os
> termos de
>  (a+b)*(a+c), no que sobra, chega- se facilmente a:
> 1/2*(a+b)*(a+c)*(b+c) = 1 ou
> (a+b)*(a+c)*(b+c) = 2.
> Para x, y e z inteiros só atendem soluções em a, b e c com dois ímpares e
> um par ou três pares.
> Então, só atendem a= 0, b=1 e c= 1 ou a=-2, b=1 e c= 1. E suas
> permutações. Que trazendo para as variáveis originais dá o mesmo resultado
> já encontrado.
> Mas minha dúvida é: para esses casos de polinômios simétricos há alguma
> técnica para escolha de variáveis e para fatorar, como é.g. , na mudança de
> variáveis das cônicas para retirada do termo xy, possibilitando a
> identificação da cônica?
> Saudações,
> PJMS
>
> Em 22 de mar de 2018 7:56 AM, "Claudio Buffara" 
> escreveu:
>
>> Tudo muito bom, mas o que ninguém explicou é como foram obtidas as
>> fatorações/transformações algébricas mágicas.
>> Insight?
>> Conhecimentos prévios?
>> Tentativa e erro e muito braço?
>>
>> []s,
>> Claudio.
>>
>>
>> 2018-03-21 18:54 GMT-03:00 :
>>
>>> Sim, e fazendo a=u/2, b=v/2 e c=w/2 temos (u+v+w)(uv+uw+vw)-uvw=2, ou
>>> seja, u^2v+uv^2+u^2w+uw^2+v^2w+vw^2+2uvw=2, mas
>>> u^2v+uv^2+u^2w+uw^2+v^2w+vw^2+2uvw=(u+v)(u+w)(v+w). Assim, podemos ter
>>> u+v=2, u+w=v+w=1, o que dá w=0, u=v=1; u+v=2, u+w=v+w=-1, o que dá w=-2,
>>> u=v=1; u+v=-2, u+w=1, v+w=-1, o que dá w=1, u=0, v=-2; u+v=-2, u+w=-1,
>>> v+w=1, o que dá w=1, u=-2, v=0 e as coisas simétricas. Daí saem as soluções
>>> (1,0,0), (2,-1,-1), (-3/2,-1/2,3/2), (-3/2,3/2,-1/2) (e as coisas
>>> simétricas), mas as duas últimas não são inteiras, de modo que só temos as
>>> soluções que já tínhamos achado...
>>>Abraços,
>>>  Gugu
>>>
>>> Quoting Bernardo Freitas Paulo da Costa :
>>>
>>> 2018-03-20 23:14 GMT-03:00 Anderson Torres :

> Em 13 de março de 2018 20:19, Douglas Oliveira de Lima
>  escreveu:
>
>> Essa achei legal e estou postando.
>>
>> Resolva nos inteiros a seguinte equação:  (x + y)(y + z)(z + x)/2 +
>> (x + y +
>> z)3 = 1 – xyz .
>>
>>
> Substituição mágica: x=-a+b+c, y=a-b+c, z=a+b-c. Com isso, x+y=2c,
> x+y+z=a+b+c e
>
> 4abc + (a+b+c)^3 + (-a+b+c)(a-b+c)(a+b-c) = 1
>
> Usando polinômios simétricos,
>
> 4(a+b+c)(ab+ac+bc) - 4abc = 1
>
> Agora estou confuso...
>

 Note que a,b,c não precisam mais ser inteiros, podem ser inteiros
 divididos por 2 (se não me engano)

 Abraços,
 --
 Bernardo Freitas Paulo da Costa

 --
 Esta mensagem foi verificada pelo sistema de antiv?rus e
  acredita-se estar livre de perigo.


 
 =
 Instru??es para entrar na lista, sair da lista e usar a lista em
 http://www.mat.puc-rio.br/~obmlistas/obm-l.html
 
 =

>>>
>>>
>>>
>>>
>>> --
>>> Esta mensagem foi verificada pelo sistema de antivírus e
>>> acredita-se estar livre de perigo.
>>>
>>> 
>>> =
>>> Instru�ões para entrar na lista, sair da lista e usar a lista em
>>> http://www.mat.puc-rio.br/~obmlistas/obm-l.html
>>> 
>>> =
>>>
>>
>>
>> --
>> Esta mensagem foi verificada pelo sistema de antivírus e
>> acredita-se estar livre de perigo.
>
>

-- 
Esta mensagem foi verificada pelo sistema de antiv�rus e
 acredita-se estar livre de perigo.



Re: [obm-l] Re: [obm-l] Re: [obm-l] Re: [obm-l] Teoria dos números

2018-03-22 Por tôpico gugu

   Oi Claudio,
   Eu não sei de onde veio a substituição mágica do Anderson Torres -  
só achei uma fatoração na expressão obtida a partir dela... Não sou  
especialmente fã desse tipo de problema.

   Abraços,
 Gugu

Quoting Claudio Buffara :


Tudo muito bom, mas o que ninguém explicou é como foram obtidas as
fatorações/transformações algébricas mágicas.
Insight?
Conhecimentos prévios?
Tentativa e erro e muito braço?

[]s,
Claudio.


2018-03-21 18:54 GMT-03:00 :


Sim, e fazendo a=u/2, b=v/2 e c=w/2 temos (u+v+w)(uv+uw+vw)-uvw=2, ou
seja, u^2v+uv^2+u^2w+uw^2+v^2w+vw^2+2uvw=2, mas
u^2v+uv^2+u^2w+uw^2+v^2w+vw^2+2uvw=(u+v)(u+w)(v+w). Assim, podemos ter
u+v=2, u+w=v+w=1, o que dá w=0, u=v=1; u+v=2, u+w=v+w=-1, o que dá w=-2,
u=v=1; u+v=-2, u+w=1, v+w=-1, o que dá w=1, u=0, v=-2; u+v=-2, u+w=-1,
v+w=1, o que dá w=1, u=-2, v=0 e as coisas simétricas. Daí saem as soluções
(1,0,0), (2,-1,-1), (-3/2,-1/2,3/2), (-3/2,3/2,-1/2) (e as coisas
simétricas), mas as duas últimas não são inteiras, de modo que só temos as
soluções que já tínhamos achado...
   Abraços,
 Gugu

Quoting Bernardo Freitas Paulo da Costa :

2018-03-20 23:14 GMT-03:00 Anderson Torres :



Em 13 de março de 2018 20:19, Douglas Oliveira de Lima
 escreveu:


Essa achei legal e estou postando.

Resolva nos inteiros a seguinte equação:  (x + y)(y + z)(z + x)/2 + (x
+ y +
z)3 = 1 – xyz .



Substituição mágica: x=-a+b+c, y=a-b+c, z=a+b-c. Com isso, x+y=2c,
x+y+z=a+b+c e

4abc + (a+b+c)^3 + (-a+b+c)(a-b+c)(a+b-c) = 1

Usando polinômios simétricos,

4(a+b+c)(ab+ac+bc) - 4abc = 1

Agora estou confuso...



Note que a,b,c não precisam mais ser inteiros, podem ser inteiros
divididos por 2 (se não me engano)

Abraços,
--
Bernardo Freitas Paulo da Costa

--
Esta mensagem foi verificada pelo sistema de antiv?rus e
 acredita-se estar livre de perigo.


=
Instru??es para entrar na lista, sair da lista e usar a lista em
http://www.mat.puc-rio.br/~obmlistas/obm-l.html
=






--
Esta mensagem foi verificada pelo sistema de antivírus e
acredita-se estar livre de perigo.

=
Instru�ões para entrar na lista, sair da lista e usar a lista em
http://www.mat.puc-rio.br/~obmlistas/obm-l.html
=



--
Esta mensagem foi verificada pelo sistema de antiv?rus e
 acredita-se estar livre de perigo.





--
Esta mensagem foi verificada pelo sistema de antiv�rus e
acredita-se estar livre de perigo.

=
Instru��es para entrar na lista, sair da lista e usar a lista em
http://www.mat.puc-rio.br/~obmlistas/obm-l.html
=


[obm-l] Re: [obm-l] Re: [obm-l] Re: [obm-l] Re: [obm-l] Teoria dos números

2018-03-22 Por tôpico Pedro José
Boa noite!
Vi duas proposições de substituições de variáveis, nas notas anteriores e
ratifico os questionamentos do Cláudio.
Aventurei uma substituição:
a=x+y ; b=x+z; c = y + z.
Aí, na munheca cancelam-se os termos com expoentes cúbicos. E separando os
termos de
 (a+b)*(a+c), no que sobra, chega- se facilmente a:
1/2*(a+b)*(a+c)*(b+c) = 1 ou
(a+b)*(a+c)*(b+c) = 2.
Para x, y e z inteiros só atendem soluções em a, b e c com dois ímpares e
um par ou três pares.
Então, só atendem a= 0, b=1 e c= 1 ou a=-2, b=1 e c= 1. E suas permutações.
Que trazendo para as variáveis originais dá o mesmo resultado já encontrado.
Mas minha dúvida é: para esses casos de polinômios simétricos há alguma
técnica para escolha de variáveis e para fatorar, como é.g. , na mudança de
variáveis das cônicas para retirada do termo xy, possibilitando a
identificação da cônica?
Saudações,
PJMS

Em 22 de mar de 2018 7:56 AM, "Claudio Buffara" 
escreveu:

> Tudo muito bom, mas o que ninguém explicou é como foram obtidas as
> fatorações/transformações algébricas mágicas.
> Insight?
> Conhecimentos prévios?
> Tentativa e erro e muito braço?
>
> []s,
> Claudio.
>
>
> 2018-03-21 18:54 GMT-03:00 :
>
>> Sim, e fazendo a=u/2, b=v/2 e c=w/2 temos (u+v+w)(uv+uw+vw)-uvw=2, ou
>> seja, u^2v+uv^2+u^2w+uw^2+v^2w+vw^2+2uvw=2, mas
>> u^2v+uv^2+u^2w+uw^2+v^2w+vw^2+2uvw=(u+v)(u+w)(v+w). Assim, podemos ter
>> u+v=2, u+w=v+w=1, o que dá w=0, u=v=1; u+v=2, u+w=v+w=-1, o que dá w=-2,
>> u=v=1; u+v=-2, u+w=1, v+w=-1, o que dá w=1, u=0, v=-2; u+v=-2, u+w=-1,
>> v+w=1, o que dá w=1, u=-2, v=0 e as coisas simétricas. Daí saem as soluções
>> (1,0,0), (2,-1,-1), (-3/2,-1/2,3/2), (-3/2,3/2,-1/2) (e as coisas
>> simétricas), mas as duas últimas não são inteiras, de modo que só temos as
>> soluções que já tínhamos achado...
>>Abraços,
>>  Gugu
>>
>> Quoting Bernardo Freitas Paulo da Costa :
>>
>> 2018-03-20 23:14 GMT-03:00 Anderson Torres 
>>> :
>>>
 Em 13 de março de 2018 20:19, Douglas Oliveira de Lima
  escreveu:

> Essa achei legal e estou postando.
>
> Resolva nos inteiros a seguinte equação:  (x + y)(y + z)(z + x)/2 + (x
> + y +
> z)3 = 1 – xyz .
>
>
 Substituição mágica: x=-a+b+c, y=a-b+c, z=a+b-c. Com isso, x+y=2c,
 x+y+z=a+b+c e

 4abc + (a+b+c)^3 + (-a+b+c)(a-b+c)(a+b-c) = 1

 Usando polinômios simétricos,

 4(a+b+c)(ab+ac+bc) - 4abc = 1

 Agora estou confuso...

>>>
>>> Note que a,b,c não precisam mais ser inteiros, podem ser inteiros
>>> divididos por 2 (se não me engano)
>>>
>>> Abraços,
>>> --
>>> Bernardo Freitas Paulo da Costa
>>>
>>> --
>>> Esta mensagem foi verificada pelo sistema de antiv?rus e
>>>  acredita-se estar livre de perigo.
>>>
>>>
>>> 
>>> =
>>> Instru??es para entrar na lista, sair da lista e usar a lista em
>>> http://www.mat.puc-rio.br/~obmlistas/obm-l.html
>>> 
>>> =
>>>
>>
>>
>>
>>
>> --
>> Esta mensagem foi verificada pelo sistema de antivírus e
>> acredita-se estar livre de perigo.
>>
>> =
>> Instru�ões para entrar na lista, sair da lista e usar a lista em
>> http://www.mat.puc-rio.br/~obmlistas/obm-l.html
>> =
>>
>
>
> --
> Esta mensagem foi verificada pelo sistema de antivírus e
> acredita-se estar livre de perigo.

-- 
Esta mensagem foi verificada pelo sistema de antiv�rus e
 acredita-se estar livre de perigo.



[obm-l] Re: [obm-l] Re: [obm-l] Re: [obm-l] Teoria dos números

2018-03-22 Por tôpico Claudio Buffara
Tudo muito bom, mas o que ninguém explicou é como foram obtidas as
fatorações/transformações algébricas mágicas.
Insight?
Conhecimentos prévios?
Tentativa e erro e muito braço?

[]s,
Claudio.


2018-03-21 18:54 GMT-03:00 :

> Sim, e fazendo a=u/2, b=v/2 e c=w/2 temos (u+v+w)(uv+uw+vw)-uvw=2, ou
> seja, u^2v+uv^2+u^2w+uw^2+v^2w+vw^2+2uvw=2, mas
> u^2v+uv^2+u^2w+uw^2+v^2w+vw^2+2uvw=(u+v)(u+w)(v+w). Assim, podemos ter
> u+v=2, u+w=v+w=1, o que dá w=0, u=v=1; u+v=2, u+w=v+w=-1, o que dá w=-2,
> u=v=1; u+v=-2, u+w=1, v+w=-1, o que dá w=1, u=0, v=-2; u+v=-2, u+w=-1,
> v+w=1, o que dá w=1, u=-2, v=0 e as coisas simétricas. Daí saem as soluções
> (1,0,0), (2,-1,-1), (-3/2,-1/2,3/2), (-3/2,3/2,-1/2) (e as coisas
> simétricas), mas as duas últimas não são inteiras, de modo que só temos as
> soluções que já tínhamos achado...
>Abraços,
>  Gugu
>
> Quoting Bernardo Freitas Paulo da Costa :
>
> 2018-03-20 23:14 GMT-03:00 Anderson Torres :
>>
>>> Em 13 de março de 2018 20:19, Douglas Oliveira de Lima
>>>  escreveu:
>>>
 Essa achei legal e estou postando.

 Resolva nos inteiros a seguinte equação:  (x + y)(y + z)(z + x)/2 + (x
 + y +
 z)3 = 1 – xyz .


>>> Substituição mágica: x=-a+b+c, y=a-b+c, z=a+b-c. Com isso, x+y=2c,
>>> x+y+z=a+b+c e
>>>
>>> 4abc + (a+b+c)^3 + (-a+b+c)(a-b+c)(a+b-c) = 1
>>>
>>> Usando polinômios simétricos,
>>>
>>> 4(a+b+c)(ab+ac+bc) - 4abc = 1
>>>
>>> Agora estou confuso...
>>>
>>
>> Note que a,b,c não precisam mais ser inteiros, podem ser inteiros
>> divididos por 2 (se não me engano)
>>
>> Abraços,
>> --
>> Bernardo Freitas Paulo da Costa
>>
>> --
>> Esta mensagem foi verificada pelo sistema de antiv?rus e
>>  acredita-se estar livre de perigo.
>>
>>
>> =
>> Instru??es para entrar na lista, sair da lista e usar a lista em
>> http://www.mat.puc-rio.br/~obmlistas/obm-l.html
>> =
>>
>
>
>
>
> --
> Esta mensagem foi verificada pelo sistema de antivírus e
> acredita-se estar livre de perigo.
>
> =
> Instru�ões para entrar na lista, sair da lista e usar a lista em
> http://www.mat.puc-rio.br/~obmlistas/obm-l.html
> =
>

-- 
Esta mensagem foi verificada pelo sistema de antiv�rus e
 acredita-se estar livre de perigo.



[obm-l] Re: [obm-l] Re: [obm-l] Re: [obm-l] Re: [obm-l] Re: [obm-l] Re: [obm-l] Teoria dos números

2018-03-20 Por tôpico Claudio Buffara
Seu orgulho talvez seja justificado!

Como você descobriu que qualquer terno ordenado da forma ( x , y , -(x+y)/2
) é solução da equação "sem o 1"?
Isso não me parece nem um pouco óbvio.

Eu sei que, dados três inteiros, pelo menos dois devem ter a mesma
paridade, e que, como a equação é simétrica em x,y,z, podemos supor spdg
que x e y têm a mesma paridade.
Mas daí a termos z = -(x+y)/2 é um salto bastante longo.

Além disso, supor uma solução com  z = -(x+y)/2 + h  para a equação
original (com o 1) também me parece uma sacada brilhante, ainda que leve a
um "salseiro".

[]s,
Claudio.




2018-03-20 16:33 GMT-03:00 Pedro José :

> Boa tarde!
>
> Ralph,
> parabéns pela sua resolução.
> Já, eu, caminhei por caminhos bem mais tortuosos.
> Embora extremamente deselegante é uma solução.
>
> Se xo,yo,zo é uma solução, temos que pelo menos duas incógnitas têm a
> mesma paridade.
> Como o problema é simétrico, sem perda de generalidade, vamos supor que xo
> e yo tenham a mesma paridade.
> então podemos escrever zo = -(xo+yo)/2 + h, com h inteiro.
> Sabendo-se que : [xo,yo, -(xo+yo)/2] é solução da equação sem o "1".
>
> e substituindo na equação original:
>
> h^3 + 2(xo+yo)h^2+ (3/4 (x+y)^2 + xy) h - 1=0.
>
> Como os coeficientes são inteiros as únicas possibilidades de h inteiro
> são 1 e -1.
>
> h=1.
>
> Seja a= xo +yo
>
> 3a^2 + 8a + 4xoyo=0
>
> xoyo >0, temos que 8|a| > 3a^2 ==> |a| < 8/3 ==>|a|=2, portanto xo= -1 e
> yo=-1(não podem ser positivos). Temos z=-(xo+yo)/2+h=0. (-1,-1,2) e suas
> permutações são soluções.
>
> xo.yo = 0 temos a=0 ou a= -8/3 (não atende) ==> xo=yo=0, z= 1. (0,0,1) e
> suas permutações são soluções.
>
> xo.y0 <0
>
> 3a^2 + 8a + 4xoyo=0
>
> para ter solução a inteiro:Δ = (8 + 6i)^2  ==> 64 - 48 xoyo =  64 + 96 i
> + 36 i^2 ==> xoyo = - (2i + 3î^2/4), com i par.
>
> a= i e xoyo = -(2i+ 3i^2/4) então xo e yo são soluções da equação t^2 -it
> - (2i +3i^2/4) = 0. i =2k; t^2 -2kt-(4k+3k^2)=0
>
> Δ = 4k^2 +16k + 12k^2 = 16k(k+1), que nunca será um quadrado perfeito com
> k<>0. Então não há soluções inteiras. (k=0, recaí em xoyo=0)
>
> h=-1
>
> Seja a= xo+yo
>
> -3a^2 +8a -( 4xoyo - 8) = 0
>
> 4xoyo> 8 ==> 8a >3a^2; a <=2; absurdo não atende 4xoyo>8.
>
> 4xoyo-8=0
>
> temos que a=o, não há inteiros que xoyo=2 e xo+yo=0.
>
> 4xoyo - 8 < 0
>
> Δ = (8 + 6i)^2 ; 64 - 48xoyo + 32 = 64 + 96 i + 36 i^2;  xoyo = -2/3 + 2i
> + 3/4i^2, xoyo não pertence aos inteiros não há solução.
>
>
> Ficam apenas: (0,0,1) ; (0,1,0); (1,0,0) ; (-1,-1,2); (-1,2;-1); (2,-1,-1)
>
> Ralph,
>
> Fiz esse salseiro todo, ao invés de fatorar. E olha, que ontem estava
> orgulhoso de ter achado a solução.
>
>
> Saudações,
> PJMS
>
>
>
> Em 20 de março de 2018 12:10, Pedro José  escreveu:
>
>> É acabou me ajudando. Resolvi de uma outra forma, mais complicada, usando
>> a fórmula. Quando tiver um tempo eu posto.
>>
>>
>> Em 19 de mar de 2018 21:02, "Ralph Teixeira" 
>> escreveu:
>>
>>> Opa, opa, opa! Pedro, voce achou uma formula assim generica, z=-(x+y)/2,
>>> que resolve esta equacao? Beleza, excelente ideia, temos um caminho!
>>>
>>> Porque, se z=-(x+y)/2 eh SEMPRE solucao disso (independente de
>>> "inteiros" ou nao), quer dizer que essa coisa horrorosa, passando tudo para
>>> o outro lado, tem z+(x+y)/2 como fator, ou seja, x+y+2z como fator.
>>> Analogamente, vai ter 2x+y+z e x+2y+z tambem!
>>>
>>> Em suma, a gente pode voltar na primeira equacao com a sua ideia, jogar
>>> tudo para a esquerda (exceto pelo 1 chato que nao aparece na sua
>>> expressao), e fatorar. Vejamos... Acho que fica assim:
>>>
>>> 1/2*(2x+y+z)(x+2y+z)(x+y+2z)=1
>>> (2x+y+z)(x+2y+z)(x+y+2z)=2
>>>
>>> Confiram se eu nao errei contas... Mas agora ficou **bem** facil! :D
>>>
>>> Abraco, Ralph.
>>>
>>> 2018-03-19 14:33 GMT-03:00 Pedro José :
>>>
 Bom dia!

 Estou só conjecturando. Pois, não consegui nenhuma restrição.
 A única coisa que consegui, mas não me adiantou de nada, é que:
 x,y pares ou x,y ímpares e z = -(x+y)/2 é solução de

 *(x + y)(y + z)(z + x)/2 + (x + y + z)3 =  – xyz*
 Também, não consegui provar que é a única família de solução da
 equação acima para inteiros.

 Em 19 de março de 2018 14:14, Claudio Buffara <
 claudio.buff...@gmail.com> escreveu:

> Podem existir soluções não triviais envolvendo inteiros negativos.
>
> 2018-03-19 10:17 GMT-03:00 Pedro José :
>
>> Bom dia!
>>
>> Poderia postar a solução? Não consegui achar nenhuma restrição para
>> trabalhar num subconjunto  pequeno dos inteiros.
>> Creio que vá ser apenas a trivial (0,0,1) e suas permutações.
>>
>> grato,
>> PJMS
>>
>> Em 13 de março de 2018 20:19, Douglas Oliveira de Lima <
>> profdouglaso.del...@gmail.com> escreveu:
>>
>>> Essa achei legal e estou postando.
>>>
>>> *Resolva nos inteiros a seguinte equação:  (x + y)(y + z)(z + x)/2 +
>>> 

[obm-l] Re: [obm-l] Re: [obm-l] Re: [obm-l] Re: [obm-l] Re: [obm-l] Teoria dos números

2018-03-20 Por tôpico Pedro José
Boa tarde!

Ralph,
parabéns pela sua resolução.
Já, eu, caminhei por caminhos bem mais tortuosos.
Embora extremamente deselegante é uma solução.

Se xo,yo,zo é uma solução, temos que pelo menos duas incógnitas têm a mesma
paridade.
Como o problema é simétrico, sem perda de generalidade, vamos supor que xo
e yo tenham a mesma paridade.
então podemos escrever zo = -(xo+yo)/2 + h, com h inteiro.
Sabendo-se que : [xo,yo, -(xo+yo)/2] é solução da equação sem o "1".

e substituindo na equação original:

h^3 + 2(xo+yo)h^2+ (3/4 (x+y)^2 + xy) h - 1=0.

Como os coeficientes são inteiros as únicas possibilidades de h inteiro são
1 e -1.

h=1.

Seja a= xo +yo

3a^2 + 8a + 4xoyo=0

xoyo >0, temos que 8|a| > 3a^2 ==> |a| < 8/3 ==>|a|=2, portanto xo= -1 e
yo=-1(não podem ser positivos). Temos z=-(xo+yo)/2+h=0. (-1,-1,2) e suas
permutações são soluções.

xo.yo = 0 temos a=0 ou a= -8/3 (não atende) ==> xo=yo=0, z= 1. (0,0,1) e
suas permutações são soluções.

xo.y0 <0

3a^2 + 8a + 4xoyo=0

para ter solução a inteiro:Δ = (8 + 6i)^2  ==> 64 - 48 xoyo =  64 + 96 i +
36 i^2 ==> xoyo = - (2i + 3î^2/4), com i par.

a= i e xoyo = -(2i+ 3i^2/4) então xo e yo são soluções da equação t^2 -it -
(2i +3i^2/4) = 0. i =2k; t^2 -2kt-(4k+3k^2)=0

Δ = 4k^2 +16k + 12k^2 = 16k(k+1), que nunca será um quadrado perfeito com
k<>0. Então não há soluções inteiras. (k=0, recaí em xoyo=0)

h=-1

Seja a= xo+yo

-3a^2 +8a -( 4xoyo - 8) = 0

4xoyo> 8 ==> 8a >3a^2; a <=2; absurdo não atende 4xoyo>8.

4xoyo-8=0

temos que a=o, não há inteiros que xoyo=2 e xo+yo=0.

4xoyo - 8 < 0

Δ = (8 + 6i)^2 ; 64 - 48xoyo + 32 = 64 + 96 i + 36 i^2;  xoyo = -2/3 + 2i +
3/4i^2, xoyo não pertence aos inteiros não há solução.


Ficam apenas: (0,0,1) ; (0,1,0); (1,0,0) ; (-1,-1,2); (-1,2;-1); (2,-1,-1)

Ralph,

Fiz esse salseiro todo, ao invés de fatorar. E olha, que ontem estava
orgulhoso de ter achado a solução.


Saudações,
PJMS



Em 20 de março de 2018 12:10, Pedro José  escreveu:

> É acabou me ajudando. Resolvi de uma outra forma, mais complicada, usando
> a fórmula. Quando tiver um tempo eu posto.
>
>
> Em 19 de mar de 2018 21:02, "Ralph Teixeira"  escreveu:
>
>> Opa, opa, opa! Pedro, voce achou uma formula assim generica, z=-(x+y)/2,
>> que resolve esta equacao? Beleza, excelente ideia, temos um caminho!
>>
>> Porque, se z=-(x+y)/2 eh SEMPRE solucao disso (independente de "inteiros"
>> ou nao), quer dizer que essa coisa horrorosa, passando tudo para o outro
>> lado, tem z+(x+y)/2 como fator, ou seja, x+y+2z como fator. Analogamente,
>> vai ter 2x+y+z e x+2y+z tambem!
>>
>> Em suma, a gente pode voltar na primeira equacao com a sua ideia, jogar
>> tudo para a esquerda (exceto pelo 1 chato que nao aparece na sua
>> expressao), e fatorar. Vejamos... Acho que fica assim:
>>
>> 1/2*(2x+y+z)(x+2y+z)(x+y+2z)=1
>> (2x+y+z)(x+2y+z)(x+y+2z)=2
>>
>> Confiram se eu nao errei contas... Mas agora ficou **bem** facil! :D
>>
>> Abraco, Ralph.
>>
>> 2018-03-19 14:33 GMT-03:00 Pedro José :
>>
>>> Bom dia!
>>>
>>> Estou só conjecturando. Pois, não consegui nenhuma restrição.
>>> A única coisa que consegui, mas não me adiantou de nada, é que:
>>> x,y pares ou x,y ímpares e z = -(x+y)/2 é solução de
>>>
>>> *(x + y)(y + z)(z + x)/2 + (x + y + z)3 =  – xyz*
>>> Também, não consegui provar que é a única família de solução da equação
>>> acima para inteiros.
>>>
>>> Em 19 de março de 2018 14:14, Claudio Buffara >> > escreveu:
>>>
 Podem existir soluções não triviais envolvendo inteiros negativos.

 2018-03-19 10:17 GMT-03:00 Pedro José :

> Bom dia!
>
> Poderia postar a solução? Não consegui achar nenhuma restrição para
> trabalhar num subconjunto  pequeno dos inteiros.
> Creio que vá ser apenas a trivial (0,0,1) e suas permutações.
>
> grato,
> PJMS
>
> Em 13 de março de 2018 20:19, Douglas Oliveira de Lima <
> profdouglaso.del...@gmail.com> escreveu:
>
>> Essa achei legal e estou postando.
>>
>> *Resolva nos inteiros a seguinte equação:  (x + y)(y + z)(z + x)/2 +
>> (x + y + z)3 = 1 – xyz* .
>>
>> Abraço do
>> Douglas Oliveira
>>
>> --
>> Esta mensagem foi verificada pelo sistema de antivírus e
>> acredita-se estar livre de perigo.
>
>
>
> --
> Esta mensagem foi verificada pelo sistema de antivírus e
> acredita-se estar livre de perigo.
>


 --
 Esta mensagem foi verificada pelo sistema de antivírus e
 acredita-se estar livre de perigo.

>>>
>>>
>>> --
>>> Esta mensagem foi verificada pelo sistema de antivírus e
>>> acredita-se estar livre de perigo.
>>>
>>
>>
>> --
>> Esta mensagem foi verificada pelo sistema de antivírus e
>> acredita-se estar livre de perigo.
>
>

-- 
Esta mensagem foi verificada pelo sistema de antiv�rus e
 acredita-se estar livre de perigo.



[obm-l] Re: [obm-l] Re: [obm-l] Re: [obm-l] Re: [obm-l] Re: [obm-l] Teoria dos números

2018-03-20 Por tôpico Pedro José
É acabou me ajudando. Resolvi de uma outra forma, mais complicada, usando a
fórmula. Quando tiver um tempo eu posto.

Em 19 de mar de 2018 21:02, "Ralph Teixeira"  escreveu:

> Opa, opa, opa! Pedro, voce achou uma formula assim generica, z=-(x+y)/2,
> que resolve esta equacao? Beleza, excelente ideia, temos um caminho!
>
> Porque, se z=-(x+y)/2 eh SEMPRE solucao disso (independente de "inteiros"
> ou nao), quer dizer que essa coisa horrorosa, passando tudo para o outro
> lado, tem z+(x+y)/2 como fator, ou seja, x+y+2z como fator. Analogamente,
> vai ter 2x+y+z e x+2y+z tambem!
>
> Em suma, a gente pode voltar na primeira equacao com a sua ideia, jogar
> tudo para a esquerda (exceto pelo 1 chato que nao aparece na sua
> expressao), e fatorar. Vejamos... Acho que fica assim:
>
> 1/2*(2x+y+z)(x+2y+z)(x+y+2z)=1
> (2x+y+z)(x+2y+z)(x+y+2z)=2
>
> Confiram se eu nao errei contas... Mas agora ficou **bem** facil! :D
>
> Abraco, Ralph.
>
> 2018-03-19 14:33 GMT-03:00 Pedro José :
>
>> Bom dia!
>>
>> Estou só conjecturando. Pois, não consegui nenhuma restrição.
>> A única coisa que consegui, mas não me adiantou de nada, é que:
>> x,y pares ou x,y ímpares e z = -(x+y)/2 é solução de
>>
>> *(x + y)(y + z)(z + x)/2 + (x + y + z)3 =  – xyz*
>> Também, não consegui provar que é a única família de solução da equação
>> acima para inteiros.
>>
>> Em 19 de março de 2018 14:14, Claudio Buffara 
>> escreveu:
>>
>>> Podem existir soluções não triviais envolvendo inteiros negativos.
>>>
>>> 2018-03-19 10:17 GMT-03:00 Pedro José :
>>>
 Bom dia!

 Poderia postar a solução? Não consegui achar nenhuma restrição para
 trabalhar num subconjunto  pequeno dos inteiros.
 Creio que vá ser apenas a trivial (0,0,1) e suas permutações.

 grato,
 PJMS

 Em 13 de março de 2018 20:19, Douglas Oliveira de Lima <
 profdouglaso.del...@gmail.com> escreveu:

> Essa achei legal e estou postando.
>
> *Resolva nos inteiros a seguinte equação:  (x + y)(y + z)(z + x)/2 +
> (x + y + z)3 = 1 – xyz* .
>
> Abraço do
> Douglas Oliveira
>
> --
> Esta mensagem foi verificada pelo sistema de antivírus e
> acredita-se estar livre de perigo.



 --
 Esta mensagem foi verificada pelo sistema de antivírus e
 acredita-se estar livre de perigo.

>>>
>>>
>>> --
>>> Esta mensagem foi verificada pelo sistema de antivírus e
>>> acredita-se estar livre de perigo.
>>>
>>
>>
>> --
>> Esta mensagem foi verificada pelo sistema de antivírus e
>> acredita-se estar livre de perigo.
>>
>
>
> --
> Esta mensagem foi verificada pelo sistema de antivírus e
> acredita-se estar livre de perigo.

-- 
Esta mensagem foi verificada pelo sistema de antiv�rus e
 acredita-se estar livre de perigo.



[obm-l] Re: [obm-l] Re: [obm-l] Re: [obm-l] Re: [obm-l] Teoria dos números

2018-03-19 Por tôpico Ralph Teixeira
Opa, opa, opa! Pedro, voce achou uma formula assim generica, z=-(x+y)/2,
que resolve esta equacao? Beleza, excelente ideia, temos um caminho!

Porque, se z=-(x+y)/2 eh SEMPRE solucao disso (independente de "inteiros"
ou nao), quer dizer que essa coisa horrorosa, passando tudo para o outro
lado, tem z+(x+y)/2 como fator, ou seja, x+y+2z como fator. Analogamente,
vai ter 2x+y+z e x+2y+z tambem!

Em suma, a gente pode voltar na primeira equacao com a sua ideia, jogar
tudo para a esquerda (exceto pelo 1 chato que nao aparece na sua
expressao), e fatorar. Vejamos... Acho que fica assim:

1/2*(2x+y+z)(x+2y+z)(x+y+2z)=1
(2x+y+z)(x+2y+z)(x+y+2z)=2

Confiram se eu nao errei contas... Mas agora ficou **bem** facil! :D

Abraco, Ralph.

2018-03-19 14:33 GMT-03:00 Pedro José :

> Bom dia!
>
> Estou só conjecturando. Pois, não consegui nenhuma restrição.
> A única coisa que consegui, mas não me adiantou de nada, é que:
> x,y pares ou x,y ímpares e z = -(x+y)/2 é solução de
>
> *(x + y)(y + z)(z + x)/2 + (x + y + z)3 =  – xyz*
> Também, não consegui provar que é a única família de solução da equação
> acima para inteiros.
>
> Em 19 de março de 2018 14:14, Claudio Buffara 
> escreveu:
>
>> Podem existir soluções não triviais envolvendo inteiros negativos.
>>
>> 2018-03-19 10:17 GMT-03:00 Pedro José :
>>
>>> Bom dia!
>>>
>>> Poderia postar a solução? Não consegui achar nenhuma restrição para
>>> trabalhar num subconjunto  pequeno dos inteiros.
>>> Creio que vá ser apenas a trivial (0,0,1) e suas permutações.
>>>
>>> grato,
>>> PJMS
>>>
>>> Em 13 de março de 2018 20:19, Douglas Oliveira de Lima <
>>> profdouglaso.del...@gmail.com> escreveu:
>>>
 Essa achei legal e estou postando.

 *Resolva nos inteiros a seguinte equação:  (x + y)(y + z)(z + x)/2 + (x
 + y + z)3 = 1 – xyz* .

 Abraço do
 Douglas Oliveira

 --
 Esta mensagem foi verificada pelo sistema de antivírus e
 acredita-se estar livre de perigo.
>>>
>>>
>>>
>>> --
>>> Esta mensagem foi verificada pelo sistema de antivírus e
>>> acredita-se estar livre de perigo.
>>>
>>
>>
>> --
>> Esta mensagem foi verificada pelo sistema de antivírus e
>> acredita-se estar livre de perigo.
>>
>
>
> --
> Esta mensagem foi verificada pelo sistema de antivírus e
> acredita-se estar livre de perigo.
>

-- 
Esta mensagem foi verificada pelo sistema de antiv�rus e
 acredita-se estar livre de perigo.



[obm-l] Re: [obm-l] Re: [obm-l] Re: [obm-l] Teoria dos números

2018-03-19 Por tôpico Claudio Buffara
De fato, procurando soluções com x+y+z = 0, a equação fica:
(-z)(-x)(-y)/2 + 0^3 = 1 - xyz ==>
-xyz/2 = 1 - xyz ==>
xyz = 2 ==>
(x,y,z) = (-1,-1,2) ou (-1,2,-1) ou (2,-1,-1)

Mas ainda não se provou que estas são as únicas soluções.




2018-03-19 14:22 GMT-03:00 Douglas Oliveira de Lima <
profdouglaso.del...@gmail.com>:

> E (-1,-1,2) e suas permutacoes.
>
> Em 19 de mar de 2018 10:25, "Pedro José"  escreveu:
>
>> Bom dia!
>>
>> Poderia postar a solução? Não consegui achar nenhuma restrição para
>> trabalhar num subconjunto  pequeno dos inteiros.
>> Creio que vá ser apenas a trivial (0,0,1) e suas permutações.
>>
>> grato,
>> PJMS
>>
>> Em 13 de março de 2018 20:19, Douglas Oliveira de Lima <
>> profdouglaso.del...@gmail.com> escreveu:
>>
>>> Essa achei legal e estou postando.
>>>
>>> *Resolva nos inteiros a seguinte equação:  (x + y)(y + z)(z + x)/2 + (x
>>> + y + z)3 = 1 – xyz* .
>>>
>>> Abraço do
>>> Douglas Oliveira
>>>
>>> --
>>> Esta mensagem foi verificada pelo sistema de antivírus e
>>> acredita-se estar livre de perigo.
>>
>>
>>
>> --
>> Esta mensagem foi verificada pelo sistema de antivírus e
>> acredita-se estar livre de perigo.
>
>
> --
> Esta mensagem foi verificada pelo sistema de antivírus e
> acredita-se estar livre de perigo.
>

-- 
Esta mensagem foi verificada pelo sistema de antiv�rus e
 acredita-se estar livre de perigo.



[obm-l] Re: [obm-l] Re: [obm-l] Re: [obm-l] Teoria dos números

2018-03-19 Por tôpico Pedro José
Bom dia!

Estou só conjecturando. Pois, não consegui nenhuma restrição.
A única coisa que consegui, mas não me adiantou de nada, é que:
x,y pares ou x,y ímpares e z = -(x+y)/2 é solução de

*(x + y)(y + z)(z + x)/2 + (x + y + z)3 =  – xyz*
Também, não consegui provar que é a única família de solução da equação
acima para inteiros.

Em 19 de março de 2018 14:14, Claudio Buffara 
escreveu:

> Podem existir soluções não triviais envolvendo inteiros negativos.
>
> 2018-03-19 10:17 GMT-03:00 Pedro José :
>
>> Bom dia!
>>
>> Poderia postar a solução? Não consegui achar nenhuma restrição para
>> trabalhar num subconjunto  pequeno dos inteiros.
>> Creio que vá ser apenas a trivial (0,0,1) e suas permutações.
>>
>> grato,
>> PJMS
>>
>> Em 13 de março de 2018 20:19, Douglas Oliveira de Lima <
>> profdouglaso.del...@gmail.com> escreveu:
>>
>>> Essa achei legal e estou postando.
>>>
>>> *Resolva nos inteiros a seguinte equação:  (x + y)(y + z)(z + x)/2 + (x
>>> + y + z)3 = 1 – xyz* .
>>>
>>> Abraço do
>>> Douglas Oliveira
>>>
>>> --
>>> Esta mensagem foi verificada pelo sistema de antivírus e
>>> acredita-se estar livre de perigo.
>>
>>
>>
>> --
>> Esta mensagem foi verificada pelo sistema de antivírus e
>> acredita-se estar livre de perigo.
>>
>
>
> --
> Esta mensagem foi verificada pelo sistema de antivírus e
> acredita-se estar livre de perigo.
>

-- 
Esta mensagem foi verificada pelo sistema de antiv�rus e
 acredita-se estar livre de perigo.



[obm-l] Re: [obm-l] Re: [obm-l] Re: [obm-l] Teoria dos números

2017-11-23 Por tôpico Pedro José
Bom dia!
Consegui entender como o Anderson chegou a solução. E realmente é 3^n no
denominador, ou seja, (10^k-1)/3^n, onde 10^k = 1 mod3^n.
E o número m, de algarismos zeros, que deve ser acrescidos a esquerda do
resultado acima é o menor inteiro m, que atende 10^(m+1) > 3^n.
Para o caso particular 1/3^2002, seriam 955 algarismos zeros inseridos a
esquerda.

Saudações,
PJMS


Em 22 de novembro de 2017 11:45, Pedro José  escreveu:

> Bom dia!
>
> Não entendi como o Anderson chegou a solução, para determinar o período
> propriamente dito. Todavia fiz um experimento e realmente dá certo para
> (10^k-1)/3^n. Acho que ele se enganou e reportou n somente ao invés de
> 3^n.
> Todavia, em alguns casos, precisa colocar algarismos zeros a esquerda do
> resultado
> Por exemplo para 1/3^3.
> O menor expoente que satisfaz 10^k= 1 mod27 é k= 3
> então o período seria (10^3-1)/3 = 37.
> Porém, usando a fórmula proposta pelo Bernardo, o número de algarismos do
> período seria = 3^(3-2) = 3.
> Portanto, seria necessário completar com "zeros" a esquerda para alguns
> casos e creio que seja 3^2002 no denominador e não 2002, como apresentado.
>
> O mesmo acontece para 1/3^4
> a ordem de 10 mod81 é 9
> então o período daria (10^9-1)/81 = 12345679.
> Porém o número de algarismos do período é, segundo o Bernardo, e com
> propriedade: 3^(2)=9 e novamente precisamos acrescentar um algarismo zero a
> esquerda.
>
> Um outro ponto, é quanto a quantidade de átomos do universo. É uma
> estimativa, e para o universo observável, ou seja, cuja distância a partir
> de um observador é menor ou igual do que a distância percorrida pela luz,
> desde o Big Bang até o instante de observação. Minha crença é de que o
> universo é infinito e não limitado a três dimensões, as três dimensões é
> uma limitação nossa e não existencial. Mas...
> Ademais, a estimativa além de considerar o universo como finito, despreza
> possíveis fontes de átomos, só considerando os das estrelas e usa
> equivalência para átomos de Hidrogênio.
>
> Bernardo,
> pode ficar tranquilo que a ordem de grandeza, tradicional, para a
> estimativa da quantidade de átomos do universo é 10^80 (na verdade a
> estimativa é 4 * 10^79, mas como 4 > 3,16 a ordem é 10^80).
> Já eu tenho que tomar um remédio para memória que é muito bom,
> sensacional. Se alguém precisar, posso passar o nome depois, pois não
> lembro o nome. Fiquei em dúvida, agora, não me lembro se tomei o remédio
> hoje ou não.
>
> Saudações,
> PJMS.
>
>
>
> Em 21 de novembro de 2017 23:27, Bernardo Freitas Paulo da Costa <
> bernardo...@gmail.com> escreveu:
>
>> 2017-11-21 22:41 GMT-02:00 Anderson Torres 
>> :
>> > Que treta... Bem, a ideia seria descobrir a potência de dez que deixa
>> > resto um módulo 3^2002, e daí realizar a divisão longa
>> > ((10^k-1)/2002)...
>> >
>> > Em 21 de novembro de 2017 17:13, Vinícius Raimundo
>> >  escreveu:
>> >> Encontre o período na representação decimal de 1/3^2002
>>
>> Eu acho que por "o período" o enunciado quer dizer o número de dígitos
>> no período, (assim 1/3 = 0.3... tem período 1, 1/7 =
>> 0.142857142... tem período 6, etc).
>>
>> E neste caso uma solução por recorrência mostra que o período é
>> 3^{n-2} para n >= 2 (se não me falha a memória).  Se for isso, basta
>> mostrar que 10^{3^n} - 1 é divisível por 3^{n+2}, mas não por 3^{n+3}.
>> Com n=0, isso é 10^1 - 1 é divisível por 9 = 3^2, o passo de indução
>> você usa (a-1)^ = a^3 - 3a^2 + 3a - 1.  Agora, calcular a parte
>> periódica, vai demorar... Se eu acertei as contas, tem "só" 3^2000
>> dígitos, que é muito mais do que o número de átomos no universo (por
>> volta de 10^{80}, de novo de memória) :)
>>
>> Abraços,
>> --
>> Bernardo Freitas Paulo da Costa
>>
>> --
>> Esta mensagem foi verificada pelo sistema de antivírus e
>>  acredita-se estar livre de perigo.
>>
>>
>> =
>> Instru�ões para entrar na lista, sair da lista e usar a lista em
>> http://www.mat.puc-rio.br/~obmlistas/obm-l.html
>> =
>>
>
>

-- 
Esta mensagem foi verificada pelo sistema de antiv�rus e
 acredita-se estar livre de perigo.



[obm-l] Re: [obm-l] Re: [obm-l] Re: [obm-l] Teoria dos números

2017-11-22 Por tôpico Pedro José
Bom dia!

Não entendi como o Anderson chegou a solução, para determinar o período
propriamente dito. Todavia fiz um experimento e realmente dá certo para
(10^k-1)/3^n. Acho que ele se enganou e reportou n somente ao invés de 3^n.
Todavia, em alguns casos, precisa colocar algarismos zeros a esquerda do
resultado
Por exemplo para 1/3^3.
O menor expoente que satisfaz 10^k= 1 mod27 é k= 3
então o período seria (10^3-1)/3 = 37.
Porém, usando a fórmula proposta pelo Bernardo, o número de algarismos do
período seria = 3^(3-2) = 3.
Portanto, seria necessário completar com "zeros" a esquerda para alguns
casos e creio que seja 3^2002 no denominador e não 2002, como apresentado.

O mesmo acontece para 1/3^4
a ordem de 10 mod81 é 9
então o período daria (10^9-1)/81 = 12345679.
Porém o número de algarismos do período é, segundo o Bernardo, e com
propriedade: 3^(2)=9 e novamente precisamos acrescentar um algarismo zero a
esquerda.

Um outro ponto, é quanto a quantidade de átomos do universo. É uma
estimativa, e para o universo observável, ou seja, cuja distância a partir
de um observador é menor ou igual do que a distância percorrida pela luz,
desde o Big Bang até o instante de observação. Minha crença é de que o
universo é infinito e não limitado a três dimensões, as três dimensões é
uma limitação nossa e não existencial. Mas...
Ademais, a estimativa além de considerar o universo como finito, despreza
possíveis fontes de átomos, só considerando os das estrelas e usa
equivalência para átomos de Hidrogênio.

Bernardo,
pode ficar tranquilo que a ordem de grandeza, tradicional, para a
estimativa da quantidade de átomos do universo é 10^80 (na verdade a
estimativa é 4 * 10^79, mas como 4 > 3,16 a ordem é 10^80).
Já eu tenho que tomar um remédio para memória que é muito bom, sensacional.
Se alguém precisar, posso passar o nome depois, pois não lembro o nome.
Fiquei em dúvida, agora, não me lembro se tomei o remédio hoje ou não.

Saudações,
PJMS.



Em 21 de novembro de 2017 23:27, Bernardo Freitas Paulo da Costa <
bernardo...@gmail.com> escreveu:

> 2017-11-21 22:41 GMT-02:00 Anderson Torres :
> > Que treta... Bem, a ideia seria descobrir a potência de dez que deixa
> > resto um módulo 3^2002, e daí realizar a divisão longa
> > ((10^k-1)/2002)...
> >
> > Em 21 de novembro de 2017 17:13, Vinícius Raimundo
> >  escreveu:
> >> Encontre o período na representação decimal de 1/3^2002
>
> Eu acho que por "o período" o enunciado quer dizer o número de dígitos
> no período, (assim 1/3 = 0.3... tem período 1, 1/7 =
> 0.142857142... tem período 6, etc).
>
> E neste caso uma solução por recorrência mostra que o período é
> 3^{n-2} para n >= 2 (se não me falha a memória).  Se for isso, basta
> mostrar que 10^{3^n} - 1 é divisível por 3^{n+2}, mas não por 3^{n+3}.
> Com n=0, isso é 10^1 - 1 é divisível por 9 = 3^2, o passo de indução
> você usa (a-1)^ = a^3 - 3a^2 + 3a - 1.  Agora, calcular a parte
> periódica, vai demorar... Se eu acertei as contas, tem "só" 3^2000
> dígitos, que é muito mais do que o número de átomos no universo (por
> volta de 10^{80}, de novo de memória) :)
>
> Abraços,
> --
> Bernardo Freitas Paulo da Costa
>
> --
> Esta mensagem foi verificada pelo sistema de antivírus e
>  acredita-se estar livre de perigo.
>
>
> =
> Instru�ões para entrar na lista, sair da lista e usar a lista em
> http://www.mat.puc-rio.br/~obmlistas/obm-l.html
> =
>

-- 
Esta mensagem foi verificada pelo sistema de antiv�rus e
 acredita-se estar livre de perigo.



[obm-l] Re: [obm-l] Re: [obm-l] Re: [obm-l] Teoria dos números

2017-07-07 Por tôpico Pedro José
Em 7 de julho de 2017 10:36, Pedro José  escreveu:

> Bom dia!
>
> Faltou um pedacinho.
>
> 2) a < max(b,c)
>
> (i) b >= c
>
> c=2 ==> a^b+b^2=2ab ==> a >b/2 ==> (b/s)^b < 2ab ==> (b/2)^b<2b^2
>
> Para b>=3 (b/2)^b cresce mais rápido que 2b^2 e b=5 ==> (5/2)^5 > 50 ==>
> b<=4.
>
> Temos b=4 ou b=3 ou b=2.
>
> b=4 ==> a^4 + 16 = 8a, a< 4; não atende para a=1, a=2, a=3.
>
> b=3 ==> a^3 + 9 = 6a, a <3; não atende para a=1 ou a=2.
>
> b=2 ==> a^2 + 4 = 4a. a < 2; a=1 não atende.
>
> Não surgiu nenhum terno novo, mas de toda sorte, deveria ter sido posto a
> prova sob essa condição que fora negligenciada.
>
> Saudações,
> PJMS
>
>
> Em 7 de julho de 2017 08:52, Pedro José  escreveu:
>
>> Bom dia!
>>
>> Desculpe-me pela solução. Não consegui nada elegante, fui para grosseria.
>> Fui fatiando.
>>
>> 1) a >= max(b,c)
>>
>>
>> (i) a=b=c ==> b<=3; pois a^b+b^c> a^b e a^b>abc=a^3 se a>4.
>> Por paridade só 2 atende, testando é solução. (2,2,2)
>>
>>  (ii) a=b>c ==> b<=2; pois, a^3 +b^c> a^3>abc=a^2c
>>
>>  b=1 absurdo, pois b>c.
>>
>>  a=2 e b=2 ==>c=1, não atende.
>>
>>
>> (iii) a=c>b ==>b<=2; pois a^3+b^c>a^3>abc=a^2b.
>>
>>  b=1 ==> a+1 = ac ==> a| a +1 ==> a =1; absurdo pois a>b=1.
>>
>>  b=2 ==> a^2 +2^c = 2ac ==> a= c [image: Imagem inline 1] raiz (c^2-2^c)
>>
>> c^2 cresce mais lentamente que c^2 para c>=3. Portanto c<=4, pois 5^2 <
>> 2^5.
>>
>> c=2 ou c=3 ou c=4.
>>
>> c=2 ==> a= 2 e b=1, não atende; pois b=2.
>>
>> c=3 ==>a=4, não atende a restrição(iii)  a=c.
>>
>> c=4 ==>a=c=4 e b=2, atende. outra solução. (4,2,4)
>>
>>
>>
>> (iv) a>max (b,c)
>>
>>
>> b<=2; pois, a^3+b^c > a^3>abc
>>
>> a= c [image: Imagem inline 1] raiz (c^2-2^c). Novamente temos a
>> restrição c<=4.
>>
>> c=1 ==> a= 1; não atende (iv)
>>
>> c=2 ==> a=c=b; não atende (iv)
>>
>> c=3 ==> a= 4 atende. (4,2,3) é solução.
>>
>>
>> c= 4 não atende (iv); pois, a=b=4
>>
>> Agora é partir para o complemento de 1)
>>
>>
>> 2) a < max(b,c)
>>
>>
>> (i) b >= c
>>
>>
>> b>=c ==> c<=2; pois, a^b+b^3>b^3>abc.
>>
>> c=1 ==> a^b+b = ab ==> a^b < ab ==> a^(b-1) < b ==> a=1 ==> 1+b =b,
>> absurdo.
>> para a>=2: a^(b-1) > b; pois b>a.
>>
>>
>> (ii) c > b
>>
>>
>> Para b=1 ==> a + 1 =ac ==> a | a + 1 ==> a=1 ==> c=2; atende (1,1,2) é
>> solução
>>
>>
>>
>> Para b=2 ==> a^2 + 2^c = 2ac ==> 2^c < 2ac ==> 2^(c-1) > Mas 2^(c-1) > c^2 para c>6.
>> Então:
>> c=3, c= 4 ou c= 5 ou c=6.
>>
>>
>> c=3 ==> a^2+8 =6a ==> a=2 ou a= 4. a=4 fere 2).
>> (2,2,3) atende.
>>
>> c=4 ==> a^2 + 16 = 8a ==> a=4=c não atende 2.
>>
>> c=5==> a^2 + 32 = 10a ==> não há raízes reais , não atende.
>>
>> c= 6 ==> a^2 + 64 = 12 a ==> não há raízes reais , não atende.
>>
>>
>>
>> Para b >=3 ==> b^(c-1) > c^2, não há mais soluções.
>>
>>
>> s= {(2,2,2); (4,2,4); (4,2,3); (1,1,2); (2,2,3)}
>>
>> Foi na marra, sem talento, mas acho que são só esses 5 ternos.
>>
>> Saudações,
>> PJMS.
>>
>>
>>
>>
>> Em 6 de julho de 2017 23:25, Douglas Oliveira de Lima <
>> profdouglaso.del...@gmail.com> escreveu:
>>
>>> Opa , sim, é a•b•c
>>>
>>> Em 6 de jul de 2017 11:14 PM, "Carlos Nehab" 
>>> escreveu:
>>>
 Oi, Douglas,

 Esse "abc" é a x b x c (produto) ou o inteiro de algarismos a, b e c
 (100a+10b+c)?

 Abs
 Nehab


 
  Livre
 de vírus. www.avast.com
 .

 <#m_-8456725342991579909_m_-2253413763399002655_m_2930426211637426034_m_1267597801263667645_DAB4FAD8-2DD7-40BB-A1B8-4E2AA1F9FDF2>

 Em 6 de julho de 2017 14:03, Douglas Oliveira de Lima <
 profdouglaso.del...@gmail.com> escreveu:

> Encontrar todos os inteiros positivos a,b e c tais que a^b+b^c=abc.
>
> --
> Esta mensagem foi verificada pelo sistema de antivírus e
> acredita-se estar livre de perigo.



 --
 Esta mensagem foi verificada pelo sistema de antivírus e
 acredita-se estar livre de perigo.
>>>
>>>
>>> --
>>> Esta mensagem foi verificada pelo sistema de antivírus e
>>> acredita-se estar livre de perigo.
>>>
>>
>>
>

-- 
Esta mensagem foi verificada pelo sistema de antiv�rus e
 acredita-se estar livre de perigo.



[obm-l] Re: [obm-l] Re: [obm-l] Re: [obm-l] Teoria dos números

2017-07-07 Por tôpico Pedro José
Bom dia!

Desculpe-me pela solução. Não consegui nada elegante, fui para grosseria.
Fui fatiando.

1) a >= max(b,c)


(i) a=b=c ==> b<=3; pois a^b+b^c> a^b e a^b>abc=a^3 se a>4.
Por paridade só 2 atende, testando é solução. (2,2,2)

 (ii) a=b>c ==> b<=2; pois, a^3 +b^c> a^3>abc=a^2c

 b=1 absurdo, pois b>c.

 a=2 e b=2 ==>c=1, não atende.


(iii) a=c>b ==>b<=2; pois a^3+b^c>a^3>abc=a^2b.

 b=1 ==> a+1 = ac ==> a| a +1 ==> a =1; absurdo pois a>b=1.

 b=2 ==> a^2 +2^c = 2ac ==> a= c [image: Imagem inline 1] raiz (c^2-2^c)

c^2 cresce mais lentamente que c^2 para c>=3. Portanto c<=4, pois 5^2 < 2^5.

c=2 ou c=3 ou c=4.

c=2 ==> a= 2 e b=1, não atende; pois b=2.

c=3 ==>a=4, não atende a restrição(iii)  a=c.

c=4 ==>a=c=4 e b=2, atende. outra solução. (4,2,4)



(iv) a>max (b,c)


b<=2; pois, a^3+b^c > a^3>abc

a= c [image: Imagem inline 1] raiz (c^2-2^c). Novamente temos a restrição
c<=4.

c=1 ==> a= 1; não atende (iv)

c=2 ==> a=c=b; não atende (iv)

c=3 ==> a= 4 atende. (4,2,3) é solução.


c= 4 não atende (iv); pois, a=b=4

Agora é partir para o complemento de 1)


2) a < max(b,c)


(i) b >= c


b>=c ==> c<=2; pois, a^b+b^3>b^3>abc.

c=1 ==> a^b+b = ab ==> a^b < ab ==> a^(b-1) < b ==> a=1 ==> 1+b =b, absurdo.
para a>=2: a^(b-1) > b; pois b>a.


(ii) c > b


Para b=1 ==> a + 1 =ac ==> a | a + 1 ==> a=1 ==> c=2; atende (1,1,2) é
solução



Para b=2 ==> a^2 + 2^c = 2ac ==> 2^c < 2ac ==> 2^(c-1)  c^2 para c>6.
Então:
c=3, c= 4 ou c= 5 ou c=6.


c=3 ==> a^2+8 =6a ==> a=2 ou a= 4. a=4 fere 2).
(2,2,3) atende.

c=4 ==> a^2 + 16 = 8a ==> a=4=c não atende 2.

c=5==> a^2 + 32 = 10a ==> não há raízes reais , não atende.

c= 6 ==> a^2 + 64 = 12 a ==> não há raízes reais , não atende.



Para b >=3 ==> b^(c-1) > c^2, não há mais soluções.


s= {(2,2,2); (4,2,4); (4,2,3); (1,1,2); (2,2,3)}

Foi na marra, sem talento, mas acho que são só esses 5 ternos.

Saudações,
PJMS.




Em 6 de julho de 2017 23:25, Douglas Oliveira de Lima <
profdouglaso.del...@gmail.com> escreveu:

> Opa , sim, é a•b•c
>
> Em 6 de jul de 2017 11:14 PM, "Carlos Nehab" 
> escreveu:
>
>> Oi, Douglas,
>>
>> Esse "abc" é a x b x c (produto) ou o inteiro de algarismos a, b e c
>> (100a+10b+c)?
>>
>> Abs
>> Nehab
>>
>>
>> 
>>  Livre
>> de vírus. www.avast.com
>> .
>>
>> <#m_2930426211637426034_m_1267597801263667645_DAB4FAD8-2DD7-40BB-A1B8-4E2AA1F9FDF2>
>>
>> Em 6 de julho de 2017 14:03, Douglas Oliveira de Lima <
>> profdouglaso.del...@gmail.com> escreveu:
>>
>>> Encontrar todos os inteiros positivos a,b e c tais que a^b+b^c=abc.
>>>
>>> --
>>> Esta mensagem foi verificada pelo sistema de antivírus e
>>> acredita-se estar livre de perigo.
>>
>>
>>
>> --
>> Esta mensagem foi verificada pelo sistema de antivírus e
>> acredita-se estar livre de perigo.
>
>
> --
> Esta mensagem foi verificada pelo sistema de antivírus e
> acredita-se estar livre de perigo.
>

-- 
Esta mensagem foi verificada pelo sistema de antiv�rus e
 acredita-se estar livre de perigo.



[obm-l] Re: [obm-l] Re: [obm-l] Re: [obm-l] Teoria dos números

2016-08-29 Por tôpico Pedro José
Boa noite!

Infelizmente sua conjectura só valeria se a,b,c,d < q,n e não ao produto
como está escrito.

Não sei como achar todas famílias de solução,mas aqui vão algumas.


a= 2, b=5, c=4 e d =7 para n=1 e q=8. atende a restrição pois qn=8.

1/2 + 5/8 = 1/4 + 7/8.

e poderíamos fazer todos os pares d-b= 2, com b,d pertencentes a |N e d<8.

a=4, b=6, c=5, d=8, pra n=2 e q=20, atende a restrição.

1/4 + 6/40 = 1/5 + 8/40 e ainda atenderiam d-b = 2 com b,d pertencentes a
|N e d<40.


a=4, b = 33, c= 10, d= 39 para n = 2 e q = 20, atende a restrição,
claramente.

1/4 + 33/40 = 1/10 +39/40.

a = 144, b =0, c= 500, d = 356 para n=6 e q=600, n=5 ==> n! = 120 e qn! =
72.000

1/44 + 0 = 1/500 + 356/72.000 e também para d-b = 356 e b < 3.000


A mesma solução acima vale para n=6 e q = 100 e vale também para d-b= 356 e
d < 600.

Sempre que n<< q e q for um primo elevado aum expoente maior ou igual a 3,
ou tiver pelo menos dois fatores primos diferentes haverá uma solução com a
<>c e b<>d. não sei se é necessário mais é suficiente. Podem haver outras
famílias.

Saudações,
PJMS




Em 29 de agosto de 2016 13:35, Pedro José  escreveu:

> Bom dia!
>
> está errado.
> Eu havia lido que errado que n e q eram superiores à a,b,c,d e é o produto
> qn que é não vale. Tenho que refazer, se conseguir.
>
> Saudações,
> PJMS
>
>
> Em 29 de agosto de 2016 10:43, Israel Meireles Chrisostomo <
> israelmchrisost...@gmail.com> escreveu:
>
>> Muito obrigado PJMS
>>
>> Em 29 de agosto de 2016 09:34, Pedro José  escreveu:
>>
>>> Bom dia!
>>>
>>> A igualdade torna-se: (a-c) n! q = (b-d) ac
>>> então temos que ter |a-c| n! q = |b-d| ac
>>>
>>> Para a<>c temos que;
>>> |b-d| < q (i)
>>> (ii) ac < |a-c| n!, pois, min(|a-c|) = 1 e n! >= n(n-1) e n> max(a,c) e
>>> n-1>= max(a,c)
>>> (i) e (ii) ==> |a-c| n! q > |b-d| ac se a -c <>0
>>> Então só há solução se a-c = 0 e portanto ==> b-d =0. Daí a sua solução.
>>>
>>> Saudações,
>>> PJMS
>>>
>>>
>>>
>>> Em 25 de agosto de 2016 00:09, Israel Meireles Chrisostomo <
>>> israelmchrisost...@gmail.com> escreveu:
>>>
 Olá pessoal.Como provar que se a,b,c,d>>
>>>
>>>
>>> --
>>> Esta mensagem foi verificada pelo sistema de antivírus e
>>> acredita-se estar livre de perigo.
>>
>>
>>
>> --
>> Esta mensagem foi verificada pelo sistema de antivírus e
>> acredita-se estar livre de perigo.
>>
>
>

-- 
Esta mensagem foi verificada pelo sistema de antiv�rus e
 acredita-se estar livre de perigo.



[obm-l] Re: [obm-l] Re: [obm-l] Re: [obm-l] Teoria dos números

2016-08-29 Por tôpico Pedro José
Bom dia!

está errado.
Eu havia lido que errado que n e q eram superiores à a,b,c,d e é o produto
qn que é não vale. Tenho que refazer, se conseguir.

Saudações,
PJMS


Em 29 de agosto de 2016 10:43, Israel Meireles Chrisostomo <
israelmchrisost...@gmail.com> escreveu:

> Muito obrigado PJMS
>
> Em 29 de agosto de 2016 09:34, Pedro José  escreveu:
>
>> Bom dia!
>>
>> A igualdade torna-se: (a-c) n! q = (b-d) ac
>> então temos que ter |a-c| n! q = |b-d| ac
>>
>> Para a<>c temos que;
>> |b-d| < q (i)
>> (ii) ac < |a-c| n!, pois, min(|a-c|) = 1 e n! >= n(n-1) e n> max(a,c) e
>> n-1>= max(a,c)
>> (i) e (ii) ==> |a-c| n! q > |b-d| ac se a -c <>0
>> Então só há solução se a-c = 0 e portanto ==> b-d =0. Daí a sua solução.
>>
>> Saudações,
>> PJMS
>>
>>
>>
>> Em 25 de agosto de 2016 00:09, Israel Meireles Chrisostomo <
>> israelmchrisost...@gmail.com> escreveu:
>>
>>> Olá pessoal.Como provar que se a,b,c,d>> naturais tais que
>>> 1/a+b/(n!q)=1/c+d/(n!q)
>>>
>>> a não ser que a=c e  e b=d.
>>>
>>> --
>>> Esta mensagem foi verificada pelo sistema de antivírus e
>>> acredita-se estar livre de perigo.
>>
>>
>>
>> --
>> Esta mensagem foi verificada pelo sistema de antivírus e
>> acredita-se estar livre de perigo.
>
>
>
> --
> Esta mensagem foi verificada pelo sistema de antivírus e
> acredita-se estar livre de perigo.
>

-- 
Esta mensagem foi verificada pelo sistema de antiv�rus e
 acredita-se estar livre de perigo.



[obm-l] RE: [obm-l] RE: [obm-l] Re: [obm-l] Teoria dos números

2013-10-27 Por tôpico marcone augusto araújo borges
Saulo,eu acho que vc mostrou duas soluções,mas não mostrou que são as únicas.

From: marconeborge...@hotmail.com
To: obm-l@mat.puc-rio.br
Subject: [obm-l] RE: [obm-l] Re: [obm-l] Teoria dos números
Date: Sun, 27 Oct 2013 09:16:24 +




Por que n deve ser ímpar?

Date: Sat, 26 Oct 2013 14:23:19 -0200
Subject: [obm-l] Re: [obm-l] Teoria dos números
From: saulo.nil...@gmail.com
To: obm-l@mat.puc-rio.br

(m-1)(m+1)=n2^(n-1)m deve ser imnpar senao nao consegue multiplos de 2n deve 
ser impar tambem.
m=1 e n=0 satisfazencontrando o numero impar do lado e3squerdo que e um fator 
de m entao encontramos o valor de m

m-1=2^x*vm+1=2^y*bvb=nx+y=n-1vb=1+x+y2=b2^y-v2^x2=b2^y-(1+x+y)2^x/bb^22^y-2b-(1+x+y)2^x=0delta=4+42^y2^x(1+x+y)
b=(1+sqrt(1+2^t(1+t)))/2^yt=x+yb deve ser inteiro e impart=4^, 
y=1b=10/2=5v=1x=3m=9n=5



2013/10/23 marcone augusto araújo borges marconeborge...@hotmail.com




Determine todas as soluções de n.2^(n-1) + 1 = m^2,m e n naturais.



 
 



  


  


  


  


 
 



  


  


  


  

  
--

Esta mensagem foi verificada pelo sistema de antivírus e 

 acredita-se estar livre de perigo.





--

Esta mensagem foi verificada pelo sistema de antivírus e 

 acredita-se estar livre de perigo.   
--

Esta mensagem foi verificada pelo sistema de antivírus e 

 acredita-se estar livre de perigo.   
-- 
Esta mensagem foi verificada pelo sistema de antivírus e
 acredita-se estar livre de perigo.



[obm-l] Re: [obm-l] Re: [obm-l] RE: [obm-l] Re: [obm-l] Teoria dos números

2013-08-31 Por tôpico Johann Dirichlet

Em 30-08-2013 10:29, Ralph Teixeira escreveu:


Ah, droga, bobeei. Nao ajudou tanto quanto eu achava... :-( :-(



Mas o caminho deve ser este. Que tal o famigerado módulo 49? Afinal esse 
monte de primos incita raízes primitivas...


On Aug 29, 2013 12:23 PM, marcone augusto araújo borges 
marconeborge...@hotmail.com mailto:marconeborge...@hotmail.com wrote:


7^x modulo 9 dá 1,7 e 4 e 3^y dá quase sempre 0
O que interessa  para 7^x modulo 9 é 4,o que ocorre apenas quando
x é da forma 3.k + 2
Como x tambem é ímpar,só pode ser da forma 6.n + 5,mas...


Date: Thu, 29 Aug 2013 09:21:24 -0300
Subject: [obm-l] Re: [obm-l] Teoria dos números
From: ralp...@gmail.com mailto:ralp...@gmail.com
To: obm-l@mat.puc-rio.br mailto:obm-l@mat.puc-rio.br

Tente agora modulo 9.

On Aug 28, 2013 9:50 PM, marcone augusto araújo borges
marconeborge...@hotmail.com mailto:marconeborge...@hotmail.com
wrote:

Eu já postei a questão aqui,mas infelizmente não obtive resposta.
Sei que vão aparecendo outras questões interessantes e por
isso peço licença para reapresentá-la
Determine todos os inteiros positivos x,y tais que 7^x - 3^y = 4

Claro que x = 1 e y = 1 satisfaz(desconfio que seja a unica
solução)
Eu só consegui concluir que x e y são ímpares,analisando módulo 4.
Desde já agradeço.
[Upload Photo to Facebook]
[Google+]
[Twitt]
[Send by Gmail]
[Upload Video to Facebook]
[Google+]
[Twitt]
[Send by Gmail]


-- 
Esta mensagem foi verificada pelo sistema de antivírus e

acredita-se estar livre de perigo.


-- 
Esta mensagem foi verificada pelo sistema de antivírus e

acredita-se estar livre de perigo.

-- 
Esta mensagem foi verificada pelo sistema de antivírus e

acredita-se estar livre de perigo.


--
Esta mensagem foi verificada pelo sistema de antivírus e
acredita-se estar livre de perigo. 



--
Esta mensagem foi verificada pelo sistema de antivírus e
acredita-se estar livre de perigo.



[obm-l] Re: [obm-l] RE: [obm-l] Re: [obm-l] Teoria dos números

2013-08-30 Por tôpico Ralph Teixeira
Ah, droga, bobeei. Nao ajudou tanto quanto eu achava... :-( :-(
On Aug 29, 2013 12:23 PM, marcone augusto araújo borges 
marconeborge...@hotmail.com wrote:

 7^x modulo 9 dá 1,7 e 4 e 3^y dá quase sempre 0
 O que interessa  para 7^x modulo 9 é 4,o que ocorre apenas quando x é da
 forma 3.k + 2
 Como x tambem é ímpar,só pode ser da forma 6.n + 5,mas...

 --
 Date: Thu, 29 Aug 2013 09:21:24 -0300
 Subject: [obm-l] Re: [obm-l] Teoria dos números
 From: ralp...@gmail.com
 To: obm-l@mat.puc-rio.br

 Tente agora modulo 9.
 On Aug 28, 2013 9:50 PM, marcone augusto araújo borges 
 marconeborge...@hotmail.com wrote:

 Eu já postei a questão aqui,mas infelizmente não obtive resposta.
 Sei que vão aparecendo outras questões interessantes e por isso peço
 licença para reapresentá-la

 Determine todos os inteiros positivos x,y tais que 7^x - 3^y = 4

 Claro que x = 1 e y = 1 satisfaz(desconfio que seja a unica solução)
 Eu só consegui concluir que x e y são ímpares,analisando módulo 4.
 Desde já agradeço.


 --
 Esta mensagem foi verificada pelo sistema de antivírus e
 acredita-se estar livre de perigo.


 --
 Esta mensagem foi verificada pelo sistema de antivírus e
 acredita-se estar livre de perigo.

 --
 Esta mensagem foi verificada pelo sistema de antivírus e
 acredita-se estar livre de perigo.


-- 
Esta mensagem foi verificada pelo sistema de antivírus e
 acredita-se estar livre de perigo.



[obm-l] Re: [obm-l] Re: [obm-l] RE: [obm-l] Re: [obm-l] Teoria dos números

2013-08-30 Por tôpico Ralph Teixeira
Acho que voce pensou em 7^x como multiplicacao - ele quer potencia...:-(
:-(
 On Aug 29, 2013 9:17 PM, Eduardo Wilner eduardowil...@yahoo.com.br
wrote:

 Observe que (1 + 3k , 1 + 7k) , k inteiro, satisfaz a equação diofantina

 [ ]'s


   --
  *De:* marcone augusto araújo borges marconeborge...@hotmail.com
 *Para:* obm-l@mat.puc-rio.br obm-l@mat.puc-rio.br
 *Enviadas:* Quinta-feira, 29 de Agosto de 2013 12:18
 *Assunto:* [obm-l] RE: [obm-l] Re: [obm-l] Teoria dos números

 7^x modulo 9 dá 1,7 e 4 e 3^y dá quase sempre 0
 O que interessa  para 7^x modulo 9 é 4,o que ocorre apenas quando x é da
 forma 3.k + 2
 Como x tambem é ímpar,só pode ser da forma 6.n + 5,mas...

 --
 Date: Thu, 29 Aug 2013 09:21:24 -0300
 Subject: [obm-l] Re: [obm-l] Teoria dos números
 From: ralp...@gmail.com
 To: obm-l@mat.puc-rio.br

 Tente agora modulo 9.
 On Aug 28, 2013 9:50 PM, marcone augusto araújo borges 
 marconeborge...@hotmail.com wrote:

 Eu já postei a questão aqui,mas infelizmente não obtive resposta.
 Sei que vão aparecendo outras questões interessantes e por isso peço
 licença para reapresentá-la

 Determine todos os inteiros positivos x,y tais que 7^x - 3^y = 4

 Claro que x = 1 e y = 1 satisfaz(desconfio que seja a unica solução)
 Eu só consegui concluir que x e y são ímpares,analisando módulo 4.
 Desde já agradeço.


 --
 Esta mensagem foi verificada pelo sistema de antivírus e
 acredita-se estar livre de perigo.


 --
 Esta mensagem foi verificada pelo sistema de antivírus e
 acredita-se estar livre de perigo.

 --
 Esta mensagem foi verificada pelo sistema de antivírus e
 acredita-se estar livre de perigo.



 --
 Esta mensagem foi verificada pelo sistema de antivírus e
 acredita-se estar livre de perigo.


-- 
Esta mensagem foi verificada pelo sistema de antivírus e
 acredita-se estar livre de perigo.



[obm-l] Re: [obm-l] RE: [obm-l] Re: [obm-l] Teoria dos números

2013-08-29 Por tôpico Eduardo Wilner

Observe que (1 + 3k , 1 + 7k) , k inteiro, satisfaz a equação diofantina

[ ]'s





 De: marcone augusto araújo borges marconeborge...@hotmail.com
Para: obm-l@mat.puc-rio.br obm-l@mat.puc-rio.br 
Enviadas: Quinta-feira, 29 de Agosto de 2013 12:18
Assunto: [obm-l] RE: [obm-l] Re: [obm-l] Teoria dos números
 


 
7^x modulo 9 dá 1,7 e 4 e 3^y dá quase sempre 0
O que interessa  para 7^x modulo 9 é 4,o que ocorre apenas quando x é da forma 
3.k + 2
Como x tambem é ímpar,só pode ser da forma 6.n + 5,mas...




Date: Thu, 29 Aug 2013 09:21:24 -0300
Subject: [obm-l] Re: [obm-l] Teoria dos números
From: ralp...@gmail.com
To: obm-l@mat.puc-rio.br


Tente agora modulo 9.
On Aug 28, 2013 9:50 PM, marcone augusto araújo borges 
marconeborge...@hotmail.com wrote:

Eu já postei a questão aqui,mas infelizmente não obtive resposta.
Sei que vão aparecendo outras questões interessantes e por isso peço licença 
para reapresentá-la
 
Determine todos os inteiros positivos x,y tais que 7^x - 3^y = 4


Claro que x = 1 e y = 1 satisfaz(desconfio que seja a unica solução)
Eu só consegui concluir que x e y são ímpares,analisando módulo 4.
Desde já agradeço.
 


-- 
Esta mensagem foi verificada pelo sistema de antivírus e 
acredita-se estar livre de perigo. 
-- 
Esta mensagem foi verificada pelo sistema de antivírus e 
acredita-se estar livre de perigo.
-- 
Esta mensagem foi verificada pelo sistema de antivírus e 
acredita-se estar livre de perigo. 
-- 
Esta mensagem foi verificada pelo sistema de antiv�rus e
 acredita-se estar livre de perigo.



[obm-l] Re: [obm-l] RE: [obm-l] RE: [obm-l] TEORIA DOS NÚMEROS

2012-04-15 Por tôpico Hermann
Para deixar de receber o email, vc tem que responder corretamente a 15 questões 
em 30 dias corridos. Acho que esse é o regulamento atual.

rsrsrsrsrs
- Original Message - 
  From: Gabriel Franco 
  To: obm-l@mat.puc-rio.br 
  Sent: Saturday, March 24, 2012 1:57 PM
  Subject: [obm-l] RE: [obm-l] RE: [obm-l] TEORIA DOS NÚMEROS



  como faço para não receber mais esses emails ?


--
  From: joao_maldona...@hotmail.com
  To: obm-l@mat.puc-rio.br
  Subject: [obm-l] RE: [obm-l] TEORIA DOS NÚMEROS 
  Date: Sat, 24 Mar 2012 13:45:20 -0300


  Eu elaborei  uma solução que diria FEIA, na verdade uma bonita seria uma 
fatoração que em aparecesse facilmente o 97


  Enfim, fatorando o 111
  Chamando y de 333^555 + 555^333
  y = 111^333(3^333 333^222 + 5^333) = 111^333 ((3^5 111^2)^111 + (5^3)^111)
  Mas  (3^5 111^2)^111  = (243*14*14)^111 = 1 (mod 97)
  5^3 = 28
  (5^3)^111 = (28^3)^37 = -1 mod 97


  Logo y é divisível por 97


  []'s
  João



--
  Date: Sat, 24 Mar 2012 07:45:16 -0300
  Subject: [obm-l] TEORIA DOS NÚMEROS
  From: vanderma...@gmail.com
  To: obm-l@mat.puc-rio.br


  Pessoal, vejam a seguinte questão:
   
  Prove que 333^555 + 555^333 é múltiplo de 97.
   
  Tentei de tudo, mas não consegui.
   
  Um abraço,
   
  Vanderlei


[obm-l] Re: [obm-l] Re: [obm-l] Re: [obm-l] Re: [obm-l] RE: [obm-l] Re: [obm-l] RE: [obm-l] TEORIA DOS NÚMEROS

2012-03-25 Por tôpico Marcelo Salhab Brogliato
Vamos lá:
333^555 + 555^333 = 111^555 * 3^555 + 111^333 * 5^333 = 111^333 * 111^222 *
3^555 + 111^333 * 5^333

--

Como 97 é primo, pelo pequeno teorema de fermat, temos que: x^96 == 1 (mod
97).

Como 111 == 15 (mod 96) e 111 == 14 (mod 97), temos que: 111^111 == 14^15
(mod 97).
Mas, 14^2 == 2 (mod 97), então: 14^15 == 2^7 * 14 (mod 97) == 46

Então: 111^111 == 46 (mod 97).
Assim, 111^222 == 46^2 == 79 (mod 97).
Por fim, 111^333 == 46^3 == 79*46 == 45 (mod 97).

--

Voltando, temos: 111^333 * 111^222 * 3^555 + 111^333 * 5^333 == 45 * 79 *
3^555 + 45 * 5^333 (mod 97).

--

Como 555 == 75 (mod 96), temos: 3^555 == 3^75 (mod 97)
3^5 = 243 == 49 (mod 97), então: 3^75 == 49^15 (mod 97)
49^2 == 73 (mod 97), então: 49^15 == 73^7 * 49 (mod 97)
73^2 == 91 (mod 97), então: 73^7 * 49 == 91^3 * 73 * 49 == 70 (mod 97)
Enfim, 3^555 == 70 (mod 97)

Como 333 == 45 (mod 96), temos 5^333 == 5^45 (mod 97)
5^4 = 625 == 43 (mod 97), então: 5^45 == 43^11 * 5 (mod 97)
43^2 == 6 (mod 97), então: 43^11 * 5 == 6^5 * 43 * 5 = 36*36*6*43*5 == 45
(mod 97)
Enfim, 5^333 == 45 (mod 97)

--

Novamente, voltando, temos: 45 * 79 * 70 + 45 * 45 (mod 97).
Agora ficou fácil, hehe. =]

Concluindo, ficamos com 333^555 + 555^333 == 33 (mod 97).

Abraços,
Salhab




2012/3/24 Vanderlei * vanderma...@gmail.com

 *é mesmo? Então o enunciado está errado? Essa questão está no material do
 Poliedro, caderno do ITA número 1.

 *
 Em 24 de março de 2012 18:59, Bernardo Freitas Paulo da Costa 
 bernardo...@gmail.com escreveu:

 2012/3/24 Vanderlei * vanderma...@gmail.com:
  Pois é caro João, eu também cheguei nesse seu resto de 45. Mas vamos
  continuar na luta, alguma saída deve ter! A sua ideia parece ser muito
 boa,
  uma vez que o primeiro resultado é verdadeiro!
 O Maple 10 acha que

 333^555 + 555^333 mod 97 = 33...

 --
 Bernardo Freitas Paulo da Costa

 =
 Instruções para entrar na lista, sair da lista e usar a lista em
 http://www.mat.puc-rio.br/~obmlistas/obm-l.html
 =





[obm-l] Re: [obm-l] Re: [obm-l] Re: [obm-l] Re: [obm-l] Re: [obm-l] RE: [obm-l] Re: [obm-l] RE: [obm-l] TEORIA DOS NÚMEROS

2012-03-25 Por tôpico Vanderlei *
*obrigado Marcelo! Então o enunciado está errado mesmo! 97 não divide a
soma!

*
2012/3/25 Marcelo Salhab Brogliato msbro...@gmail.com

  Vamos lá:
 333^555 + 555^333 = 111^555 * 3^555 + 111^333 * 5^333 = 111^333 * 111^222
 * 3^555 + 111^333 * 5^333

 --

 Como 97 é primo, pelo pequeno teorema de fermat, temos que: x^96 == 1 (mod
 97).

 Como 111 == 15 (mod 96) e 111 == 14 (mod 97), temos que: 111^111 == 14^15
 (mod 97).
 Mas, 14^2 == 2 (mod 97), então: 14^15 == 2^7 * 14 (mod 97) == 46

 Então: 111^111 == 46 (mod 97).
 Assim, 111^222 == 46^2 == 79 (mod 97).
 Por fim, 111^333 == 46^3 == 79*46 == 45 (mod 97).

 --

 Voltando, temos: 111^333 * 111^222 * 3^555 + 111^333 * 5^333 == 45 * 79 *
 3^555 + 45 * 5^333 (mod 97).

 --

 Como 555 == 75 (mod 96), temos: 3^555 == 3^75 (mod 97)
 3^5 = 243 == 49 (mod 97), então: 3^75 == 49^15 (mod 97)
 49^2 == 73 (mod 97), então: 49^15 == 73^7 * 49 (mod 97)
 73^2 == 91 (mod 97), então: 73^7 * 49 == 91^3 * 73 * 49 == 70 (mod 97)
 Enfim, 3^555 == 70 (mod 97)

 Como 333 == 45 (mod 96), temos 5^333 == 5^45 (mod 97)
 5^4 = 625 == 43 (mod 97), então: 5^45 == 43^11 * 5 (mod 97)
 43^2 == 6 (mod 97), então: 43^11 * 5 == 6^5 * 43 * 5 = 36*36*6*43*5 == 45
 (mod 97)
 Enfim, 5^333 == 45 (mod 97)

 --

 Novamente, voltando, temos: 45 * 79 * 70 + 45 * 45 (mod 97).
 Agora ficou fácil, hehe. =]

 Concluindo, ficamos com 333^555 + 555^333 == 33 (mod 97).

 Abraços,
 Salhab




 2012/3/24 Vanderlei * vanderma...@gmail.com

 *é mesmo? Então o enunciado está errado? Essa questão está no material
 do Poliedro, caderno do ITA número 1.

 *
 Em 24 de março de 2012 18:59, Bernardo Freitas Paulo da Costa 
 bernardo...@gmail.com escreveu:

 2012/3/24 Vanderlei * vanderma...@gmail.com:
  Pois é caro João, eu também cheguei nesse seu resto de 45. Mas vamos
  continuar na luta, alguma saída deve ter! A sua ideia parece ser muito
 boa,
  uma vez que o primeiro resultado é verdadeiro!
 O Maple 10 acha que

 333^555 + 555^333 mod 97 = 33...

 --
 Bernardo Freitas Paulo da Costa

 =
 Instruções para entrar na lista, sair da lista e usar a lista em
 http://www.mat.puc-rio.br/~obmlistas/obm-l.html
 =






[obm-l] RE: [obm-l] Re: [obm-l] RE: [obm-l] TEORIA DOS NÚMEROS

2012-03-24 Por tôpico João Maldonado

3^5 = 243
111 = 111-97 = 14 (mod 97)
[]'sJoão

Date: Sat, 24 Mar 2012 14:16:40 -0300
Subject: [obm-l] Re: [obm-l] RE: [obm-l] TEORIA DOS NÚMEROS
From: vanderma...@gmail.com
To: obm-l@mat.puc-rio.br

João, não compreendi essa parte: (3^5 111^2)^111  = (243*14*14)^111 = 1 (mod 97)
 
Um abraço,
 
Vanderlei


2012/3/24 João Maldonado joao_maldona...@hotmail.com



Eu elaborei  uma solução que diria FEIA, na verdade uma bonita seria uma 
fatoração que em aparecesse facilmente o 97 


Enfim, fatorando o 111
Chamando y de 333^555 + 555^333
y = 111^333(3^333 333^222 + 5^333) = 111^333 ((3^5 111^2)^111 + (5^3)^111)
Mas  (3^5 111^2)^111  = (243*14*14)^111 = 1 (mod 97)
5^3 = 28
(5^3)^111 = (28^3)^37 = -1 mod 97


Logo y é divisível por 97


[]'s
João





Date: Sat, 24 Mar 2012 07:45:16 -0300
Subject: [obm-l] TEORIA DOS NÚMEROS
From: vanderma...@gmail.com
To: obm-l@mat.puc-rio.br



Pessoal, vejam a seguinte questão:
 
Prove que 333^555 + 555^333 é múltiplo de 97.
 
Tentei de tudo, mas não consegui.
 
Um abraço,
 
Vanderlei


  

[obm-l] RE: [obm-l] Re: [obm-l] RE: [obm-l] TEORIA DOS NÚMEROS

2012-03-24 Por tôpico João Maldonado

Na verdade a tática consiste em reduzir a casos cada vez menores. Agora que 
estou vendo o problema pela segunda vez acho que está errado.  Se eu não errei 
em nada (ate o ponto de 28^ 111, depois desse ponto eu passei a errar, hehe), 
(28^3)^37 teria que dar -1, mas não dá.
28^96 = 1 (mod 97) (tanto pelo teorema do totiente de euler ou pelo pequeno 
teorema de fermat, já que 97 é primo, logo28^111 = 28^15 = (28^3)^5 = 
(784*28)^5 = (8*28)^5 = (30)^5 = 900*900*30 =  9*9*30*100*100 = 81*30*3*3 = 
49*10*9 = 53.10 = 530 = 500+30 = 5*3 + 30 = 45, diferente do esperado
[]'sJoão
Date: Sat, 24 Mar 2012 17:48:09 -0300
Subject: [obm-l] Re: [obm-l] RE: [obm-l] TEORIA DOS NÚMEROS
From: vanderma...@gmail.com
To: obm-l@mat.puc-rio.br

Obrigado, mas ainda não vi com fez (5^3)^111 = (28^3)^37 = -1 mod 97
Como fez (28^3)^37? Na calculadora?
 
é muito grande!


 
2012/3/24 João Maldonado joao_maldona...@hotmail.com



Eu elaborei  uma solução que diria FEIA, na verdade uma bonita seria uma 
fatoração que em aparecesse facilmente o 97 


Enfim, fatorando o 111
Chamando y de 333^555 + 555^333
y = 111^333(3^333 333^222 + 5^333) = 111^333 ((3^5 111^2)^111 + (5^3)^111)
Mas  (3^5 111^2)^111  = (243*14*14)^111 = 1 (mod 97)
5^3 = 28
(5^3)^111 = (28^3)^37 = -1 mod 97


Logo y é divisível por 97


[]'s
João





Date: Sat, 24 Mar 2012 07:45:16 -0300
Subject: [obm-l] TEORIA DOS NÚMEROS
From: vanderma...@gmail.com
To: obm-l@mat.puc-rio.br



Pessoal, vejam a seguinte questão:
 
Prove que 333^555 + 555^333 é múltiplo de 97.
 
Tentei de tudo, mas não consegui.
 
Um abraço,
 
Vanderlei


  

[obm-l] Re: [obm-l] RE: [obm-l] Re: [obm-l] RE: [obm-l] TEORIA DOS NÚMEROS

2012-03-24 Por tôpico Vanderlei *
Pois é caro João, eu também cheguei nesse seu resto de 45. Mas vamos
continuar na luta, alguma saída deve ter! A sua ideia parece ser muito boa,
uma vez que o primeiro resultado é verdadeiro!

2012/3/24 João Maldonado joao_maldona...@hotmail.com

  Na verdade a tática consiste em reduzir a casos cada vez menores. Agora
 que estou vendo o problema pela segunda vez acho que está errado.  Se eu
 não errei em nada (ate o ponto de 28^ 111, depois desse ponto eu passei a
 errar, hehe), (28^3)^37 teria que dar -1, mas não dá.

 28^96 = 1 (mod 97) (tanto pelo teorema do totiente de euler ou pelo
 pequeno teorema de fermat, já que 97 é primo, logo
 28^111 = 28^15 = (28^3)^5 = (784*28)^5 = (8*28)^5 = (30)^5 = 900*900*30 =
  9*9*30*100*100 = 81*30*3*3 = 49*10*9 = 53.10 = 530 = 500+30 = 5*3 + 30 =
 45, diferente do esperado

 []'s
 João

  --
 Date: Sat, 24 Mar 2012 17:48:09 -0300
 Subject: [obm-l] Re: [obm-l] RE: [obm-l] TEORIA DOS NÚMEROS

 From: vanderma...@gmail.com
 To: obm-l@mat.puc-rio.br

 Obrigado, mas ainda não vi com fez (5^3)^111 = (28^3)^37 = -1 mod 97
 Como fez (28^3)^37? Na calculadora?

 é muito grande!



 2012/3/24 João Maldonado joao_maldona...@hotmail.com

  Eu elaborei  uma solução que diria FEIA, na verdade uma bonita seria
 uma fatoração que em aparecesse facilmente o 97

 Enfim, fatorando o 111
 Chamando y de 333^555 + 555^333
 y = 111^333(3^333 333^222 + 5^333) = 111^333 ((3^5 111^2)^111 + (5^3)^111)
 Mas  (3^5 111^2)^111  = (243*14*14)^111 = 1 (mod 97)
 5^3 = 28
 (5^3)^111 = (28^3)^37 = -1 mod 97

 Logo y é divisível por 97

 []'s
 João

  --
 Date: Sat, 24 Mar 2012 07:45:16 -0300
 Subject: [obm-l] TEORIA DOS NÚMEROS
 From: vanderma...@gmail.com
 To: obm-l@mat.puc-rio.br

 Pessoal, vejam a seguinte questão:

 *Prove que 333^555 + 555^333 é múltiplo de 97.*

 Tentei de tudo, mas não consegui.

 Um abraço,

 Vanderlei





[obm-l] Re: [obm-l] Re: [obm-l] RE: [obm-l] Re: [obm-l] RE: [obm-l] TEORIA DOS NÚMEROS

2012-03-24 Por tôpico Bernardo Freitas Paulo da Costa
2012/3/24 Vanderlei * vanderma...@gmail.com:
 Pois é caro João, eu também cheguei nesse seu resto de 45. Mas vamos
 continuar na luta, alguma saída deve ter! A sua ideia parece ser muito boa,
 uma vez que o primeiro resultado é verdadeiro!
O Maple 10 acha que

333^555 + 555^333 mod 97 = 33...

-- 
Bernardo Freitas Paulo da Costa

=
Instruções para entrar na lista, sair da lista e usar a lista em
http://www.mat.puc-rio.br/~obmlistas/obm-l.html
=


[obm-l] Re: [obm-l] Re: [obm-l] Re: [obm-l] RE: [obm-l] Re: [obm-l] RE: [obm-l] TEORIA DOS NÚMEROS

2012-03-24 Por tôpico Vanderlei *
*é mesmo? Então o enunciado está errado? Essa questão está no material do
Poliedro, caderno do ITA número 1.

*
Em 24 de março de 2012 18:59, Bernardo Freitas Paulo da Costa 
bernardo...@gmail.com escreveu:

 2012/3/24 Vanderlei * vanderma...@gmail.com:
  Pois é caro João, eu também cheguei nesse seu resto de 45. Mas vamos
  continuar na luta, alguma saída deve ter! A sua ideia parece ser muito
 boa,
  uma vez que o primeiro resultado é verdadeiro!
 O Maple 10 acha que

 333^555 + 555^333 mod 97 = 33...

 --
 Bernardo Freitas Paulo da Costa

 =
 Instruções para entrar na lista, sair da lista e usar a lista em
 http://www.mat.puc-rio.br/~obmlistas/obm-l.html
 =



[obm-l] RES: [obm-l] Re: [obm-l] RE: [obm-l] Re: [obm-l] Teoria dos Números

2011-03-23 Por tôpico Joâo Gabriel Preturlan
Olá, amigos!

Vi esse problema há algumas semanas e ele me tirou do sério, pq me parecia
mais simples do que de fato é.

Não sei se o amigo que propôs o problema conseguiu resolver pq não se
manifestou mais...

Então aqui vai uma solução (SPOILER ALERT!!!):

 

a=512=2^9

b=675=3^3*5^2

c=720=2^4*3^2*5

 

Agora, note que 2c^2=3ab  (2*2^8*3^4*5^2=3*2^9*3^3*5^2).

Usando isso, vou seguir a dica do Victor e vou tentar encontrar a expressão
x^3+y^3+z^3-3xyz=(x+y+z)(z^2+y^2+z^2-xy-xz-yz)

 

Assim, voltando para os a’s, b’s e c’s:

 

N = a^3 + b^3 + c^3 = a^3 + b^3 - c^3 + 2c^3 = a^3 + b^3 - c^3 +2c^2*c = a^3
+ b^3 - c^3 + 3abc

Agora, sendo x = -c (para ajustar o polinômio):

N = a^3 + b^3 + x^3 – 3abx = (a+b+x)(a^2+b^2+x^2-ab-bx-ax) = 467 * 1745209

 

467 é primo, então Alternativa E: 4+6+7 = 17.

 

Abraços!

 

João Gabriel Preturlan

 

De: owner-ob...@mat.puc-rio.br [mailto:owner-ob...@mat.puc-rio.br] Em nome
de Victor Hugo Rodrigues
Enviada em: domingo, 13 de março de 2011 01:01
Para: obm-l@mat.puc-rio.br
Assunto: [obm-l] Re: [obm-l] RE: [obm-l] Re: [obm-l] Teoria dos Números

 

Fatore a^3+b^3+c^3-3abc.

Em 12 de março de 2011 15:55, abelardo matias abelardo_92...@hotmail.com
escreveu:

Não consegui, fico ainda com duas parcelas e não sei mais como continuar!
Uma outra dica..

  _  

Date: Wed, 9 Mar 2011 20:03:58 -0300
Subject: [obm-l] Re: [obm-l] Teoria dos Números
From: victorhcr.victorh...@gmail.com
To: obm-l@mat.puc-rio.br
CC: mat.mo...@gmail.com



Essa é muito boa, hehehe... tenta chamar uns fatores dos números de a e de b
pra enxergar melhor a questão e vê se ele aparece nos outros.

Em 9 de março de 2011 08:34, Marcelo Costa mat.mo...@gmail.com escreveu:

Seja p o maior fator primo do número N = 512^3 +  675^3  +  720^3. A soma
dos algarismos de p é igual a:

 

a) 13

b) 14

c) 15

d) 16

e) 17

 

Agradeço desde já a atenção dada.

 

Marcelo.

 

 

  _  

No virus found in this message.
Checked by AVG - www.avg.com
Version: 10.0.1204 / Virus Database: 1498/3504 - Release Date: 03/13/11



[obm-l] RE: [obm-l] RES: [obm-l] Re: [obm-l] RE: [obm-l] Re: [obm-l] Teoria dos Números

2011-03-23 Por tôpico marcone augusto araújo borges

Eu já estava quase pedindo para alguem resolver o problema.Valeu!
 


From: jgpretur...@uol.com.br
To: obm-l@mat.puc-rio.br
Subject: [obm-l] RES: [obm-l] Re: [obm-l] RE: [obm-l] Re: [obm-l] Teoria dos 
Números
Date: Wed, 23 Mar 2011 21:23:02 -0300








Olá, amigos!
Vi esse problema há algumas semanas e ele me tirou do sério, pq me parecia mais 
simples do que de fato é.
Não sei se o amigo que propôs o problema conseguiu resolver pq não se 
manifestou mais...
Então aqui vai uma solução (SPOILER ALERT!!!):
 
a=512=2^9
b=675=3^3*5^2
c=720=2^4*3^2*5
 
Agora, note que 2c^2=3ab  (2*2^8*3^4*5^2=3*2^9*3^3*5^2).
Usando isso, vou seguir a dica do Victor e vou tentar encontrar a expressão 
x^3+y^3+z^3-3xyz=(x+y+z)(z^2+y^2+z^2-xy-xz-yz)
 
Assim, voltando para os a’s, b’s e c’s:
 
N = a^3 + b^3 + c^3 = a^3 + b^3 - c^3 + 2c^3 = a^3 + b^3 - c^3 +2c^2*c = a^3 + 
b^3 - c^3 + 3abc
Agora, sendo x = -c (para ajustar o polinômio):
N = a^3 + b^3 + x^3 – 3abx = (a+b+x)(a^2+b^2+x^2-ab-bx-ax) = 467 * 1745209
 
467 é primo, então Alternativa E: 4+6+7 = 17.
 
Abraços!
 
João Gabriel Preturlan
 

De: owner-ob...@mat.puc-rio.br [mailto:owner-ob...@mat.puc-rio.br] Em nome de 
Victor Hugo Rodrigues
Enviada em: domingo, 13 de março de 2011 01:01
Para: obm-l@mat.puc-rio.br
Assunto: [obm-l] Re: [obm-l] RE: [obm-l] Re: [obm-l] Teoria dos Números
 
Fatore a^3+b^3+c^3-3abc.

Em 12 de março de 2011 15:55, abelardo matias abelardo_92...@hotmail.com 
escreveu:

Não consegui, fico ainda com duas parcelas e não sei mais como continuar! Uma 
outra dica..



Date: Wed, 9 Mar 2011 20:03:58 -0300
Subject: [obm-l] Re: [obm-l] Teoria dos Números
From: victorhcr.victorh...@gmail.com
To: obm-l@mat.puc-rio.br
CC: mat.mo...@gmail.com




Essa é muito boa, hehehe... tenta chamar uns fatores dos números de a e de b 
pra enxergar melhor a questão e vê se ele aparece nos outros.

Em 9 de março de 2011 08:34, Marcelo Costa mat.mo...@gmail.com escreveu:
Seja p o maior fator primo do número N = 512^3 +  675^3  +  720^3. A soma dos 
algarismos de p é igual a:

 

a) 13

b) 14

c) 15

d) 16

e) 17

 

Agradeço desde já a atenção dada.

 

Marcelo.
 
 



No virus found in this message.
Checked by AVG - www.avg.com
Version: 10.0.1204 / Virus Database: 1498/3504 - Release Date: 03/13/11 
  

[obm-l] RE: [obm-l] RES: [obm-l] Re: [obm-l] RE: [obm-l] Re: [obm-l] Teoria dos Números

2011-03-23 Por tôpico João Maldonado

Olá João
Fiz um programa que todos os fatores primos e 1745209 = 229*7621, e 7621 é 
primo.
Acho que ainda falta fatorar a última  expressão.
[]'sJoão

From: jgpretur...@uol.com.br
To: obm-l@mat.puc-rio.br
Subject: [obm-l] RES: [obm-l] Re: [obm-l] RE: [obm-l] Re: [obm-l] Teoria dos 
Números
Date: Wed, 23 Mar 2011 21:23:02 -0300



Olá, amigos!Vi esse problema há algumas semanas e ele me tirou do sério, pq me 
parecia mais simples do que de fato é.Não sei se o amigo que propôs o problema 
conseguiu resolver pq não se manifestou mais...Então aqui vai uma solução 
(SPOILER ALERT!!!): a=512=2^9b=675=3^3*5^2c=720=2^4*3^2*5 Agora, note que 
2c^2=3ab  (2*2^8*3^4*5^2=3*2^9*3^3*5^2).Usando isso, vou seguir a dica do 
Victor e vou tentar encontrar a expressão 
x^3+y^3+z^3-3xyz=(x+y+z)(z^2+y^2+z^2-xy-xz-yz) Assim, voltando para os a’s, b’s 
e c’s: N = a^3 + b^3 + c^3 = a^3 + b^3 - c^3 + 2c^3 = a^3 + b^3 - c^3 +2c^2*c = 
a^3 + b^3 - c^3 + 3abcAgora, sendo x = -c (para ajustar o polinômio):N = a^3 + 
b^3 + x^3 – 3abx = (a+b+x)(a^2+b^2+x^2-ab-bx-ax) = 467 * 1745209 467 é primo, 
então Alternativa E: 4+6+7 = 17. Abraços! João Gabriel Preturlan De: 
owner-ob...@mat.puc-rio.br [mailto:owner-ob...@mat.puc-rio.br] Em nome de 
Victor Hugo Rodrigues
Enviada em: domingo, 13 de março de 2011 01:01
Para: obm-l@mat.puc-rio.br
Assunto: [obm-l] Re: [obm-l] RE: [obm-l] Re: [obm-l] Teoria dos Números Fatore 
a^3+b^3+c^3-3abc.Em 12 de março de 2011 15:55, abelardo matias 
abelardo_92...@hotmail.com escreveu:Não consegui, fico ainda com duas 
parcelas e não sei mais como continuar! Uma outra dica..Date: Wed, 9 Mar 2011 
20:03:58 -0300
Subject: [obm-l] Re: [obm-l] Teoria dos Números
From: victorhcr.victorh...@gmail.com
To: obm-l@mat.puc-rio.br
CC: mat.mo...@gmail.com

Essa é muito boa, hehehe... tenta chamar uns fatores dos números de a e de b 
pra enxergar melhor a questão e vê se ele aparece nos outros.Em 9 de março de 
2011 08:34, Marcelo Costa mat.mo...@gmail.com escreveu:Seja p o maior fator 
primo do número N = 512^3 +  675^3  +  720^3. A soma dos algarismos de p é 
igual a: a) 13b) 14c) 15d) 16e) 17 Agradeço desde já a atenção dada. Marcelo.  
No virus found in this message.
Checked by AVG - www.avg.com
Version: 10.0.1204 / Virus Database: 1498/3504 - Release Date: 03/13/11 
  

[obm-l] Re: [obm-l] RE: [obm-l] Re: [obm-l] Teoria dos Números

2011-03-13 Por tôpico Victor Hugo Rodrigues
Fatore a^3+b^3+c^3-3abc.

Em 12 de março de 2011 15:55, abelardo matias
abelardo_92...@hotmail.comescreveu:

  Não consegui, fico ainda com duas parcelas e não sei mais como continuar!
 Uma outra dica..

 --
 Date: Wed, 9 Mar 2011 20:03:58 -0300
 Subject: [obm-l] Re: [obm-l] Teoria dos Números
 From: victorhcr.victorh...@gmail.com
 To: obm-l@mat.puc-rio.br
 CC: mat.mo...@gmail.com


 Essa é muito boa, hehehe... tenta chamar uns fatores dos números de a e de
 b pra enxergar melhor a questão e vê se ele aparece nos outros.

 Em 9 de março de 2011 08:34, Marcelo Costa mat.mo...@gmail.com escreveu:

 Seja p o maior fator primo do número N = 512^3 +  675^3  +  720^3. A soma
 dos algarismos de p é igual a:

 a) 13
 b) 14
 c) 15
 d) 16
 e) 17

 Agradeço desde já a atenção dada.

 Marcelo.





[obm-l] RE: [obm-l] Re: [obm-l] Re: [obm-l] RE: [obm-l] Teoria dos números 2

2011-01-21 Por tôpico marcone augusto araújo borges

É para determinar o volume do buraco cilindro,não é?
 
 Date: Wed, 19 Jan 2011 13:22:05 -0200
 Subject: [obm-l] Re: [obm-l] Re: [obm-l] RE: [obm-l] Teoria dos números 2
 From: henrique.re...@gmail.com
 To: obm-l@mat.puc-rio.br
 
 Aplica-se semelhança para encontrar a altura da broca que perfurou o cone:
 
 (16-h)/2 = 16/6 -- 48-3h = 16 -- 3h = 32 -- h = 32/3
 
 V = (1/3)*(pi*4)*(32/3) = 128*pi/9
 
 Em 19 de janeiro de 2011 12:55, Ana Paula Almeida aps...@gmail.com escreveu:
 
 
  Quem puder dar uma ajuda no exercício abaixo :
 
  Uma broca de raio r = 2 perfura um cone circular reto de altura H = 16 e
  raio R = 6
  ao longo de seu eixo central. O resultado é um tronco de cone perfurado
  conforme
  ilustrado acima. O volume do buraco cilíndrico é então
 
 
 
 
 -- 
 Henrique
 
 =
 Instruções para entrar na lista, sair da lista e usar a lista em
 http://www.mat.puc-rio.br/~obmlistas/obm-l.html
 =
  

[obm-l] Re: [obm-l] Re: [obm-l] RE: [obm-l] Teoria dos números 2

2011-01-20 Por tôpico Henrique Rennó
Aplica-se semelhança para encontrar a altura da broca que perfurou o cone:

(16-h)/2 = 16/6 -- 48-3h = 16 -- 3h = 32 -- h = 32/3

V = (1/3)*(pi*4)*(32/3) = 128*pi/9

Em 19 de janeiro de 2011 12:55, Ana Paula Almeida aps...@gmail.com escreveu:


 Quem puder dar uma ajuda no exercício abaixo :

 Uma broca de raio r = 2 perfura um cone circular reto de altura H = 16 e
 raio R = 6
 ao longo de seu eixo central. O resultado é um tronco de cone perfurado
 conforme
 ilustrado acima. O volume do buraco cilíndrico é então




-- 
Henrique

=
Instruções para entrar na lista, sair da lista e usar a lista em
http://www.mat.puc-rio.br/~obmlistas/obm-l.html
=


[obm-l] Re: [obm-l] Re: [obm-l] Re: [obm-l] Re: [obm-l] Re: [obm-l] Teoria dos números

2010-12-21 Por tôpico Willy George do Amaral Petrenko
Claro, claro, foi um erro de tipografia.

2010/12/21 Henrique Rennó henrique.re...@gmail.com

 Minha dúvida é sobre o expoente do termo a^'pq - 2q', não seria a^'pq - 2p'
 ?

 Em 18/12/10, Willy George do Amaral Petrenkowgapetre...@gmail.com
 escreveu:
  Escreva num papel e veja algum caso particular. Por exemplo:
 
  a^5 + a^4 + a^3 + a^2 + a + 1 = a^3*(a^2 + a + 1) + a^2 + a + 1 = (a^3 +
  1)*(a^2 + a + 1)
 
  Repare que se n = 9, a primeira parcela ficaria (a^6 + a^3 + 1).
 
  2010/12/17 Henrique Rennó henrique.re...@gmail.com
 
  Não entendi como a^'n-1' + a^'n-2' + ... + a + 1 = (a^'pq - p' + a^'pq
  - 2q' + ... + a^p + 1)*(a^'p - 1' + a^'p - 2' + ... + a + 1).
 
  Em 17/12/10, Willy George do Amaral Petrenkowgapetre...@gmail.com
  escreveu:
   Observe que a^n - 1 = (a - 1)*(a^'n-1' + a^'n-2' + ... + a + 1). Se
 a^n
   -
  1
   é primo então a 1a parcela deve ser 1 (a 2a não pode para a0), e
 então
   a
  =
   2.
  
   Agora observe que se n = p*q então a^'n-1' + a^'n-2' + ... + a + 1 =
  (a^'pq
   - p' + a^'pq - 2q' + ... + a^p + 1)*(a^'p - 1' + a^'p - 2' + ... + a +
  1),
   ambas parcelas maiores que 1 para p,q 1.
   n composto =  a^n - 1 composto logo a^n - 1 primo = n primo.
  
  
  
   2010/12/16 marcone augusto araújo borges marconeborge...@hotmail.com
 
  
Mostre que se a e n são inteiros positivos,com n = 2 ,tais que a^n
 -
   1
  é
   primo,então necessariamente a = 2 e n é primo.
  
  
 
 
  --
  Henrique
 
 
 =
  Instruções para entrar na lista, sair da lista e usar a lista em
  http://www.mat.puc-rio.br/~obmlistas/obm-l.html
 
 =
 
 


 --
 Henrique

 =
 Instruções para entrar na lista, sair da lista e usar a lista em
 http://www.mat.puc-rio.br/~obmlistas/obm-l.html
 =



[obm-l] Re: [obm-l] RE: [obm-l] Re: [obm-l] Re : [obm-l] Re: [obm-l] RE: [obm-l] Teoria dos números (2 questões simples)

2009-08-24 Por tôpico luiz silva


Ola,
 
Tente isso..acho q funciona .
 
1) b^2+ab+1 = 0 mod (a^2+ab+1)
 
2) a^2+ab+1= 0 mod (a^2+ab+1)
 
Substiutua (2) em (1)
 
Abs
Felipe
 
--- Em sex, 21/8/09, marcone augusto araújo borges 
marconeborge...@hotmail.com escreveu:


De: marcone augusto araújo borges marconeborge...@hotmail.com
Assunto: [obm-l] RE: [obm-l] Re: [obm-l] Re: [obm-l] Re: [obm-l] RE: [obm-l] 
Teoria dos números (2 questões simples)
Para: obm-l@mat.puc-rio.br
Data: Sexta-feira, 21 de Agosto de 2009, 21:38




#yiv1877891977 #yiv1193512529 .hmmessage P
{
margin:0px;padding:0px;}
#yiv1877891977 #yiv1193512529 {
font-size:10pt;font-family:Verdana;}

Hugo esclareceu,obrigado.Mas o Diogo soicitou ajuda em outra questão: se 
a^2+ab+1 divide b^2+ab+1 então a=b.Alguém poderia ajudar?
 


Date: Fri, 21 Aug 2009 16:34:51 -0700
From: luizfelipec...@yahoo.com.br
Subject: [obm-l] Re: [obm-l] Re: [obm-l] Re: [obm-l] RE: [obm-l] Teoria dos 
números (2 questões simples)
To: obm-l@mat.puc-rio.br






Hugo,
 
Valeu!!
Abs
Felipe

--- Em sex, 21/8/09, Hugo Fernando Marques Fernandes hfernande...@gmail.com 
escreveu:


De: Hugo Fernando Marques Fernandes hfernande...@gmail.com
Assunto: [obm-l] Re: [obm-l] Re: [obm-l] RE: [obm-l] Teoria dos números (2 
questões simples)
Para: obm-l@mat.puc-rio.br
Data: Sexta-feira, 21 de Agosto de 2009, 16:14



Todos os números são da forma 3k+1, 3k ou 3k-1. Como 3k não é primo, k1, então 
todos os primos maiores que 3 são da forma 3k+1 ou 3k-1.
 
Abraços.
 
Hugo.


2009/8/21 luiz silva luizfelipec...@yahoo.com.br






Ola Marcone,
 
Pq vc trabalha com primos da forma 3k+1 ou 3k-1..creio que nem todos os primos 
podem ser representados nesta forma...O correto não seria 2k+1 ou 2k-1 ??Ou 
então, se quiser representar primos maiores que 3, não seria correto trabalhar 
com primos da forma 3+2k?
 
Abs
Felipe

--- Em qui, 20/8/09, marcone augusto araújo borges 
marconeborge...@hotmail.com escreveu:


De: marcone augusto araújo borges marconeborge...@hotmail.com
Assunto: [obm-l] RE: [obm-l] Teoria dos números (2 questões simples)
Para: obm-l@mat.puc-rio.br
Data: Quinta-feira, 20 de Agosto de 2009, 21:10 



Olá,Diogo.Um comentário singelo:o único primo múltiplo de 3 é o próprio 3.Se a 
é primo e diferente de 3, então a=3k+1 ou a=3k-1,dai a^2=3p+1 e a^2 +2=3q, o 
que é uma contradição(pois a^2+2 é primo).Portanto a=3.Se eu estiver 
errado,certamente alguem irá corrigir.Um abraço.
 


Date: Thu, 20 Aug 2009 15:15:39 -0700
From: diog...@yahoo.com.br
Subject: [obm-l] Teoria dos números (2 questões simples)
To: obm-l@mat.puc-rio.br




Teoria dos números (2 questões simples)?
1. Mostre que se (a) e (a² + 2) são ambos primos então a=3
2. Mostre que se (a² +ab +1) divide (b² +ab + 1) então a=b.


Se puder ajudar, agradeço.


Veja quais são os assuntos do momento no Yahoo! + Buscados: Top 10 - 
Celebridades - Música - Esportes


Novo Internet Explorer 8: mais rápido e muito mais seguro. Baixe agora, é 
grátis! 





Veja quais são os assuntos do momento no Yahoo! + Buscados: Top 10 - 
Celebridades - Música - Esportes



Veja quais são os assuntos do momento no Yahoo! + Buscados: Top 10 - 
Celebridades - Música - Esportes


Instale o novo Internet Explorer 8 versão especial para o MSN. Download aqui 


  

Veja quais são os assuntos do momento no Yahoo! +Buscados
http://br.maisbuscados.yahoo.com

[obm-l] Re: [obm-l] Re: [obm-l] Re: [obm-l] RE : [obm-l] Teoria dos números (2 questões simpl es)

2009-08-21 Por tôpico luiz silva
Hugo,
 
Valeu!!
Abs
Felipe

--- Em sex, 21/8/09, Hugo Fernando Marques Fernandes hfernande...@gmail.com 
escreveu:


De: Hugo Fernando Marques Fernandes hfernande...@gmail.com
Assunto: [obm-l] Re: [obm-l] Re: [obm-l] RE: [obm-l] Teoria dos números (2 
questões simples)
Para: obm-l@mat.puc-rio.br
Data: Sexta-feira, 21 de Agosto de 2009, 16:14



Todos os números são da forma 3k+1, 3k ou 3k-1. Como 3k não é primo, k1, então 
todos os primos maiores que 3 são da forma 3k+1 ou 3k-1.
 
Abraços.
 
Hugo.


2009/8/21 luiz silva luizfelipec...@yahoo.com.br






Ola Marcone,
 
Pq vc trabalha com primos da forma 3k+1 ou 3k-1..creio que nem todos os primos 
podem ser representados nesta forma...O correto não seria 2k+1 ou 2k-1 ??Ou 
então, se quiser representar primos maiores que 3, não seria correto trabalhar 
com primos da forma 3+2k?
 
Abs
Felipe

--- Em qui, 20/8/09, marcone augusto araújo borges 
marconeborge...@hotmail.com escreveu:


De: marcone augusto araújo borges marconeborge...@hotmail.com
Assunto: [obm-l] RE: [obm-l] Teoria dos números (2 questões simples)
Para: obm-l@mat.puc-rio.br
Data: Quinta-feira, 20 de Agosto de 2009, 21:10 



Olá,Diogo.Um comentário singelo:o único primo múltiplo de 3 é o próprio 3.Se a 
é primo e diferente de 3, então a=3k+1 ou a=3k-1,dai a^2=3p+1 e a^2 +2=3q, o 
que é uma contradição(pois a^2+2 é primo).Portanto a=3.Se eu estiver 
errado,certamente alguem irá corrigir.Um abraço.
 


Date: Thu, 20 Aug 2009 15:15:39 -0700
From: diog...@yahoo.com.br
Subject: [obm-l] Teoria dos números (2 questões simples)
To: obm-l@mat.puc-rio.br




Teoria dos números (2 questões simples)?
1. Mostre que se (a) e (a² + 2) são ambos primos então a=3
2. Mostre que se (a² +ab +1) divide (b² +ab + 1) então a=b.


Se puder ajudar, agradeço.


Veja quais são os assuntos do momento no Yahoo! + Buscados: Top 10 - 
Celebridades - Música - Esportes


Novo Internet Explorer 8: mais rápido e muito mais seguro. Baixe agora, é 
grátis! 





Veja quais são os assuntos do momento no Yahoo! + Buscados: Top 10 - 
Celebridades - Música - Esportes



  

Veja quais são os assuntos do momento no Yahoo! +Buscados
http://br.maisbuscados.yahoo.com

[obm-l] RE: [obm-l] Re: [obm -l] Re: [obm-l] Re: [obm-l] RE: [obm-l] Teoria dos números ( 2 questões simples)

2009-08-21 Por tôpico marcone augusto araújo borges

Hugo esclareceu,obrigado.Mas o Diogo soicitou ajuda em outra questão: se 
a^2+ab+1 divide b^2+ab+1 então a=b.Alguém poderia ajudar?
 


Date: Fri, 21 Aug 2009 16:34:51 -0700
From: luizfelipec...@yahoo.com.br
Subject: [obm-l] Re: [obm-l] Re: [obm-l] Re: [obm-l] RE: [obm-l] Teoria dos 
números (2 questões simples)
To: obm-l@mat.puc-rio.br






Hugo,
 
Valeu!!
Abs
Felipe

--- Em sex, 21/8/09, Hugo Fernando Marques Fernandes hfernande...@gmail.com 
escreveu:


De: Hugo Fernando Marques Fernandes hfernande...@gmail.com
Assunto: [obm-l] Re: [obm-l] Re: [obm-l] RE: [obm-l] Teoria dos números (2 
questões simples)
Para: obm-l@mat.puc-rio.br
Data: Sexta-feira, 21 de Agosto de 2009, 16:14



Todos os números são da forma 3k+1, 3k ou 3k-1. Como 3k não é primo, k1, então 
todos os primos maiores que 3 são da forma 3k+1 ou 3k-1.
 
Abraços.
 
Hugo.


2009/8/21 luiz silva luizfelipec...@yahoo.com.br






Ola Marcone,
 
Pq vc trabalha com primos da forma 3k+1 ou 3k-1..creio que nem todos os primos 
podem ser representados nesta forma...O correto não seria 2k+1 ou 2k-1 ??Ou 
então, se quiser representar primos maiores que 3, não seria correto trabalhar 
com primos da forma 3+2k?
 
Abs
Felipe

--- Em qui, 20/8/09, marcone augusto araújo borges 
marconeborge...@hotmail.com escreveu:


De: marcone augusto araújo borges marconeborge...@hotmail.com
Assunto: [obm-l] RE: [obm-l] Teoria dos números (2 questões simples)
Para: obm-l@mat.puc-rio.br
Data: Quinta-feira, 20 de Agosto de 2009, 21:10 



Olá,Diogo.Um comentário singelo:o único primo múltiplo de 3 é o próprio 3.Se a 
é primo e diferente de 3, então a=3k+1 ou a=3k-1,dai a^2=3p+1 e a^2 +2=3q, o 
que é uma contradição(pois a^2+2 é primo).Portanto a=3.Se eu estiver 
errado,certamente alguem irá corrigir.Um abraço.
 


Date: Thu, 20 Aug 2009 15:15:39 -0700
From: diog...@yahoo.com.br
Subject: [obm-l] Teoria dos números (2 questões simples)
To: obm-l@mat.puc-rio.br




Teoria dos números (2 questões simples)?
1. Mostre que se (a) e (a² + 2) são ambos primos então a=3
2. Mostre que se (a² +ab +1) divide (b² +ab + 1) então a=b.


Se puder ajudar, agradeço.


Veja quais são os assuntos do momento no Yahoo! + Buscados: Top 10 - 
Celebridades - Música - Esportes


Novo Internet Explorer 8: mais rápido e muito mais seguro. Baixe agora, é 
grátis! 





Veja quais são os assuntos do momento no Yahoo! + Buscados: Top 10 - 
Celebridades - Música - Esportes



Veja quais são os assuntos do momento no Yahoo! + Buscados: Top 10 - 
Celebridades - Música - Esportes
_
Novo Internet Explorer 8. Baixe agora, é grátis!
http://brasil.microsoft.com.br/IE8/mergulhe/?utm_source=MSN%3BHotmailutm_medium=Taglineutm_campaign=IE8

[obm-l] Re: [obm-l] Re: [obm-l] Re: [obm-l] Re: [obm-l] Re: [obm-l] teoria dos números

2009-03-31 Por tôpico Bernardo Freitas Paulo da Costa
2009/3/30 Ralph Teixeira ralp...@gmail.com:

 Tá, eu confesso: comprei o Scientific Workplace, que faz estas contas
 na boa. Tenho certeza que há outros pacotes matemáticos grátis por aí
 que também fazem estas contas grandes.

 Abraço,
        Ralph

Eu usei o bc (gratis, vem com provavelmente todos os linux do mundo,
em linha de comando, basta escrever 5^50 e ele mostra quanto vale, dai
copia de uma linha pra outra botando +) pra fazer a conta, mas
confesso que eu primeiro resolvi o problema de somar tudo varias vezes
antes de pedir pra ele confirmar que o resto mod 9 era o que eu tinha
encontrado. O que sempre é bom de fazer pra ter certeza de nao ter
deixado passar uma besteirinha !

-- 
Bernardo Freitas Paulo da Costa

=
Instruções para entrar na lista, sair da lista e usar a lista em
http://www.mat.puc-rio.br/~obmlistas/obm-l.html
=


[obm-l] Re: [obm-l] Re: [obm-l] Re: [obm-l] Teoria dos Números

2003-08-16 Por tôpico Eduardo Casagrande Stabel
Deixem eu me corrigir!

Se eu descobrir um outro erro, deixarei que outros me corrijam, para eu não
entrar num loop de auto-correções. A resposta que o Cláudio deu está certa,
vou explicar o porquê. Na verdade, não se precisa usar polinômios na
solução.

Segundo o que o Cláudio - pelo teorema de Euler - escreveu p(a) = a^n - 1
divide q(a) = a^Phi(a^n - 1) - 1. Suponhamos que n não divide Phi(a^n - 1),
então

Phi(a^n - 1) = qn + r sendo 0  r  n

Daí

q(a) = a^(qn + r) - 1 = a^r * a^(qn) - a^r + a^r - 1 = a^r * (a^(qn) - 1) +
(a^r - 1)

É claro que p(a) = a^n - 1 divide a^(qn) - 1, portanto a^n - 1 deve dividir
a^r - 1, só que este último número é menor do que a^n - 1, uma contradição.

Obrigado!
Duda.

From: Eduardo Casagrande Stabel [EMAIL PROTECTED]
 Oi Yuri.

 Eu acho que você tem razão.

 Fixando n, nós temos duas expressões

 p(a) = a^n - 1

 q(a) = a^Phi(a^n - 1) - 1

 A primeira é um polinômio de grau n, a segunda não tem cara de ser um
 polinômio (de fato, fazendo a crescer, ela cresce na forma a^a, que é
muito
 mais veloz do que um polinômio, portanto não pode ser um polinômio). O
 Dirichlet intuiu que nós podemos chegar a uma expressão relação do tipo
 t^n-1 | t^(fi(a^n-1))-1, mas não sei como se chegaria a isso. O caminho
que
 o Cláudio usou - utilizando o teorema de Euler - colocará o t dentro do
 expoente que tem o Phi.

 Acho que ainda está por resolver.

 Duda.

 From: [EMAIL PROTECTED]
 
  Oi Claudio,
 
  Eu não entendi pq vc considerou polinômios para provar a última
passagem,
  jah que a está fixo. Ou seja, vc tem que
  a^n - 1 divide a^Phi(a^n - 1) - 1
   e não que x^n-1 divide x^Phi(a^n - 1) - 1 para todo x.
 Se eu tiver falado alguma besteira, me avisem!
Ateh mais,
   Yuri
  -- Mensagem original --
 
  on 16.08.03 05:54, Eduardo Casagrande Stabel at [EMAIL PROTECTED]
 wrote:
  
   Olá pessoal!
  
   Prove que se n  1 e a  0 são inteiros então n | PHY(a^n - 1).
  
   PHY é a função de Euler.
  
   Abraço,
   Duda.
  
  
  Oi, Duda:
  
  Eh claro que mdc(a,a^n - 1) = 1
  
  Entao, pelo teorema de Euler, teremos:
  a^Phi(a^n - 1) == 1 (mod a^n - 1) ==
  
  a^n - 1 divide a^Phi(a^n - 1) - 1 ==
  
  n divide Phi(a^n - 1)
  
  ***
  
  Essa ultima passagem pode ser vista da seguinte forma:
  
  Sejam x^n - 1 e x^n - 1 polinomios (portanto m, n inteiros)
  
  x^n - 1 divide x^m - 1 mas n nao divide m ==
  
  m = qn + r com 0  r = n-1 ==
  
  x^m - 1 = x^(qn + r) - 1 = x^(qn)*x^r - x^r + x^r - 1 =
  = x^r(x^(qn) - 1) + x^r - 1 ==
  
  x^n - 1 divide x^r - 1 com 0  r  n ==
  
  contradicao.
  
  
  Um abraco,
  Claudio.
  
 
=

  Instruções para entrar na lista, sair da lista e usar a lista em
  http://www.mat.puc-rio.br/~nicolau/olimp/obm-l.html
 
=
  
 
  []'s, Yuri
  ICQ: 64992515
 
 
  --
  Use o melhor sistema de busca da Internet
  Radar UOL - http://www.radaruol.com.br
 
 
 
 
=
  Instruções para entrar na lista, sair da lista e usar a lista em
  http://www.mat.puc-rio.br/~nicolau/olimp/obm-l.html
 
=
 
 

 =
 Instruções para entrar na lista, sair da lista e usar a lista em
 http://www.mat.puc-rio.br/~nicolau/olimp/obm-l.html
 =



=
Instruções para entrar na lista, sair da lista e usar a lista em
http://www.mat.puc-rio.br/~nicolau/olimp/obm-l.html
=